Re: [Logica-l] Re: Meio off-topic: Construção do Reino de Deus na UFRN

2022-12-16 Por tôpico Ricardo Pereira
Oi, Cassiano!
Seguem meus comentários.


>> Nesse caso, muito do que é não-ciência não será confundido com ela (nem
>>> tem essa pretensão) e, consequentemente, não haverá toda uma gama de
>>> problemas associados a essa confusão.
>>>
>>
> Eu argumento em outro sentido (acho). Os problemas que tenho em vista
> dizem respeito à sobreposição de finalidades.
>

Então entendo que para cada finalidade uma determinada distinção se mostra
apropriada. Podemos pensar em ciência/pseudo-ciência e também em
ciência/não-ciência.


> A ciência nos diz que podemos fazer muitas coisas, inclusive conhecer os
> fenômenos de maneira mais, ou menos, confiável do que a religião ou a
> política (quer dizer, ao menos, pretensamente mais confiável…).
>

Não é nitidamente mais confiável?


> Mas até onde consigo entender, a ciência não nos diz pq deveríamos fazer x
> ou y. A religião e a política fazem isso? São elas q devem fazer isso? Não
> sei. Sei q um modelo como o do CNRS da França pode ser belo, ma non
> funziona in tutti quanti i paesi…
>
> Da maneira como a entendo, a ciência não faz isso, ainda que possa nos
fazer mudar de ideia porque pode revelar conflitos entre objetivos ou
consequências e possibilidades não previstas antes.


>
> Já o caso de atividades que se parecem muito com a ciência, em todas as
>> suas características superficiais, mas não têm poder de previsão, por
>> exemplo, merecem ser tratadas em separado justamente porque muitas pessoas
>> serão enganadas (nem precisamos entrar no mérito de se há má fé ou não).
>>
>
> Aqui me parece q vc está usando o poder de previsão como característica
> definidora da ciência. Eu diria o seguinte, concordo, mas formularia
> diferentemente: o q torna uma teoria científica é q ela afirma q deve haver
> alguma coisa, algum funcionamento dos fenômenos, por trás (digamos) da
> teoria q sustenta as previsões e q lá na frente condiz com as evidências q
> somos capazes de ter.  Acho q é fundamental, então, distinguir entre o q
> simplesmente é literário, político, religioso (não cientifico) e o q é
> pseudocientífico, ou ciência picareta, como já vi escreverem. Mas esse
> limite “denorex” - “parece, mas não é” - é impossível definir
> absolutamente, embora caso a caso talvez não.
>

Não sei se é impossível.
Que critério vc imagina que estaria sendo usado caso a caso, mas não seria
generalizável?


> Na história da ciência há inúmeros exemplos a citar de teorias q pareciam
> ser científicas, mas depois viu-se que não eram - ou ao menos deixaram de
> ser em vista de modificações metodológicas ou novas descobertas.
>

Acho que esse problema afetaria tanto a divisão
ciência/pseudo-ciência quanto a ciência/não-ciência. Vejo como uma
dificuldade, mas que não torna a tentativa desnecessária.


>
>  Melhor mesmo é vc aprender a capturar a cobra com segurança e devolvê-la
> ao mato sem matá-la; se for impossível reencaminhar a cobra, buscar
> entregá-la à polícia ambiental ou equivalente. E ter sempre de reserva,
> tanto qto possível, alguma dose de soro antiofídico.
> Reconhecer espécies não é tão difícil tb, mas exige tempo e nem sempre
> temos tempo - a disposição para aprender q anima a ciência, em muitos
> casos, não basta, é preciso agir, e o tempo da ação - assim como o do
> lumpen - é outro.
>

Na verdade eu capturei com sucesso (sem machucá-la). A dúvida era se
soltaria perto. Minha esposa preferiu que chamássemos os bombeiros.
Gostod e cobras e lagartos em geral. Já tentei criar calangos bico doce
(aqui tem bastante) e iguanas, mas tive medo dos últimos não comerem
direito e soltei.
Quanto à coral, vi que não é fácil saber quando é falsa, então não tentei.

>
>> A genuína disposição para aprender não seria uma condição necessária e
>> suficiente, então?
>>
>

> Eu não disse isso. Pode ser, mas em se tratando de uma atitude, mais do q
> qq outra coisa, não acho q seja disso q se trata.
>

Meu ponto é que a própria atitude genuína parece implicar que, uma vez
tendo reconhecido certos métodos nitidamente eficientes pra entender e
prever as coisas, há uma expectativa de que reconhecendo o mesmo contexto
eu deveria aplicar os mesmos métodos SE o que me move primordialmente ainda
é a disposição de aprender. Claro que ela é apenas uma das coisas que nos
move, mas aí entra justamente a possibilidade de nos deixar mover
conscientes dos reais motivos (até onde a gente consegue ou julga
suportável/desejável).


> Acho que isso é fruto da divisão extrema do trabalho. Parece que a gente
>> acaba relaxando em relação aos fundamentos racionais e filosóficos da
>> ciência justamente porque no dia a dia geralmente podemos passar sem eles.
>>
>>
>
> Concordo, só acho q não se trata de fundamentos filosfóficos ou racionais
> (descreio disso).
>

Interessante.
A descrença na razão em geral ou especificamente em relação aos fundamentos
(relaxamento quanto aos) terem algum papel nesse estado de coisas?



> A ênfase na disposição para aprender q eu defendo ressalta menos os
> métodos e mais uma 

Re: [Logica-l] Re: Meio off-topic: Construção do Reino de Deus na UFRN

2022-12-06 Por tôpico Ricardo Pereira
Obrigado, Julio!
Muito interessante o artigo!
:-)

On Tue, 6 Dec 2022 at 10:23, Julio Stern  wrote:

> Caro Ricardo:
> Segue a referencia  ``da Lanca e do Escudo''
>
> Caroline Ting  (2022).
> Oposiçãocontraditória e Oposição Complementar na Cultura Chinesa.
>
>
> https://hojemacau.com.mo/2022/10/17/oposicao-contraditoria-e-oposicao-complementar-na-cultura-chinesa/
>
>
> <https://hojemacau.com.mo/2022/10/17/oposicao-contraditoria-e-oposicao-complementar-na-cultura-chinesa/>
> Oposição contraditória e oposição complementar na cultura chinesa
> <https://hojemacau.com.mo/2022/10/17/oposicao-contraditoria-e-oposicao-complementar-na-cultura-chinesa/>
> OPOSIÇÃO CONTRADITÓRIA: A palavra chinesa para expressar “contradição” 矛盾
> (Máodùn), é formada por uma lança矛 (máo) oposta a um escudo盾 (dùn). A
> etimologia vem de um extrato de um conto encontrado p…
> hojemacau.com.mo
>
>
> O artigo eh muito bonito, e vale a pena ser lido!
> O veiculo onde ele foi publicado eh um jornal em lingua Portugues de
> Macau!
> Deleitem-se.
>
> Tudo de bom,   ---Julio Stern
>
>
> --
> *From:* Ricardo Pereira 
> *Sent:* Tuesday, December 6, 2022 12:33 PM
> *To:* Julio Stern 
> *Cc:* LOGICA-L ; Marcelo Finger <
> mfin...@ime.usp.br>; eduardoochs ;
> dura...@gmail.com 
> *Subject:* Re: [Logica-l] Re: Meio off-topic: Construção do Reino de Deus
> na UFRN
>
> Oi, Júlio!
> Não sei se vc está usando uma referência desconhecida pra mim (o lance da
> espada e escudo).
> Acho que já devo ter falado isso em outras conversas, mas me parece que a
> problemática é um emaranhado de ideias bem difícil de desatar.
> Por exemplo, é como se a qualquer momento as bases comuns que a gente
> supõe numa conversa, que nos dão a possibilidade de entendimento mútuo,
> pudessem ser revogadas dependendo do nível de estresse (por causa de temas
> polêmicos) das outras pessoas envolvidas. Isso torna a coisa toda bastante
> contraproducente. Algo como esse relato aqui (que não é meu, mas reflete
> bem o fenômeno):
>
> "Uma vez tive uma estudante de filosofia que se contradisse. Falei: ‘Olha,
> você se contradisse: você disse que A era X e depois que não era X’. Ela
> disse ‘e daí? Por que eu não deveria me contradizer?’. Eu disse: ‘Bem, você
> pode, é claro, mas depois perceberá que não terá dito nada: assim, se você
> me disser coisas como “eu li o livro, mas não olhei para ele”, você não
> estará exprimindo significado algum  e não poderemos conversar’. Ela disse:
> ‘Bem, eu não vejo sentido algum em conversar, mesmo’. Eu, então, mudei de
> assunto. Agora, o que está sendo desafiado aqui é algo que todos
> normalmente temos como certo: a lei da não-contradição."
> ---O trecho completo está aqui: https://tinyurl.com/58mzzm8x
>
> On Mon, 28 Nov 2022 at 13:13, Julio Stern  wrote:
>
> Caros Redistas:
> Acho que este topico Nao eh nada off-topic.
> O antidoto para o relativismo total / scepticismo radical
> e outras mazelas epistemologicas que nos afligem
> pode ser encontrado na combinacao de 2 elementos:
> (1) Escudo:  Criterios de demarcacao para Ciencia;
> (2) Lanca:  Testes de hipotese para confirmacao de Leis Naturais.
> Com Escudo e Lanca,  matar os dragoes fica facil.
> Sem as armas antes assinaladas, fica impossivel.
> Tudo de bom,   ---Julio Stern
>
>
> --
> *From:* logica-l@dimap.ufrn.br  on behalf of
> rhalah 
> *Sent:* Monday, November 28, 2022 2:05 PM
> *To:* LOGICA-L 
> *Cc:* Marcelo Finger ; LOGICA-L <
> logica-l@dimap.ufrn.br>; eduardoochs ;
> dura...@gmail.com 
> *Subject:* Re: [Logica-l] Re: Meio off-topic: Construção do Reino de Deus
> na UFRN
>
> Olá, Marcelo e demais colegas listeiros!
>
> Aproveito essa questão do Marcelo, que creio resumir uma parte importante
> e interessante do problema, pra dar uma de re-animator em não uma, mas duas
> conversas antigas da lista (uma é esta).
>
> O tema que me interessa, especificamente, é a questão do relativismo
> epistemológico e como podemos ter ao menos um norte que permita algum senso
> de equilíbrio, respeito e justiça no tratamento das pessoas (culturas
> inteiras também devem ser respeitadas?).
>
> Tivemos aqui na escola em que trabalho (um campus do IFRN) uma
> apresentação do professor Jafelice (professor de física aposentado da UFRN)
> que me deixou bastante incomodado. O problema não foi tanto ele ter falado
> o que falou, mas a ausência de espanto entre os professores presentes,
> principalmente os das ciências naturais. Isso reforça a minha impressão de
> que há realmente dois mundos (as tais duas culturas) que se comunicam muito
> pouco, mesmo quando um dos mundos questiona os fundamentos do trabalh

Re: [Logica-l] Re: Meio off-topic: Construção do Reino de Deus na UFRN

2022-12-06 Por tôpico Ricardo Pereira
Oi Cassiano!

Seguem alguns comentários.

O que eu quero dizer com esses exemplos? Bem, eu penso q o mais importante
> é opor a ciência ao que não é ciência, e não à falsa ciência, ou à ciência
> mentirosa (quem ousar explicar "pseudo", por favor, não sei como fazê-lo,
> acho q penso muito etimologicamente para isso).
>

Creio que as distinções se justificam sempre que refletem fenômenos que
julgamos relevantes também de maneiras distintas.
Nesse caso, muito do que é não-ciência não será confundido com ela (nem tem
essa pretensão) e, consequentemente, não haverá toda uma gama de problemas
associados a essa confusão.
Já o caso de atividades que se parecem muito com a ciência, em todas as
suas características superficiais, mas não têm poder de previsão, por
exemplo, merecem ser tratadas em separado justamente porque muitas pessoas
serão enganadas (nem precisamos entrar no mérito de se há má fé ou não).

Estou me lembrando aqui de um caso recente que aconteceu comigo, em que uma
suposta cobra coral apareceu em minha casa (tivemos de chamar os bombeiros,
já que queríamos matá-la). Pelo que vejo da cultura local, há corais,
não-corais, falsas corais e até falsas falsas corais. Consigo dar conta de
algumas não-corais e falsas falsas corais, mas as corais e falsas corais
(difíceis de distinguir das verdadeiras) são muito perigosas pra mim. Nosso
corpo de bombeiros recomenda não tentar fazer a distinção dessa última
dupla, mas creio que as razões são contrárias às do nosso contexto (evitar
dano causado por uma das cobras vs. conseguir benefício fornecido por uma
das ciências).


> A estratégia de definir ciência pelo recurso às condições sine qua non
> etc. mostra-se muito limitante quando se trata no fundo de uma atitude: a
> genuína atitude científica nada tem q ver com condições suficientes ou
> necessárias, mas com uma genuína disposição para aprender.
>

A genuína disposição para aprender não seria uma condição necessária e
suficiente, então?

E se ela, juntamente com o conhecimento acumulado, acabasse implicando que
devemos proceder de tais e tais maneiras, usando tais e tais métodos e
ferramentas, não passaria a ser obrigação de quem tem genuína disposição de
aprender, diante desse histórico, adaptar suas práticas?
Veja que segundo o que falei acima é possível pensar que os antigos faziam
ciência dadas as suas limitações, mas que a ciência evoluiu a ponto de uma
pessoa que tiver conhecimento dessa evolução não estar sendo fiel à sua
disposição de aprender quando não leva esses desenvolvimentos em conta.

Atualmente, há pessoas com altíssima educação científica ocupando altos
> cargos governamentais e defendendo posições bem duvidosas.
>

Acho que isso é fruto da divisão extrema do trabalho. Parece que a gente
acaba relaxando em relação aos fundamentos racionais e filosóficos da
ciência justamente porque no dia a dia geralmente podemos passar sem eles.
Talvez rapidamente essa maneira de trabalhar se desfaça em tempo de
desconfiança como o nosso, em que esses fundamentos precisam ser
reafirmados e defendidos, mas não parece ter sobrado gente o suficiente
tomando conta deles.


> Então, a questão é para que se usa o conhecimento, não é o conhecimento em
> si. Não existe ciência, nem conhecimento algum, pairando no ar. Se a
> abstração e o distanciamento de contextos locais é o q permite, em certo
> grau, a universalização do conhecimento, ao mesmo tempo sem aproximar de
> contextos locais é impossível aprender (o interesse específico do químico
> na tabela periódica, citado por Peirce no texto acima, é um exemplo disso).
>

Não sei se entendi. Não é contraditório afirmar que o distanciamento de
contextos locais permite universalizar o conhecimento (entendo que está
sendo aprendido um conhecimento universal) e ao mesmo tempo dizer que sem
aproximação dos contextos locais é impossível aprender?


> Nesse sentido, eu diria q a tentativa de impor um modo de conhecer e uma
> maneira de exprimir o conhecimento como a única verdade racional é um
> equívoco colonialista.
>

Acho que a palavra 'impor' aqui é fundamental. Se eu acho que alguém está
com uma crença falsa a respeito de algo, argumentar e tentar convencer essa
pessoa de que ela está errada é desrespeitoso em si? Se em vez de uma
pessoa falarmos de uma coletividade a coisa muda? Como exatamente é isso?


> De fato, uma luta dos povos indígenas é para q os seus modos de saber
> sejam reconhecidos como legitimamente científicos, já q localmente
> funcionam.
>

Gostaria de entender e pensar mais sobre esse ponto. Alguma coisa me diz
que nele se concentra uma proporção maior do problema do que o tamanho do
texto daria a entender.
Se julgamos que a ciência mainstream está num patamar mais elevado que
determinado sistema de crenças S de nossa própria sociedade, por que
esperar que essa relação também não se aplique para o caso de um sistema de
crenças S' de uma outra sociedade que parece ser análogo a S? Por que
meramente pensar nessa hipótese é ser 

Re: [Logica-l] Re: Meio off-topic: Construção do Reino de Deus na UFRN

2022-12-06 Por tôpico Ricardo Pereira
Oi, Júlio!
Não sei se vc está usando uma referência desconhecida pra mim (o lance da
espada e escudo).
Acho que já devo ter falado isso em outras conversas, mas me parece que a
problemática é um emaranhado de ideias bem difícil de desatar.
Por exemplo, é como se a qualquer momento as bases comuns que a gente supõe
numa conversa, que nos dão a possibilidade de entendimento mútuo, pudessem
ser revogadas dependendo do nível de estresse (por causa de temas
polêmicos) das outras pessoas envolvidas. Isso torna a coisa toda bastante
contraproducente. Algo como esse relato aqui (que não é meu, mas reflete
bem o fenômeno):

"Uma vez tive uma estudante de filosofia que se contradisse. Falei: ‘Olha,
você se contradisse: você disse que A era X e depois que não era X’. Ela
disse ‘e daí? Por que eu não deveria me contradizer?’. Eu disse: ‘Bem, você
pode, é claro, mas depois perceberá que não terá dito nada: assim, se você
me disser coisas como “eu li o livro, mas não olhei para ele”, você não
estará exprimindo significado algum  e não poderemos conversar’. Ela disse:
‘Bem, eu não vejo sentido algum em conversar, mesmo’. Eu, então, mudei de
assunto. Agora, o que está sendo desafiado aqui é algo que todos
normalmente temos como certo: a lei da não-contradição."
---O trecho completo está aqui: https://tinyurl.com/58mzzm8x

On Mon, 28 Nov 2022 at 13:13, Julio Stern  wrote:

> Caros Redistas:
> Acho que este topico Nao eh nada off-topic.
> O antidoto para o relativismo total / scepticismo radical
> e outras mazelas epistemologicas que nos afligem
> pode ser encontrado na combinacao de 2 elementos:
> (1) Escudo:  Criterios de demarcacao para Ciencia;
> (2) Lanca:  Testes de hipotese para confirmacao de Leis Naturais.
> Com Escudo e Lanca,  matar os dragoes fica facil.
> Sem as armas antes assinaladas, fica impossivel.
> Tudo de bom,   ---Julio Stern
>
>
> --
> *From:* logica-l@dimap.ufrn.br  on behalf of
> rhalah 
> *Sent:* Monday, November 28, 2022 2:05 PM
> *To:* LOGICA-L 
> *Cc:* Marcelo Finger ; LOGICA-L <
> logica-l@dimap.ufrn.br>; eduardoochs ;
> dura...@gmail.com 
> *Subject:* Re: [Logica-l] Re: Meio off-topic: Construção do Reino de Deus
> na UFRN
>
> Olá, Marcelo e demais colegas listeiros!
>
> Aproveito essa questão do Marcelo, que creio resumir uma parte importante
> e interessante do problema, pra dar uma de re-animator em não uma, mas duas
> conversas antigas da lista (uma é esta).
>
> O tema que me interessa, especificamente, é a questão do relativismo
> epistemológico e como podemos ter ao menos um norte que permita algum senso
> de equilíbrio, respeito e justiça no tratamento das pessoas (culturas
> inteiras também devem ser respeitadas?).
>
> Tivemos aqui na escola em que trabalho (um campus do IFRN) uma
> apresentação do professor Jafelice (professor de física aposentado da UFRN)
> que me deixou bastante incomodado. O problema não foi tanto ele ter falado
> o que falou, mas a ausência de espanto entre os professores presentes,
> principalmente os das ciências naturais. Isso reforça a minha impressão de
> que há realmente dois mundos (as tais duas culturas) que se comunicam muito
> pouco, mesmo quando um dos mundos questiona os fundamentos do trabalho do
> outro.
>
> Alguns trechos que me surpreenderam:
>
> "A ciência é colonizadora: impõe uma visão única de mundo e é fonte de
> poder do pensamento único dominante; isto a torna epistemicida e etnocida
> em inúmeras ocasiões."
>
> "Quando consideramos evidente que o conhecimento científico é superior aos
> outros sistemas de conhecimento, autóctones, nos rendemos à  visão de mundo
> colonizatória."
>
> "A vertente da antropologia que adoto é a culturalista e hermenêutica
> (e.g., GEERTZ, 1997) e a concepção epistemológica, relativista – a qual,
> por sua vez, se articula melhor com um enfoque construtivista em educação,
> em especial aquele interpretado desde a perspectiva da biologia do
> conhecimento (MATURANA, 2001). Adotar essa concepção significa acatar uma
> pluralidade de alternativas para o ser humano construir conhecimento, todas
> igualmente válidas e legítimas. O conhecimento científico, portanto, não é
> visto como especial ou superior, nem implica em alternativa privilegiada
> (em nenhum sentido, seja social, ontológico, filosófico etc.); acata-se a
> existência de uma diversidade epistemológica – no fundo, base das demais
> diversidades."
>
> Os demais trechos que achei problemáticos estão nesse docs (aceita
> comentários):
>
> https://docs.google.com/document/d/1g3H-Tn2pBX3jQG9feb8roDgUd9rp380t8ZdKg77oAu4/edit?usp=sharing
>
> Em relação à problemática exemplificada pelo texto mencionado acima, tenho
> compartilhado com algumas pessoas o que acho ser o melhor resumo de como
> chegamos até aqui:
>
> https://docs.google.com/document/d/1KNTjo16NoQAn-b5UiUVmZ86QWSjBCZkxEX2drSFHrww/edit?usp=sharing
>
> O texto é de 1997. Parece ter sido uma excelente previsão.
>
> Por fim, a outra conversa, que acabei achando ao pesquisar 

Re: [Logica-l] Re: Meio off-topic: Construção do Reino de Deus na UFRN

2022-12-06 Por tôpico Ricardo Pereira
Oi, Eduardo!
Obrigado pelas observações. Seguem meus comentários.


> 1. Ele ataca "a Ciência" sem delimitar qual "Ciência" ele está
>atacando. Pelo que eu entendi a "Ciência" que ele ataca não inclui
>a Antropologia, e também não inclui nenhum matemático ou físico que
>tenha visto as limitações de um certo modo tradicional de dar aulas
>e de escrever e esteja tentando usar outros modos.
>
> É mais ou menos isso. O alvo principal das críticas é a tradição
científica que alguns chamam de ciência ocidental. Talvez um dos desafios
seja justamente porque há inúmeras críticas válidas, mas também parece
haver um impulso muito grande no sentido de passar dessas críticas a um
relativismo total.
Nesse sentido, ao menos algumas vertentes da antropologia parecem ter se
percebido imperialistas e colonizadoras, passando a trabalhar de forma a
eliminar esses pecados de si mesmas.



> 2. Se todas as posições são igualmente válidas, legítimas e
>merecedoras de respeito, então: oba! Eu posso roubar, matar e
>estuprar e a minha posição vai continuar tão válida, legítima e
>merecedora de respeito quanto antes... né?
>
> Pois é.
Já há um tempo que me parece haver uma confusão entre o problema da verdade
e o problema da certeza, onde para se combater a certeza (realmente muito
perigosa), acabou se batendo muito na própria ideia de verdade, que nos dá
ao menos a responsabilidade coletiva de nos esforçarmos pra avançar* em
questões de ética e política.

* O problema é que o avanço do que parece ser um movimento bem intencionado
tem desgastado não só a ideia de verdade, mas a de vários outros conceitos
fundamentais, como imparcialidade, neutralidade, progresso (avanço),
justiça, meritocracia, eficiência (um conceito neoliberal, segundo fui
informado por um colega de trabalho!!) etc.


> 3. Tem gente que diz que não existem músicas melhor ou piores, é só
>uma questão de gosto. Essas pessoas ignoram que quando músicos se
>encontram e tentam gravar uma música juntos eles têm uma noção
>comum do que é uma música boa, e eles usam isso pra tentar fazer a
>música deles ficar a melhor possível. Com ciência e com ética é a
>mesma coisa: tem pessoas de fora que têm certeza de que tudo é
>questão de gosto e que tanto faz, mas as pessoas de dentro têm
>noções comuns do que é fazer ciência da melhor forma possível e do
>que é ser o mais ético possível.
>
> Verdade.
Eu mesmo tive (acho que ainda tenho) uma relação não muito harmônica com a
arte. Provavelmente por causa de influências familiares, que viam a arte
(muito mais do que realmente se aplica) como mero jogo de vaidades, status
e manipulação.
Concordo com sua observação sobre os grupos de especialistas, mas penso que
ainda há uma necessidade de aprovação pelos leigos em algum grau, ao menos
se pensarmos realmente em divisão do trabalho.
Nesse sentido é o mesmo que ocorre em relação ao trabalho de um mecânico,
que tem um entendimento inalcançável por mim, mas cujo
resultado sejá julgado por critérios que estão ao meu alcance (o carro está
andando suavemente, o freio funciona, o consumo está bom etc.).
A diferença seria quantitativa, comparando ao contexto ético e artístico,
em vez de qualitativa, apenas porque em relação a esses últimos ainda
estamos na infância do conhecimento e o problema em si é muito mais difícil
de tratar.


> 4. Me pareceu que essa apresentação dele foi principalmente pra dizer
>pros colegas dele: "olha, vocês são burros, a noção de ciência de
>vocês é um lixo", mas ele fez isso muito mal. Qdz, ele poderia ter
>feito algo muito mais construtivo, tipo mostrar algo sobre uma
>outra forma de aprender física, mas ele só fez umas referências a
>uns textos muito distantes.
>
> Acho que a posição de igualdade com que ele está comprometido (e que creio
ser o grande impulso que o move) o coloca em uma posição insustentável.
Algo que ele insinuou e que me deixou bastante incomodado foi que é
esperada uma reação dos cientistas que estão na zona de conforto, ainda
imersos na visão colonizadora, PORQUE eles não querem perder os privilégios.
A parte até a zona de conforto acho OK, mas essa tendência impressionante
de atribuir às intenções a maior parte da explicação (ou mesmo a
totalidade) me parece alimentar uma animosidade que torna o diálogo quase
impossível. Essa tendência, aliás, tem sido bastante forte em uma série de
contextos onde se discutem injustiças sociais.
Aliás, o Gellner faz uma referência a isso no texto que postei:

"O truque central da “etnometodologia” era a utilização do seguinte
dualismo: nossas declarações são reivindicações à verdade, mas também
feitas por pessoas de carne e osso, em geral com objetivos em mente e em
contextos sociais definidos. Os praticantes desse estilo concentravam-se
nesses últimos aspectos e permitiam que eles vencessem todas as demais
considerações. A prova ou a correção processual, por exemplo, não podiam
ser invocadas uma vez que eram, afinal de 

Re: [Logica-l] Re: Meio off-topic: Construção do Reino de Deus na UFRN

2021-12-22 Por tôpico Ricardo Pereira
Oi, Cassiano e demais.

Seguem meus dois centavos.

On Fri, 10 Dec 2021 at 00:51, Cassiano Terra Rodrigues
 wrote:
>>> Consideração: 0,003% da classe médica nacional não deve ser 
>>> bolsonarista, o que, convenhamos, é um bom sinal dadas as circunstâncias.

As coisas parecem melhores, felizmente. A partir de informações de um
amigo médico, chutaria que os médicos de esquerda estão entre 10 e
20%.

>>> Perdoem o sarcasmo, mas adoro perguntas retóricas: Como chegamos a 
>>> Bolsonaro? #mistérioprofundo

Eu acho muito pertinente essa explicação abaixo:

"Finalmente, pode-se levantar a questão, não mais se o movimento em
direção ao estabelecimento de autoridade racional e clareza contratual
deve ser feito—o que nos parece inquestionável—mas se é de fato
provável que seja feito. Aqui nós alimentamos sérias dúvidas. Em
algumas partes do mundo, onde as condições disciplinares são
extremamente ruins, houve poucas tentativas de restabelecer a
autoridade racional: o caos contínuo parece ser preferido, e os
indivíduos insatisfeitos simplesmente saem (se puderem) do contexto
caótico. A alternativa ao restabelecimento da autoridade racional é um
fenômeno bem conhecido; as coisas acabam ficando tão ruins em todo
lugar (não apenas nas escolas) que uma autoridade poderosa, do tipo
carismático e não-racional, surge com apoio suficiente para assumir o
controle. Incapazes de compreender (quanto mais implementar na
prática) a noção de autoridade racional, as pessoas se apegam faute de
mieux [trad.: por falta de algo melhor] aos representantes de alguma
ideologia específica—geralmente de tipo puritano—que, pelo menos,
“manterá a lei e a ordem”. Esse estado de coisas, por sua vez, está
fadado ao fracasso, pois (por não ser racional) carrega consigo
valores particulares que, mais cedo ou mais tarde, serão atacados. E
assim o mundo dá mais uma volta." (John Wilson, Discipline And Moral
Education, pg. 80~81)

>>> Agora, deixando de lado apenas momentaneamente o exercício de cinismo 
>>> salutar à sanidade mental, é preciso lembrar que o termo pseudociência já 
>>> foi criticado por pressupor um sentido verdadeiro e único de ciência, o q, 
>>> como se sabe, leva a dificuldades de maior monta.

Ainda não tenho um diagnóstico em que possa confiar, mas uma impressão
forte é a de que é muito difícil a gente eliminar a possibilidade de
injustiça a partir de uma distinção que não seja meramente estética.
Então, sempre que há um fim a partir do qual vários meios podem ser
comparados, a gente fica com medo de que o julgamento seja extrapolado
dos meios para os proponentes deles, permitindo que um lado se coloque
como superior e como estando justificado em oprimir ou civilizar o
outro.
Ainda não vi um caso em que me pareceu necessário abrir mão do
julgamento para evitar essa consequência, mas parece que é o que muita
gente está disposta a fazer, muitas vezes instintivamente (dada a
velocidade e ênfase que já vi em algumas reações). Acho essa uma
estratégia muito arriscada, por minar mesmo as compreensões mais
básicas que temos de racionalidade (geralmente a gente busca os
melhores meios pros nossos fins).
Especificamente quanto ao termo pseudociência, acho que podemos
assumir que ele é e será sempre usado indevidamente em vários casos,
mas ao mesmo tempo a distinção básica que creio ser a referência do
termo é muito valiosa pra ser descartada com a água da bacia. Ele não
precisa fazer contraposição a um sentido único e verdadeiro de
ciência, mas apenas passar a ideia de que temos bons motivos pra
acreditar que determinadas práticas (as científicas) são melhores que
outras (e não precisamos abdicar de reavaliar esse status sempre que
quisermos) a partir de critérios com que nós mesmos podemos concordar
(e discutir).

>>> Mas, penso, o ponto mais importante não é esse, mas q os negacionismos 
>>> atuais - o científico, o histórico e todos os outros q conseguirem 
>>> identificar - são um sintoma da modernidade reflexiva, conforme a expressão 
>>> q tomo do professor australiano Andy Blunden, isto é, são produzidos como 
>>> efeitos de uma cultura de massificação do saber formal. Pela primeira vez 
>>> na história, o século XX viu surgir uma massa de gente letrada, altamente 
>>> especializada e intelectualizada, a par e em consequência da massificação 
>>> da alfabetização (formal ou literal).

Não acho que se trate de massificação do saber. Pelo contrário, acho
que é a carência de um saber como estabelecer as crenças de maneira
racional, atentando pro fato de que nosso nível de especialização
social torna necessário confiar em especialistas em um sem número de
contextos que fazem parte das nossas vidas.
Confiança é a palavra-chave. A quebra de confiança nas autoridades (e
instituições) é um fator central. Inclusive tem a ver com a ideia de
autoridade racional do trecho de livro anterior.

Tem também essa outra leitura mais específica que responsabiliza as
pessoas de perfil de esquerda (em que me incluo e, a até onde sei, o
autor 

Re: [Logica-l] Já conhecem este livro?

2021-07-15 Por tôpico Ricardo Pereira
Bacana!
Obrigado pela dica!
Já baixei aqui, graças ao pessoal do Libgen, mas gostaria de
aproveitar pra perguntar aos membros como vêem o equilíbrio entre a
busca pela verdade e as questões morais/tribais, principalmente no
campo da esquerda (onde me incluo).

Tem a ver com o tema do relativismo cultural e com a verdade, mas se
acharem que será off-topic pra além da conta, podem mandar no privado.

Explicando um pouco...

Desde que iniciei meu contato com a filosofia, sinto-me um tanto
dividido a esse respeito, tendo estranhado bastante a percepção de uma
hostilidade nítida não apenas em relação à ciência, mas à própria
racionalidade e até mesmo à verdade (talvez por ter vindo da
engenharia).

Sempre pensei que isso passaria com o tempo e que aos poucos
entenderia por que as coisas eram assim, sendo que meu chute inicial
era que eu estava estranhando porque não levava em conta que as
pessoas estavam discutindo num contexto acadêmico muito restrito,
enquanto eu estava contaminado pelo absolutismo do senso comum.

O problema é que hoje eu vejo que muito do relativismo e das
conclusões extremas em relação à negação da verdade e da razão estão
muito além dos muros da academia e do debate contextualizado, sendo
possível perceber inclusive em posturas de alunos do ensino médio (sou
servidor técnico-administrativo do IFRN).

Só esse ano acabei lendo The Two Cultures (C. P. Snow)¹ e Higher
Superstition (Paul R. Gross and Norman Levitt)², que explicam bastante
coisa e teriam me ajudado MUITO a me situar, se tivesse tido contato
com eles lá pelos idos de 2010, quando entrei no mestrado em
filosofia.

Quem quiser ver um exemplo prático do tipo de atitude que me preocupa,
pode dar uma olhada nesse vídeo (em inglês) que achei esses dias
(menos de 5 minutos, a até onde ela fala em epistemological
standpoints):

https://youtu.be/v5wq71rjLUA?t=206

Pra terminar, acho essas observações de Joseph Heath (em inglês)
também bastante pertinentes no que toca uma abordagem produtiva da
racionalidade e a relação da esquerda acadêmica com a ciência:

https://www.youtube.com/watch?v=8MvvmSlSCLc
(Menos de 10 minutos a partir do ponto marcado, o que inclui a
pergunta do cara da platéia e a resposta a ele).

¹ https://en.wikipedia.org/wiki/The_Two_Cultures
² https://en.wikipedia.org/wiki/Higher_Superstition

On Sat, 10 Jul 2021 at 20:16, Petrucio Viana  wrote:
>
> Boa noite,
> acredito que a maioria já conhece este livro:
>
> https://www.routledge.com/Words-of-Power-A-Feminist-Reading-of-the-History-of-Logic/Nye/p/book/9780367426903
>
> Eu não conheço (só vi a propaganda hoje).
> Acho que pode ser do interesse de quem nunca tinha ouvido falar (como eu)...
>
> abraços
> P
>
> --
> Você recebeu essa mensagem porque está inscrito no grupo "LOGICA-L" dos 
> Grupos do Google.
> Para cancelar inscrição nesse grupo e parar de receber e-mails dele, envie um 
> e-mail para logica-l+unsubscr...@dimap.ufrn.br.
> Para ver essa discussão na Web, acesse 
> https://groups.google.com/a/dimap.ufrn.br/d/msgid/logica-l/CACRvmVQpXzqYjfV8B7pSvQEnH%3DCCkOsPKrAbXk3DMcREB-fNnA%40mail.gmail.com.



-- 

[]'s ...and justice for all.

Ricardo Gentil de Araújo Pereira

-- 
Você está recebendo esta mensagem porque se inscreveu no grupo "LOGICA-L" dos 
Grupos do Google.
Para cancelar inscrição nesse grupo e parar de receber e-mails dele, envie um 
e-mail para logica-l+unsubscr...@dimap.ufrn.br.
Para ver esta discussão na web, acesse 
https://groups.google.com/a/dimap.ufrn.br/d/msgid/logica-l/CALsLM%3DN8efS0Q4nDTD8eAVyPDuX71ZDBSri59FLnsgC4a-Th3g%40mail.gmail.com.


Re: [Logica-l] Lógica Viva, Youtube e argumentação

2021-05-12 Por tôpico Ricardo Pereira
Oi, Walter e demais colegas!

Iniciei anotações de pontos importantes do vídeo, com os respectivos
tempos. A partir deles tentarei montar os argumentos.
Não tenho experiência nesse tipo de tarefa, então a coisa está bem crua
mesmo.
Segue o link do docs pra quem quiser contribuir:
https://docs.google.com/document/d/16tiG9A6WvYAoz-uiV3cd-4oMc_OTIYOXs3VrNrolJvQ/edit?usp=sharing

Mais alguns comentários abaixo:

On Thu, 6 May 2021 at 15:47, Walter Carnielli  wrote:

> Caro Ricardo ,
>
> excelente sugestão -o vídeo é péssimo, mas pelo menos um dos lados tem
> alguma posição com alguma sensatez.
>
> Pois é!
Fiquei bastante incomodado em alguns momentos.
A postura do presidente da AMB foi boa, mas não dá conta do clima atual,
onde já está tudo muito polarizado pra ser possível contar com o princípio
da caridade.
Nessas horas um bom debatedor faz muita diferença.


> Eu já pensei em a gente tomar esse tipo de vídeo e discutir.  ponto por
> ponto, evidenciando as falácias (que são muitas), os erros de interpretação
> do método científico, os pontos onde as partes cometem não-argumentos, ou
> "desargumentos"  e os ataques retóricos.
>
> Acho um exercício muito interessante e com potencial de ser de utilidade
pública.
Talvez os colegas professores que trabalharam alguma disciplina com lógica
informal ou argumentação pudessem fazer uns experimentos legais nesse
sentido.


> Mas não sei se isso é lícito, se é  juridicamente defensael a gente podee
> parar e discutir a cada momento.
>
> Não entendi.
A questão é se infringe alguma licença do vídeo?
Imagino que dá pra fazer com alguns cuidados básicos, já que o vídeo é
público.


> Seria uma coisa muito boa, sem dúvida, até pra fazer em equipe.
>
> Exato!

Volta e meia sinto a necessidade de alguma plataforma para construção
colaborativa de argumentos, que acho ser bem melhor que executar debates
como tradicionalmente se tem feito.
Se pegarmos os debates sobre a existência de deus, por exemplo, parece que
cada debate é um jogo e basta cada espectador torcer pelo "seu campeão",
independentemente de a maioria dos argumentos usados já estar circulando e
já ter contra-argumentos.

Daria um pouco de trabalho para editar , para checar dados etc, mas seria
> muito bom!
>
> Verdade!
Mas aí é que entra a beleza das plataformas colaborativas.

-- 

[]'s ...and justice for all.

Ricardo Gentil de Araújo Pereira

-- 
Você está recebendo esta mensagem porque se inscreveu no grupo "LOGICA-L" dos 
Grupos do Google.
Para cancelar inscrição nesse grupo e parar de receber e-mails dele, envie um 
e-mail para logica-l+unsubscr...@dimap.ufrn.br.
Para ver esta discussão na web, acesse 
https://groups.google.com/a/dimap.ufrn.br/d/msgid/logica-l/CALsLM%3DONJuy%2Bs2aw_34DDAPE88W5uaZg98kJ2EUEQjWs-EsSaQ%40mail.gmail.com.


[Logica-l] Lógica Viva, Youtube e argumentação

2021-05-06 Por tôpico Ricardo Pereira
Pessoal,

Falei com Marcos Silva recentemente sobre a possibilidade de se fazer
alguma análise argumentativa a partir de debates no youtube e conectar ao
projeto Lógica Viva.

O resultado seria, por exemplo, uma postagem de blog, devidamente linkada
no vídeo original e conectando as falácias encontradas aos respectivos
vídeos do canal Lógica Viva.

Seria um bom exercício de lógica informal e, dependendo do vídeo escolhido,
poderia aproveitar temas em evidência, como a polarização relativa ao
tratamento e manejo da COVID-19, pra melhorar a divulgação do projeto.

Resolvi escolher um vídeo (horrendo, mas que tem mais de 200k
visualizações) que retrata bem o nível baixo do debate e a percepção
equivocada (nítida nos comentários) e iniciarei aos poucos as anotações.

Estou bem enferrujado em lógica, mas resolvi postar aqui pro caso de mais
alguém ter interesse em participar.

O vídeo, pra quem tiver estômago, é esse:
https://www.youtube.com/watch?v=Iq0-cX8cUoo

É de cair o queixo: um retrato trágico do nosso tempo.

-- 

[]'s ...and justice for all.

Ricardo Gentil de Araújo Pereira

-- 
Você está recebendo esta mensagem porque se inscreveu no grupo "LOGICA-L" dos 
Grupos do Google.
Para cancelar inscrição nesse grupo e parar de receber e-mails dele, envie um 
e-mail para logica-l+unsubscr...@dimap.ufrn.br.
Para ver esta discussão na web, acesse 
https://groups.google.com/a/dimap.ufrn.br/d/msgid/logica-l/CALsLM%3DM8T3kXzcowpktUMwdCTPfTQBMBEKkw0CQKwmm0kTHypQ%40mail.gmail.com.


Re: [Logica-l] ativismo lógico?

2020-08-04 Por tôpico Ricardo Pereira
Oi, Marcos!

Ótima iniciativa!
Parabéns!

Tenho interesse em argumentação e no processo de crença racional.

Há um tempo, aqui mesmo na lista, foi levantada a problemática da
necessidade de uma preparação emocional sem a qual raramente conseguimos
ficar num estado mental apropriado para uma argumentação sincera e
interessada. Os vários viéses cognitivos, além dos medos e desejos já
enraizados costumam dar uma rasteira já nos nossos primeiros passos
racionais.

Penso que uma estratégia que levasse esse fenômeno em conta desde cedo
(ensino básico) seria uma contribuição excelente no combate a esse clima
absurdo que temos visto.

Gosto muito desses trechos aqui, que têm me lembrado de uma parte
importante desse desafio:

https://docs.google.com/document/d/1ytlOk6N5fsW6WEWz2XNwyRJrW9clXf2WWrB-jvMaqIM/edit?usp=sharing




On Wed, 29 Jul 2020 at 12:27, Marcos Silva  wrote:

> car@s,
>
> tenho certeza que tambem estao chateados com o cenário político
> brasileiro. e com a iminência de termos um segundo mandato (ainda mais
> desastroso) do atual presidente, se açoes conjuntas nao forem implementadas.
>
> eu tenho um projeto de extensao na ufpe para divulgar filosofia e promover
> boa saúde com práticas de yoga para a comunidade academica e dos arredores
> no recife. tudo gratuito.
>
> na pandemia, para continuar ajudando, nos reinventamos e migramos para
> redes sociais, inclusive com aulas online gratuitas.
>
> 
>
> esta semana, fiz um experimento. postei um video denunciando a falácia da
> cloroquina do presidente. é um caso fácil de detectar de um post hoc ergo
> propter hoc.
> https://www.instagram.com/p/CDMMGMZndua/
> https://www.youtube.com/watch?v=uQAUX_LM_1c
>
> 
>
> trata-se de material curto e propositalmente mais acessível. se ajudar uma
> dezena de jovens, estou no lucro.
>
> acho que há um potencial muito grande para popularizacao da
> lógica/filosofia, educaçao em metodologia científica e em política (por que
> nao?), nestas apresentaçoes populares de falácia usando casos cotidianos.
>
> estes videos curtos nao sao difíceis de fazer e podem ajudar muito na
> confusao generalizada em que nos metemos. especialmente na tentativa de
> ocupar lugares publicos, como redes sociais, onde os jovens (desorientados)
> estao.
>
> 
>
> eu sempre tive ótimas experiencias lecionando introducao aa filosofia
> usando o excelente livro (em portugues) dos professores carnielli e
> epistein, "pensamento crítico". O curso foi bem aceito por jovens alunos de
> psicologia, contabilidade, química e filosofia, em fortaleza e maceio, por
> exemplo.
>
> 
>
> uma proposta de ativismo lógico:
>
> talvez a gente pudesse se organizar para pensar e produzir este tipo de
> conteudo acessivel para popularizacao e divulgacao da lógica. é uma
> ferramenta de transformacao e autonomia intelectual muito poderosa.
> (estamos negligenciando _ eu, pelo menos, estava _ o poder das redes
> sociais...)
>
> que acham?
>
> abracos do recife,
> marcos
>
> --
> Marcos Silva (UFPE/CNPq)
> Philosophy Department
> Federal University of Pernambuco, Brazil
> Editor-in-chief Revista Perspectiva Filosófica
> 
> Marcos Silva Philosophy
> 
> "amar e mudar as coisas me interessa mais"
>
> --
> Você recebeu essa mensagem porque está inscrito no grupo "LOGICA-L" dos
> Grupos do Google.
> Para cancelar inscrição nesse grupo e parar de receber e-mails dele, envie
> um e-mail para logica-l+unsubscr...@dimap.ufrn.br.
> Para ver essa discussão na Web, acesse
> https://groups.google.com/a/dimap.ufrn.br/d/msgid/logica-l/CAGZ3pzJaRt983mmvaYSeoCbT2hF616MTMfgKmt40Et6bQoPGOg%40mail.gmail.com
> 
> .
>


-- 

[]'s ...and justice for all.

Ricardo Gentil de Araújo Pereira

-- 
Você está recebendo esta mensagem porque se inscreveu no grupo "LOGICA-L" dos 
Grupos do Google.
Para cancelar inscrição nesse grupo e parar de receber e-mails dele, envie um 
e-mail para logica-l+unsubscr...@dimap.ufrn.br.
Para ver esta discussão na web, acesse 
https://groups.google.com/a/dimap.ufrn.br/d/msgid/logica-l/CALsLM%3DPb_mzJufExtY-_eJJusUNVuP3TJ%3DRh5u5tf3U%3DYHuEdg%40mail.gmail.com.


[Logica-l] Meio-off: Nicolelis falando sobre matemática, IA e humanidade

2020-05-31 Por tôpico Ricardo Pereira
Olá, pessoALL!

Como Nicolelis é uma figura influente, achei que valeria a pena
compartilhar esse vídeo em que ele sai falando um monte de coisa, inclusive
sobre a história da matemática e sobre inteligência artificial, humanidade,
algoritmos etc.

https://www.youtube.com/watch?v=UsNOBZ-ZpK4 

Achei bem complicadas as afirmações que ele faz.

-- 

[]'s ...and justice for all.

Ricardo Gentil de Araújo Pereira

-- 
Você está recebendo esta mensagem porque se inscreveu no grupo "LOGICA-L" dos 
Grupos do Google.
Para cancelar inscrição nesse grupo e parar de receber e-mails dele, envie um 
e-mail para logica-l+unsubscr...@dimap.ufrn.br.
Para ver esta discussão na web, acesse 
https://groups.google.com/a/dimap.ufrn.br/d/msgid/logica-l/CALsLM%3DNWGSdB%2BLcO%2B2f7UG_ZxYw1X6-W%3Dh3eo%2BRezXXvEfjzmQ%40mail.gmail.com.


Re: [Logica-l] [off topic] a filosofia em defesa da educação

2019-05-16 Por tôpico Ricardo Pereira
Bacana.

Lembro-me imediatamente de uma postagem que tinha feito num outro grupo
(que nunca decolou) que, a meu ver, é bem pertinente ao momento atual.
Inclusive João Marcos estava terminando uma resposta, mas, infelizmente
acabou tendo um imprevisto e sendo impossibilitado de concluí-la.

Caso alguém tenha curiosidade:
https://groups.google.com/forum/#!topic/filosofia-ufrn/6Jm5vkCQuzU

On Tue, 14 May 2019 at 17:07, Marcos Silva  wrote:

> alagoas resistindo! exemplo da filosofia da ufal em entrevista.
>
>
> https://www.youtube.com/watch?v=OwqJWkJkwD8=IwAR2fSwzM2dJ_XqMctOM6i8VHMTAw5LkGJNQQ0VxCWi82JNfEPgZc8O59epY
>
> --
> Marcos Silva
> https://sites.google.com/site/marcossilvarj/
> Philosophie macht Spaß!
>
> *"How Colours Matter to Philosophy" (Springer, 2017)*
> http://www.springer.com/us/book/9783319673974
> *"Colours in Wittgenstein's Philosophical Development" (Palgrave, 2017)*
> http://www.palgrave.com/br/book/9783319569185
>
>
>
> --
> Você recebeu essa mensagem porque está inscrito no grupo "LOGICA-L" dos
> Grupos do Google.
> Para cancelar inscrição nesse grupo e parar de receber e-mails dele, envie
> um e-mail para logica-l+unsubscr...@dimap.ufrn.br.
> Para postar nesse grupo, envie um e-mail para logica-l@dimap.ufrn.br.
> Acesse esse grupo em
> https://groups.google.com/a/dimap.ufrn.br/group/logica-l/.
> Para ver essa discussão na Web, acesse
> https://groups.google.com/a/dimap.ufrn.br/d/msgid/logica-l/CAGZ3pz%2BMv5Hr1JtdRq%3DvDKpRiiuqs8U9HErb7AxRTDBHoO-U0w%40mail.gmail.com
> 
> .
>


-- 

[]'s ...and justice for all.

Ricardo Gentil de Araújo Pereira

-- 
Você está recebendo esta mensagem porque se inscreveu no grupo "LOGICA-L" dos 
Grupos do Google.
Para cancelar inscrição nesse grupo e parar de receber e-mails dele, envie um 
e-mail para logica-l+unsubscr...@dimap.ufrn.br.
Para postar neste grupo, envie um e-mail para logica-l@dimap.ufrn.br.
Visite este grupo em https://groups.google.com/a/dimap.ufrn.br/group/logica-l/.
Para ver esta discussão na web, acesse 
https://groups.google.com/a/dimap.ufrn.br/d/msgid/logica-l/CALsLM%3DMrpmOLmROHrRaUeZrLuLy7s6JudFF87ze3EHbh8w%3D-vg%40mail.gmail.com.


Re: [Logica-l] lógica, pra quê?

2017-07-06 Por tôpico Ricardo Pereira
Ultimamente venho me convencendo da importância dos viéses cognitivos e das
paixões (os desejos e ideais com que estamos comprometidos a priori) para
as discussões, principalmente políticas.

Há um livro voltado para filosofia da educação, de John Wilson, com
insights interessantes aplicáveis de maneira geral: Fantasy and Common
Sense in Education.

Uma das coisas que o autor observa frequentemente (não sei se no livro
acima citado) é que as pessoas são "capturadas" por uma espécie de modo de
defesa de princípios ANTES de iniciarem o processo racional que normalmente
conseguem empregar em outros contextos menos carregados emocionalmente.

Talvez tenha a ver com o dizer de Hume sobre a razão ser (e dever ser)
escrava das paixões. Se não às serve, é descartada.

Há um podcast de que gosto muito e que tem uma abordagem mais psicológica:
o You Are Not So Smart. Em vários episódios são abordados viéses cognitivos
cujo conhecimento acho relevante pra qualquer cientista/filósofo.

2017-07-05 12:24 GMT-03:00 Manuel Doria :

> Em meu paper forthcoming sobre filosofia acadêmica e o politicamente
> correto, uma das teses que defendo é que a evidência empírica sugere que
> filósofos profissionais são capazes de facilmente deixar de lado standards
> de racionalidade visando objetividade e migrar para standards de
> racionalidade visando proteção de identidade grupal quando são confrontados
> com hipóteses que vão de encontro com elementos centrais de sua visão de
> mundo.
>
> A situação em alguns casos é bem trágica. Por exemplo, nós esperamos que
> experts se dêem melhor do que não-experts em seu domínio de expertise mas
> existe evidência empírica [1] de que filósofos morais em média são mais
>  suscetíveis a serem contaminados por viéses espúrios do que não-filosófos
> morais em contextos decisórios morais. Fenômenos análogos costumam ocorrer
> pelas ciências sociais e humanidades como economia e história.
>
> Eu recomendo fortemente também o último livro do Sesardic, "When Reason
> Goes On Holiday". Mostra como que algumas das mentes mais afiadas em lógica
> matemática do Século XX são capazes de deixar tudo de lado quando
> raciocinam sobre política.
>
> [ ]'s
>
> [1] Schwitzgebel, E.; Cushman, F. Philosophers’ biased judgments persist
> despite training, expertise and reflection. Cognition 2015, 141, 127–137.
>
> 2017-07-05 11:56 GMT-03:00 Joao Marcos :
>
>> Prezada Gisele (S):
>>
>> Fico contente em vê-la participando das discussões da LOGICA-L, e
>> aproveito para parabenizá-la pela organização do *IV Workshop de
>> Filosofia e Ensino*, semana passada:
>> https://wfeufrgs.wordpress.com/
>>
>> > O texto em questão, embora seu contexto seja estadunidense, merece
>> alguma
>> > atenção no Brasil pelo simples fato de que a filosofia corre,
>> atualmente, o
>> > risco de não mais ser componente curricular obrigatório nas escolas de
>> > Ensino Médio (assim como muitas outras, vale dizer). No nosso contexto,
>> > quando então se intensifica a produção de argumentos em favor da
>> presença da
>> > filosofia na fase média de ensino básico, não é rara uma modalidade de
>> > defesa calcada em pesquisas como as mencionadas no texto, louvando-se as
>> > capacidades de pensamento crítico supostamente desenvolvidas nas aulas
>> de
>> > filosofia. Daí que o texto nos seja relevante, para compararmos com o
>> que se
>> > diz por aqui.
>> >
>> > (Digo "supostamente" porque nunca fica muito claro, nos eventos e
>> textos em
>> > que o tema é discutido, o que se entende por pensamento crítico, nem se
>> > explicitam seus vínculos com a lógica stricto sensu - tampouco se
>> oferecem
>> > exemplos de experiências didáticas de sucesso nesse sentido. Os livros
>> > didáticos selecionados pelo Plano Nacional do Livro Didático (PNLD) até
>> o
>> > último edital possuem capítulos de lógica muito fracos, desconectados
>> dos
>> > demais, e é bem sabido que a maioria dos professores de filosofia do
>> Ensino
>> > Médio não tiveram boas aulas de lógica durante a graduação (o que gera
>> > insegurança para trabalha-la quando se tornam professores, um nefasto
>> > círculo de expectativas antimatemáticas entre postulantes a estudantes e
>> > futuros professores de filosofia) nem tampouco uma preparação para o
>> > trabalho didático com lógica em contextos de escola média e fundamental.
>> > Isso tudo sem contar boa parcela dos envolvidos com ensino de filosofia
>> que
>> > simplesmente desprezam a lógica como instrumento de pensamento e
>> filosofia.)
>>
>> O conteúdo do seu relato é realmente preocupante!
>>
>> Com relação aos pontos de contato entre o conteúdo do texto que
>> circulei antes e a realidade brasileira, choca-me em particular
>> perceber a forma como a educação no Brasil é muitas vezes tratada de
>> uma forma puramente ideologizada --- mesmo por cientistas da área, e
>> pelos especialistas nacionais em teoria pedagógica.  Recordo-me por
>> exemplo de uma reunião da qual participei há 

Re: [Logica-l] identidade, igualdade e equivalência

2013-09-19 Por tôpico Ricardo Pereira
Obrigado, Jean-Yves!

Estou coletando algumas referências e certamente seu artigo será de
grande ajuda.

Grande abraço!

On Thu, Sep 19, 2013 at 12:31 PM, jean-yves beziau beziau...@gmail.com wrote:
 Ola Ricardo
 Recomando meu artigo:
 “What is the principle of identity? (identity, logic and congruence)”
 disponivel no meu website
 http://www.jyb-logic.org/
 Jean-Yves
 ___
 Logica-l mailing list
 Logica-l@dimap.ufrn.br
 http://www.dimap.ufrn.br/cgi-bin/mailman/listinfo/logica-l



-- 

[]'s ...and justice for all.

Ricardo Gentil de Araújo Pereira
___
Logica-l mailing list
Logica-l@dimap.ufrn.br
http://www.dimap.ufrn.br/cgi-bin/mailman/listinfo/logica-l


Re: [Logica-l] a proof, or just an illustration?

2013-08-23 Por tôpico Ricardo Pereira
Oi, Marcelo e ALL!

Sobre provas empíricas em matemática, apesar de estranho, me parece
uma analogia apropriada se considerarmos as provas
estatísticas/probabilísticas com que alguns trabalham atualmente.
Imagino casos onde eu esteja tentando construir uma prova sobre alguma
característica que teriam certos números, mas paralelamente fizesse um
computador sair testando na força bruta em busca de um
contra-exemplo que pouparia meu trabalho (mostrando-o impossível).
Estou falando baseado no que ouço de colegas que sabem mais matemática
que eu, então posso estar sendo infiel em relação ao que eles queriam
me dizer.

Concordo com o Marcelo quanto a ser mais apropriado usar o termo
evidência, mas faço uma observação: dependendo das hipóteses
consideradas, uma evidência empírica pode, ao meu ver, ter força de
prova (como no caso negativo acima—imagino que o Marcelo esteja se
referindo aos casos negativos como probabilidade zero, mas talvez essa
minha observação ajude a alguém).

Ex.:

* O caso dos cisnes negros

- Hipóteses iniciais

H1: há somente cisnes brancos;
H2: há somente cisnes pretos;
H3: há cisnes de ambas as cores.

A primeira observação de um cisne negro eliminaria definitivamente H1,
embora o acúmulo de cisnes pretos consecutivos seja capaz apenas de se
aproximar da certeza em relação a H2. Mesmo assim, após zilhões de
observações de cisnes pretos durante zilhões de anos em que
estivéssemos racionalmente justificados em agir considerando H2 como
verdadeira, a observação de um cisne branco eliminaria (com força de
prova) H2 e estabeleceria definitivamente H3.

Ou seja: a evidência afeta de maneira diferente qualitativa e
quantitativamente as hipóteses dependendo do que elas negam ou
afirmam.

2013/8/23 Marcelo Finger mfin...@ime.usp.br:
 Olá.

 Medidas empíricas são evidências, não provas.

 Evidência aqui é entendido como um operador que transforma
 probabilidades a priori (a prioris) em probabilidades a posteriori (a
 posterioris).  Em tese, só um número infinito de evidências seria
 capaz de gerar algo com probabilidade 1, ou seja, certeza.  Em
 ciências, existe o conceito de aceitável para a comunidade
 científca, resultante da acumulação de evidências, que é quando a
 probabilidade a posteriori se torna maior que um limite arbitrário.  E
 isso não é prova, é aceitação.

 []s

 Marcelo


 2013/8/23 Jccac jcacusto...@gmail.com:


 Soa estranho uma prova empírica para matemática mas na sua opinião a 
 possibilidade de tais estaria a priori eliminadas? Isso seria pq não importa 
 o quanto a medida seja exaustiva, nunca seria possível medir empiricamente a 
 precisão necessária para uma afirmação como Pitagoras, ou pq simplesmente 
 cada medida - ainda que tenha a precisão necessária - é meramente particular 
 e não atingiria assim nunca a generalidade dos teoremas? Ou outra coisa?

 Abs
 Júlio Custodio
 ___
 Logica-l mailing list
 Logica-l@dimap.ufrn.br
 http://www.dimap.ufrn.br/cgi-bin/mailman/listinfo/logica-l




 --
  Marcelo Finger
  Departament of Computer Science, IME
  University of Sao Paulo
  http://www.ime.usp.br/~mfinger
 ___
 Logica-l mailing list
 Logica-l@dimap.ufrn.br
 http://www.dimap.ufrn.br/cgi-bin/mailman/listinfo/logica-l



-- 

[]'s ...and justice for all.

Ricardo Gentil de Araújo Pereira
___
Logica-l mailing list
Logica-l@dimap.ufrn.br
http://www.dimap.ufrn.br/cgi-bin/mailman/listinfo/logica-l


[Logica-l] Conjunto de problemas (exercícios) para facilitar a comparação entre as lógicas.

2012-11-27 Por tôpico Ricardo Pereira
PessoALL,

Como novato no estudo da lógica, estava pensando se seria interessante
a construção de um conjunto de exercícios especificamente para
salientar as diferenças entre os sistemas lógicos. Os exercícios
deveriam ser o mais parecidos possível. Alguém sabe se já há uma
abordagem assim? Bibliografia? Sugestões pessoais? Não dá pra fazer
(ou não compensa)?


--

[]'s ...and justice for all.

Ricardo Gentil de Araújo Pereira
___
Logica-l mailing list
Logica-l@dimap.ufrn.br
http://www.dimap.ufrn.br/cgi-bin/mailman/listinfo/logica-l


[Logica-l] Site para elaborar provas no estilo Fitch

2012-10-24 Por tôpico Ricardo Pereira
http://www.proofmood.com/index_en.php

Testei a opção de exportar em Latex e ficou bonitinho. Pena que é meio
lento pra inserir os passos da prova.

-- 

[]'s ...and justice for all.

Ricardo Gentil de Araújo Pereira
___
Logica-l mailing list
Logica-l@dimap.ufrn.br
http://www.dimap.ufrn.br/cgi-bin/mailman/listinfo/logica-l


[Logica-l] A Computationally-Discovered Simplification of the Ontological Argument

2012-08-28 Por tôpico Ricardo Pereira
Abstract

The authors investigate the ontological argument computationally. The
premises and conclusion of the argument are represented in the syntax
understood by the automated reasoning engine prover9. Using the logic
of definite descriptions, the authors developed a valid representation
of the argument that required three non-logical premises. prover9,
however, discovered a simpler valid argument for God’s existence from
a single non-logical premise. Reducing the argument to one non-logical
premise brings the investigation of the soundness of the argument into
better focus. Also, the simpler representation of the argument brings
out clearly how the ontological argument constitutes an early example
of a ‘diagonal argument’ and, moreover, one used to establish a
positive conclusion rather than a paradox.

http://mally.stanford.edu/Papers/ontological-computational.pdf

-- 

[]'s ...and justice for all.

Ricardo Gentil de Araújo Pereira
___
Logica-l mailing list
Logica-l@dimap.ufrn.br
http://www.dimap.ufrn.br/cgi-bin/mailman/listinfo/logica-l


[Logica-l] Google Scholar (talvez seja útil)

2012-06-16 Por tôpico Ricardo Pereira
Mandei esse e-mail originalmente a um amigo. Como ele gostou, talvez
mais alguém aqui na lista ache útil.


-- Forwarded message --
From: Ricardo Pereira rha...@gmail.com
Date: Fri, Jun 15, 2012 at 11:09 AM
Subject: Google Scholar
To: Allan Medeiros allan...@gmail.com

Há tempos não curiava o Google Scholar e ontem vi dois recursos
novos que achei interessantes:

1 - eles disponibilizam nas preferências uma opção por meio de que é
possível exibir um link que mostra o código bibtex para resultados de
buscas de livros e artigos.

Quando se clica no link (ver anexo), aparece uma janelinha com o texto
correspondente à referência. Esse é o exemplo resultado do link no
anexo:

@book{papoulis1991stochastic,
 title={Stochastic processes},
 author={Papoulis, A. and Probability, R.V.},
 volume={3},
 year={1991},
 publisher={McGraw-Hill, New York}
}

2 - eles têm um link Minhas citações, que supostamente:
Track citations to your publications. Appear in Google Scholar search
results for your name.

--

[]'s ...and justice for all.

Ricardo Gentil de Araújo Pereira


-- 

[]'s ...and justice for all.

Ricardo Gentil de Araújo Pereira
___
Logica-l mailing list
Logica-l@dimap.ufrn.br
http://www.dimap.ufrn.br/cgi-bin/mailman/listinfo/logica-l


Re: [Logica-l] Off-topic: Homeopatia na SciAm Brasil

2012-04-07 Por tôpico Ricardo Pereira
Oi, Tony!

2012/4/6 Tony Marmo marmo.t...@gmail.com:

 Exatamente, na sua humilde opinião. Eu já expliquei que há diversas
 publicações homeopáticas que falam de experimentos feitos por homeopatas e
 resultados de tratamentos em consultórios. A literatura experimental em
 homeopatia é vasta. Quem diz que houve experimentos que não a corroboraram
 são pessoas que NÃO entendendo o que as teorias dos homeopatas dizem
 fizeram testes usando parâmetros alopáticos.

Será?
Acho que o problema com relação a esse ponto específico é a
divergência a respeito da possibilidade do estabelecimento da relação
causa-efeito entre tratamento e cura. Penso que esse procedimento,
feitas as considerações necessárias, é o mesmo para a medicina
homeopática e para a não-homeopática.
Como devo ter falado em mensagem anterior, acho que esse é o ponto
fundamental na discussão.

Vc tem alguma referência onde eu possa ver o erro do estudo
tradicional sobre a eficácia da Homeopatia, bem como a correção e
consequente confirmação da relação causa-efeito?

Se puderes, mande-me alguns links que consideras autoridades a
respeito. Darei um update em minhas fontes.

 1. Há registros de curas passíveis de serem atribuídas a remédios
 homeopáticos?

 Sim. Há 200 anos de curas obtidas e registradas por médicos homeopatas. Há
 também casos de mortes provocadas por medicação homeopática. É possível
 matar uma pessoa sadia dando-lhe remédio homeopático, conforme já foi
 atestado. Os estudos homeopáticos começaram a partir da observação de que
 alguns indivíduos ao tomarem certos remédios desenvolvem os sintomas das
 doenças para as quais os remédios serviriam.

Deixando em standby as considerações sobre a aplicação universal dos
testes probabilísticos, darei uma checada na net.
Caso tenhas alguma referência de destaque que puderes me adiantar,
ficarei grato.

 2. Há alguma inconsistência interna à teoria?

 Depende do que você está pensando por inconsistência interna.

Melhor me manter concentrado na objeção 1. Creio que ela parece ser o
ponto mais grave nas divergências, além de exigir menos conhecimento
para tratamento.


 3.  Ela faz alguma predição testável?

 Sim, todas as hipóteses homeopáticas são testáveis. Algumas funcionam,
 outras não. Por exemplo, se um medicamento funciona com um paciente para
 tratar de sintomas x, y, z, a uma certa diluição, por exemplo, 12CH, os
 efeitos do medicamento podem ser aprofundados a uma diluição mais alta, por
 exemplo, 180 CH. Mas, se a 12 ou a 30 CH o medicamento não fizer efeito, a
 180CH ele poderá ser letal, induzindo sintomas que o paciente não tinha.
 Esse tipo de hipótese seria falsificada se aumentando a diluição de um
 medicamento aparentemente inócuo para o paciente produzisse cura e não
 complicações.

Não entendi bem, mas se é testável, é um ponto positivo.
:-)

-- 

[]'s ...and justice for all.

Ricardo Gentil de Araújo Pereira
___
Logica-l mailing list
Logica-l@dimap.ufrn.br
http://www.dimap.ufrn.br/cgi-bin/mailman/listinfo/logica-l


Re: [Logica-l] Off-topic: Homeopatia na SciAm Brasil

2012-04-07 Por tôpico Ricardo Pereira
Perfeito, Julio!

Para a minha surpresa, parece haver divergência mesmo com relação à
aplicação desses testes à homeopatia. Entendo que eles são universais
e não dependem da teoria considerada.


2012/4/6 Julio Stern jmst...@hotmail.com:

 Caros:

 Nao conheco nada de homeopatia,
 mas entendo de estatistica e ensaios clinicos e pre-clinicos.

 Alguem mencionou que existem ensaios a respeito da eficiencia
 de medicamentos homeopaticos, feitos por agentes neutros,
 em condicoes controladas, com o mesmo rigor que um ensaio,
 digamos, para o FDA.

 Neste caso, a explicacao teorica de Porque o remedio funciona
 isto eh, a metaphysica da homeopatia, nao interessa diretamente para
 validarmos ou rejeitarmos a hipotese de eficiencia do medicamento.
 Mesmo que a explicacao esteja errada, o remedio funciona!

 Alguem pode me dar boas referencias para ensaios deste tipo,
 de preferencia em revistas bem aceitas na comunidade medica?

 Grato,
 ---Julio




 Date: Fri, 6 Apr 2012 16:18:06 -0300
 From: marmo.t...@gmail.com
 To: eduardoo...@gmail.com; logica-l@dimap.ufrn.br
 Subject: Re: [Logica-l] Off-topic: Homeopatia na SciAm Brasil

 Olha, essa acusação fácil de chamar pesquisa de pseudo-ciência por conta de
 uma visão pré-concebida de mundo pode facilmente aplicar-se a qualquer
 projeto de ponta. E aí reclamar do quanto se gastou fica mais fácil ainda.
 Por exemplo, gastam-se milhões para construir um laboratório gigantesco que
 passa a procurar partículas que, pelo que se saiba, só existem
 teoricamente. Este tipo de projeto que merece até colaboração internacional
 poderia ser igualmente rotulado de desperdício. Ainda mais se muitas das
 partículas buscadas não existirem.

 Mas, há investimentos em projetos menos duvidosos que saem mais caro e que
 alguns podem chamar de inúteis: por exemplo, mandar voos tripulados à Lua,
 arriscando a vida de astronautas, quando sondas não-tripuladas poderiam
 fazer o mesmo trabalho.

 O que está acontecendo com os que se queixam contra a homeopatia é que a
 indústria alopática dos EUA resolveu promover uma campanha contra a
 medicina homeopática e com seu poder de mídia conseguiu convencer as
 pessoas de certos mitos anti-homeopáticos. Mas, a mesma campanha já foi
 tentada contra a acunputura, apesar de que acunputuristas conseguem com
 agulhas anestesiar pessoas incrédulas em eventos públicos.

 Enfim, apenas peço que as pessoas, ainda que duvidem de certas teorias,
 pelo menos tenham respeito pelos outros profissionais que estudam anos e
 anos, leem livros e livros, e fazem pesquisa científica e séria no assunto.
 Ideias pré-concebidas ou uma noção falsa de que existe conhecimento
 absoluto não justifica falta de respeito aos demais, principalmente porque
 os demais os respeitam.

 Em 6 de abril de 2012 14:27, Eduardo Ochs eduardoo...@gmail.com escreveu:

  A minha resposta ao protesto do João Marcos é a seguinte: há verbas
  públicas sendo aplicadas em muitas coisas controversas... pra começar
  com um exemplo relativamente inócuo, Arte Contemporânea, que eu aposto
  que muita gente aqui despreza, mas nem todo mundo. Agora um exemplo
  menos inócuo: corrupção - uma parte GIGANTESCA das nossas verbas
  públicas são gastas em coisas que pelo menos 99% das pessoas da lista
  rotulariam tranquilamente como corrupção...
 
  O que a gente pode fazer com relação a estas verbas mal utilizadas? O
  nosso tempo e a nossa energia são limitados, e cada um de nós tem que
  escolher onde e como vamos aplicá-los para combater o mal e daí
  conseguirmos dormir bem de noite - e, claro, as respostas a estes
  onde e como são individuais, e vão depender das preferências e
  afinidades de cada um. É natural que algumas pessoas combatam
  prioritariamente o que consideram como pseudo-ciência - o João Marcos
  já explicou os porquês dele de forma super coerente, mas não tenho os
  links à mão, ele deve ter -, mas há mil outras possibilidades, e,
  quanto a mim, apesar da minha antipatia por alopatia eu gostaria que
  os médicos alopatas nos hospitais públicos fizessem faculdades
  melhores, tivessem condições de trabalho bem melhores e ganhassem bem
  mais do que ganham hoje em dia...
 
  Como eu não consigo parar, lá vai: eu gostaria também que mais
  calouros lá na Ilha das Ostras, onde eu trabalho, entrassem na
  faculdade sabendo ler e escrever direito, e gostaria que o Brasil
  fosse um estado laico - aliás, 1% de alunos ateus é muito pouco - e
  que o nosso próximo presidente fosse alguém melhor do que o Romário. A
  minha estratégia atual pra tentar salvar o mundo um pouquinho inclui
  deixar as pseudo-ciências em paz.
 
  [[]], Eduardo
 
 
  On Fri, Apr 6, 2012 at 1:33 PM, Joao Marcos botoc...@gmail.com wrote:
   Meu protesto vai ser o usual, contra a *malversação das verbas
   públicas*: um dia debatemos aqui o Núcleo de Estudos de Fenômenos
   Paranormais da UnB (aquela mesma universidade onde o ex-reitor comprou
   uma lixeira de ouro), com suas sérias investigações sobre ufologia,
   

Re: [Logica-l] Off-topic: Homeopatia na SciAm Brasil

2012-04-07 Por tôpico Ricardo Pereira
Oi, Marcelo!

2012/4/6 Marcelo Finger mfin...@ime.usp.br:

 O problema da perda/lucro é muito inferior ao fato de que esses
 resultados que levam à aceitação facilitam (ou até forçam) o emprego
 de algum veneno químico que vai ser dado para uma pessoa muito
 debilitada e enferma que, IMHO, estariam mais bem servidas com a
 injestão de compostos muitíssimos mais baratos.

 É o caso do baratíssimo sulfurofane, que já foi relacionado (com as
 mesmas técnicas estatísticas) até com diminuição da incidência de
 metátases em casos de câncer na bexiga.  Mas o sulfurofane não pode
 ser comprado, e é obtido comendo-se Brócoles CRUs (o brócole cozido,
 por incrível que pareça, não o contém).  Não tem a menor chance de ser
 aceito como terapia pelo FDA.  Quem vai investir em uma terapia
 baseada e Brócoles, que não dará o retorno de milhões de dólares.
 Pensando bem, a questão do lucro é realmente fundamental.

 Eu tenho os papers.

Nesse caso vc tem papers que indicam as benesses do brócolis. Supondo
que a metodologia é aceita e de que eles (papers) são confiáveis, é
esse o tipo de evidência pró-homeopatia que procuro.

Dito isso, assim como os interesses podem influenciar para a não
aceitação da homeopatia pelos alopatas (realmente não acho esse
termo apropriado), o inverso me parece tão esperado quanto. Isso sem
sequer distinguir as motivações sentimentais das financeiras. Se
usarmos apenas esse argumento da grande indústria, estaremos
fatalmente embarcando no caminho das teorias conspiratórias.

Pra não deixar passar: a indústria homeopática não me parece pequena
ao ponto de não se encaixar no meu parágrafo anterior. Além disso, o
mesmo argumento (conspiração) pode ser usado para uma gama enorme de
problemas (carro à água x empresas de petróleo; moto contínuo x
empresas de energia, etc).

Ah...
Manda os papers.

-- 

[]'s ...and justice for all.

Ricardo Gentil de Araújo Pereira
___
Logica-l mailing list
Logica-l@dimap.ufrn.br
http://www.dimap.ufrn.br/cgi-bin/mailman/listinfo/logica-l


Re: [Logica-l] Off-topic: Homeopatia na SciAm Brasil

2012-04-07 Por tôpico Ricardo Pereira
Oi, Tony!

2012/4/7 Tony Marmo marmo.t...@gmail.com:
 Eu já te passei uma referência de estudos feitos por um físico, aqui mesmo
 nesta thread.

Se for esse:
http://www.sciencedirect.com/science/article/pii/S1475491607000628
Pensei que não se tratava de evidência à favor da eficiência, mas
sobre uma possível eliminação de objeção à Homeopatia.
Não tive acesso ao artigo todo, mas o abstract reforçou essa idéia.
Espero que meus amigos Tico e Teco ainda estejam funcionando.
:-)

 Há livros e livros escritos por homeopatas há 2 séculos já, artigos
 publicados em revistas periódicas, pesquisas em hospitais britânicos,
 pesquisas feitas em parceria com brasileiros. Se as pessoas não conhecem
 essa literatura, não é problema de quem conhece.

Certamente. Não foi isso que quis dizer. Se dei a entender, me desculpe.

 Mas, quem não a conhece,
 não pode ficar dando chute e dizendo que homeopatia não tem base empírica.
 Conheçam primeiro e depois pensem se podem dizer algo a respeito.

Como falei, sobre a especialização de nossa sociedade, acho inviável
fazer o que já tentei várias vezes (não só sobre homeopatia): procurar
julgar, eu mesmo, a validade das teorias que pensava serem fracas,
procurando no PubMed e em sites pró e contra (pra tentar não ser
vítima de viés). Sei que ao fazer isso estarei inevitavelmente
assumindo um risco de sofrer as consequências das falhas das
autoridades a quem deleguei essa decisão, nesse caso, a
famigeradamente arrogante e conspiratória ciência mainstream.
Juntarei energia pra fazer como o Julio: dar uma fuçada na net, pra
ver se tenho uma imagem diferente da que tive na última vez em que fiz
isso.


 Em 7 de abril de 2012 09:05, Ricardo Pereira rha...@gmail.com escreveu:

 Oi, Tony!

 2012/4/6 Tony Marmo marmo.t...@gmail.com:
 
  Exatamente, na sua humilde opinião. Eu já expliquei que há diversas
  publicações homeopáticas que falam de experimentos feitos por homeopatas
  e
  resultados de tratamentos em consultórios. A literatura experimental em
  homeopatia é vasta. Quem diz que houve experimentos que não a
  corroboraram
  são pessoas que NÃO entendendo o que as teorias dos homeopatas dizem
  fizeram testes usando parâmetros alopáticos.

 Será?
 Acho que o problema com relação a esse ponto específico é a
 divergência a respeito da possibilidade do estabelecimento da relação
 causa-efeito entre tratamento e cura. Penso que esse procedimento,
 feitas as considerações necessárias, é o mesmo para a medicina
 homeopática e para a não-homeopática.
 Como devo ter falado em mensagem anterior, acho que esse é o ponto
 fundamental na discussão.

 Vc tem alguma referência onde eu possa ver o erro do estudo
 tradicional sobre a eficácia da Homeopatia, bem como a correção e
 consequente confirmação da relação causa-efeito?

 Se puderes, mande-me alguns links que consideras autoridades a
 respeito. Darei um update em minhas fontes.

  1. Há registros de curas passíveis de serem atribuídas a remédios
  homeopáticos?
 
  Sim. Há 200 anos de curas obtidas e registradas por médicos homeopatas.
  Há
  também casos de mortes provocadas por medicação homeopática. É possível
  matar uma pessoa sadia dando-lhe remédio homeopático, conforme já foi
  atestado. Os estudos homeopáticos começaram a partir da observação de
  que
  alguns indivíduos ao tomarem certos remédios desenvolvem os sintomas das
  doenças para as quais os remédios serviriam.

 Deixando em standby as considerações sobre a aplicação universal dos
 testes probabilísticos, darei uma checada na net.
 Caso tenhas alguma referência de destaque que puderes me adiantar,
 ficarei grato.

  2. Há alguma inconsistência interna à teoria?
 
  Depende do que você está pensando por inconsistência interna.

 Melhor me manter concentrado na objeção 1. Creio que ela parece ser o
 ponto mais grave nas divergências, além de exigir menos conhecimento
 para tratamento.

 
  3.  Ela faz alguma predição testável?
 
  Sim, todas as hipóteses homeopáticas são testáveis. Algumas funcionam,
  outras não. Por exemplo, se um medicamento funciona com um paciente para
  tratar de sintomas x, y, z, a uma certa diluição, por exemplo, 12CH, os
  efeitos do medicamento podem ser aprofundados a uma diluição mais alta,
  por
  exemplo, 180 CH. Mas, se a 12 ou a 30 CH o medicamento não fizer efeito,
  a
  180CH ele poderá ser letal, induzindo sintomas que o paciente não tinha.
  Esse tipo de hipótese seria falsificada se aumentando a diluição de um
  medicamento aparentemente inócuo para o paciente produzisse cura e não
  complicações.

 Não entendi bem, mas se é testável, é um ponto positivo.
 :-)

 --

 []'s ...and justice for all.

 Ricardo Gentil de Araújo Pereira





-- 

[]'s ...and justice for all.

Ricardo Gentil de Araújo Pereira
___
Logica-l mailing list
Logica-l@dimap.ufrn.br
http://www.dimap.ufrn.br/cgi-bin/mailman/listinfo/logica-l


Re: [Logica-l] Off-topic: Homeopatia na SciAm Brasil

2012-04-06 Por tôpico Ricardo Pereira
Sou do time da oposição à Homeopatia, mas concordo com a primeira
mensagem do Eduardo sobre essa discussão não ser produtiva aqui na
lista, se for acontecer (como de costume) apenas a repetição dos
argumentos já vistos em outras listas mundo a fora.
No entanto, talvez fosse um exercício interessante tentar uma
formulação lógica. Se pudéssemos formalizar o argumento de modo que
pudesse ser aceito por ambas as partes, mesmo que houvesse
divergências a respeito do valor de verdade de algumas premissas (mas
não sobre que premissas seriam consideradas), poderiamos ter um pouco
de diversão produtiva.

Em tempo: adianto que sou novato tanto em lógica quanto em filosofia.
Perdoem-me por alguma barbaridade cometida na sugestão.

2012/4/6 Tony Marmo marmo.t...@gmail.com:
 O problema é que homeopatia não é coisa mística. Ainda que esteja errada
 empiricamente, é uma teoria científica sim. Acreditar somente no que é
 científico não é crer somente em verdades. Nem toda verdade é científica,
 nem tudo que é científico é verdade.

 Em 6 de abril de 2012 02:10, Eduardo Ochs eduardoo...@gmail.com escreveu:

 Outro modelinho simplificado:

  As pessoas tipo C sempre só acreditaram no que é científico,

  As pessoas tipo H sempre acreditaram em coisas místicas,
  homeopatia, etc,

  As pessoas do tipo H-C eram místicas/religiosas quando crianças,
  mas depois viram como aquilo tudo era besteira e agora só acreditam
  em ciência,

  As pessoas tipo C-H só acreditavam no que era científico, mas um
  dia tiveram uma doença grave, se decepcionaram com a ineficácia dos
  médicos, e aí ficaram maravilhadas com a abordagem e a eficácia de
  um médico homeopata que as curou...

  O tipo *C inclui os tipos C e H-C,

  O tipo *H inclui os tipos H e C-H.

 As pessoas *C acham que as pessoas *H não só estão erradas como
 são ingênuas - demonstração: examine cada um dos quatro casos e tape
 os detalhes das historinhas você mesmo, é fácil -, e vice-versa: as
 pessoas *H acham que os *C estão errados e são ingênuos (idem).

 Agora vou enunciar algo que é verdade, mas cuja demonstração não cabe
 na margem do e-mail: os *Cs não vão conseguir convencer os *Hs por
 argumentos científicos, e os *Hs também não vão conseguir convencer
 os *Cs...

 Corolário, continuar nessa discussão sobre homeopatia não vai dar em
 nada, só vai deixar muita gente irritada.

  [[]], Eduardo


 On Fri, Apr 6, 2012 at 1:26 AM, Eduardo Ochs eduardoo...@gmail.com
 wrote:
  Não é um remédio, é um placebo. Considero anti-ético um médico
  prescrever placebos.
 
  Não é um médico, Manuel, é um médico homeopata...
 
  O que vocês acham destas definições aqui (que eu acabei de inventar)?
 
   Um médico só pode tentar curar seus pacientes usando métodos
   estritamente científicos, e nada mais,
 
   Um médico homeopata pode usar todos os métodos que conhecer,
   incluindo remédios que muita gente jura que são placebos, e pode até
   ter conversas com o paciente que façam as atitudes do paciente
   mudarem... Como os pacientes do médico homepata são relativamente
   inteligentes, muito lidos e têm doutorado (em geral em Lógica), eles
   sabem de todas as objeções científicas à homeopatia e estão
   dispostos a tentar um tratamento não garantido, porque eles podem
   desistir e correr pro médico quando desistirem de arriscar.
 
  Desconfio que ética médica e ética de médicos homeopatas são
  coisas diferentes - mas não sei os detalhes, eu só me trato com
  homeopatia, não sou nem médico nem médico homeopata.
 
   [[]],
     Eduardo
 
 
  On Fri, Apr 6, 2012 at 12:38 AM, Manuel Doria manueldo...@gmail.com
 wrote:
  Se hiper-diluições fizessem princípios ativos funcionarem com ainda
 maior
  eficácia, bebendo água da torneira eu estaria me infectando e me
 medicando
  de toda moléstia possível e imaginável.
 
  As analogias com imuno-supressão por via oral no tratamento de alergias
 e
  vacinas são falsas. Homeopatia envolve diluições onde não existe uma
  singela molécula do soluto no solvente (há remédios de 15, 30 e até 200
  diluições!) e são realizados procedimentos ritualísticos
 pré-científicos de
  como as soluções devem ser chacoalhadas exatamente cem vezes, em cada
  diluição. Muito diferente de como patógenos são criados para vacinas,
 tenha
  certeza disso.
 
  No *Phaedrus*, Platão indagava a respeito de universais capazes de
 cortar
  particulares em suas articulações naturais. As ciências naturais
  desvendaram vários destes cortes discretos. Um desses é o Número de
  Avogadro, e ele restringe o campo de atuação da homeopatia. Em um litro
  d'água, se você diluir mais de 11 vezes qualquer solução, só vai
 sobrar...
  água. Não é um remédio, é um placebo. Considero anti-ético um médico
  prescrever placebos.
 
  Abraços.
 
  2012/4/5 Tony Marmo marmo.t...@gmail.com
 
  Respeitosamente discordo. Na verdade, os detratores da homeopatia é
 que,
  sem ofensas da minha parte, revelam ignorar pontos importantes daquilo
 que
  falam.
 
  Não entendi a retratação do 

Re: [Logica-l] Wikiversidade: departamento de lógica

2012-04-06 Por tôpico Ricardo Pereira
Decio e demais colegas listeiros,

Não devo ter me expressado bem.

Não acho que o conteúdo está bom. Apenas me refiro ao potencial que
penso ser excelente.
Se os colegas, como alguns parecem demosntrar, não acreditam no
funcionamento desses sistemas tipo Wikipédia (onde qualquer um pode
editar a quase qualquer tempo*), entendo perfeitamente. Se assim o
for, por questões de coerência, imagino que os colegas também não
gostem da wikipédia como um todo (inclusive em inglês), já que o
sistema é o mesmo.

Mais uma vez: em nenhum momento estava me referindo ao conteúdo
enquanto modelo, mas ao sistema de colaboração e o respectivo objetivo
específico.

*Há mecanismos visando tornar o sistema mais robusto.

2012/4/4 Decio Krause deciokra...@gmail.com:
 Ricardo
 O verbete Introduction to Logiccomeça assim: Logic is the analysis of 
 arguments..
 Hoje em dia não se pode mais aceitar uma definição desse tipo. Basta ver a 
 seção 03-Mathematical Logic and Foundations da MSC, que diz o que é a Lógica 
 (a disciplina) hoje, para ver que teoria da recursão, forcing, modelos 
 booleanos, fundamentos da teoria dos conjuntos, etc. fogem disso. Outra coisa 
 é considerar uma lógica particular, que aí sim pode ser vista como um certo 
 mecanismo de inferências, mas são coisas distintas (para mim ao menos). Mas 
 análise de argumentos? Como levar a sério uma coisa dessas? Seria sim um 
 bom mecanismo de divulgação, mas depende de QUEM escreve. Não pode ser 
 qualquer um.

-- 

[]'s ...and justice for all.

Ricardo Gentil de Araújo Pereira
___
Logica-l mailing list
Logica-l@dimap.ufrn.br
http://www.dimap.ufrn.br/cgi-bin/mailman/listinfo/logica-l


Re: [Logica-l] Off-topic: Homeopatia na SciAm Brasil

2012-04-06 Por tôpico Ricardo Pereira
Oi, Tony!

2012/4/6 Tony Marmo marmo.t...@gmail.com:
 A questão é a seguinte: se você quer discutir uma teoria da medicina ou de
 outra ciência, você deve discutir primeiro o que a teoria diz de fato e NÃO
 o que outros imaginam que ela diga. Por exemplo, os homeopatas NÃO dizem que
 o remédio homeopático tenha a substância. Aliás, eles admitem, como é óbvio,
 que após as diluições não resta mais a substância. Eles NÃO afirmam que seja
 a presença da substância que promova a cura.

Concordo. Espero não ter dado a entender o contrário.

 Mas, eles também NÃO dizem que seja qualquer diluição feita de qualquer
 jeito de qualquer coisa funcione como remédio. Por exemplo, despejar uma
 pílula de aspirina num balde d'água ou numa piscina não faz um remédio
 homeopático. Há uma técnica que eles recomendam para fazer as diluições e
 fora dessa técnica as diluições não funcionariam. Outra coisa: os homeopatas
 NÃO dizem que fazendo diluições indefinidamente sempre se terá um remédio
 homeopático. Existe um limite na escala de diluições a partir do qual não se
 obtém mais remédio.

Entendo. Como falei, concordo com vc no trecho anterior.
:-)

 Eles SIM dizem que remédios feitos a partir destas ultra-diluições
 potencializadas ou energizadas de acordo com a técnica deles podem
 induzir uma pessoa saudável a ter sintomas ou mesmo matá-la, e curar uma
 pessoa doente que tiver os sintomas-alvo. Eles dizem isto não com base na
 física ou na química modernas, mas com base em experiência de tratamento de
 pacientes ou de pessoas que tomaram experimentalmente os remédios
 homeopáticos. Mais: os sintomas não são os únicos parâmetros para prescrever
 os remédios. Na verdade, o médico tem de considerar outras característica
 individuais do paciente.

Na minha humilde opinião, a Homeopatia carece de suporte experimental.
Se a teoria pretende explicar/melhorar os procedimentos de cura, me
parece claro que o ponto de partida seria um grau de certeza
razoável sobre a eficiência esperada da técnica em questão, mesmo
que ainda estejam pendentes as confirmações. Parece-me que os
experimentos que já ocorreram falam contra a Homeopatia, restando,
claro, a possibilidade de realização de novos experimentos.

De maneira mais subjetiva, poderia dizer que vejo a homeopatia como
uma teoria flutuante, onde as bases que seriam o suporte da teoria
há muito deixaram de existir.
Voltando à minha proposta, como maneira de tentar tornar o assunto
mais pertinente à lista, deixo a pergunta: como faríamos para
considerar a homeopatia de maneira lógica?

Me parece que há pelo menos 3 pontos a serem debatidos (e sobre os
quais provavelmente já há bastante material na net):

1 - Há registros de curas passíveis de serem atribuídas a remédios homeopáticos?

2 - Há alguma inconsistência interna à teoria?

3 - Ela faz alguma predição testável?

Isso deixando de lado o meu feeling sobre o status flutuante da
teoria, que além de ser uma opinião confessadamente pessoal minha,
provavelmente iria requerer bastante trabalho para ser debatida de
maneira produtiva.


-- 

[]'s ...and justice for all.

Ricardo Gentil de Araújo Pereira
___
Logica-l mailing list
Logica-l@dimap.ufrn.br
http://www.dimap.ufrn.br/cgi-bin/mailman/listinfo/logica-l


Re: [Logica-l] Off-topic: Homeopatia na SciAm Brasil

2012-04-06 Por tôpico Ricardo Pereira
Ótimas observações, Julio!

2012/4/6 Julio Stern jmst...@hotmail.com:
 A discussao que creio pertinente para esta lista, eh a de como
 - aceitar, validar, certificar -, ou entao
 - rejeitar, invalidar, falsificar -,  a afirmacao de que homeopatia
 (ou qq outra ferramenta) funciona, ou nao.

Pra mim esse é o ponto vital.
Não acho que se tenha como fugir da objetividade mínima necessária
representada pelos testes de hipóteses lógico-probabilísticos (puxando
a sardinha para o lado do livro que tem me cativado recentemente:
Probability Theory: the logic of science), que muitos (não estou
incluindo o Tony) encaram como especificamente pertinentes ao campo da
ciência ortodoxa (no caso específico, a alopática) e, portanto, não
válidos para a Homeopatia. Acho que o mínimo que se pode esperar para
qualquer discussão racional é que haja uma base comum aceita por ambas
as partes e que possa dar suporte à argumentação.

-- 

[]'s ...and justice for all.

Ricardo Gentil de Araújo Pereira
___
Logica-l mailing list
Logica-l@dimap.ufrn.br
http://www.dimap.ufrn.br/cgi-bin/mailman/listinfo/logica-l


Re: [Logica-l] Off-topic: Homeopatia na SciAm Brasil

2012-04-06 Por tôpico Ricardo Pereira
Fala, Tony!

2012/4/6 Tony Marmo marmo.t...@gmail.com:
 Perfeito, mas tem um detalhe: com base nos mesmos critérios que se testam
 remédios alopáticos, não se pode testar hipóteses da homeopatia.

Acho que na pior das hipóteses deve-se ser capaz de atribuir a cura
com uma probabilidade razoável (dadas as informações disponíveis) à
técnica em questão. Não vejo como isso pode ser válido para a
medicina ortodoxa e não válido para a homeopatia.
Acho que esse seria o ponto de partida para uma discussão produtiva.


-- 

[]'s ...and justice for all.

Ricardo Gentil de Araújo Pereira
___
Logica-l mailing list
Logica-l@dimap.ufrn.br
http://www.dimap.ufrn.br/cgi-bin/mailman/listinfo/logica-l


[Logica-l] Wikiversidade: departamento de lógica

2012-04-03 Por tôpico Ricardo Pereira
PessoALL,

Como wikifreak em desenvolvimento, resolvi compartilhar a seguinte página:

http://en.wikiversity.org/wiki/Philosophy

E, mais especificamente:
http://en.wikiversity.org/wiki/Topic:Logic

Parece uma boa forma de produzir material de ensino em conjunto,
incentivando a troca de experiências e se divertindo no processo.
Espero que a coisa se desenvolva.

-- 

[]'s ...and justice for all.

Ricardo Gentil de Araújo Pereira
___
Logica-l mailing list
Logica-l@dimap.ufrn.br
http://www.dimap.ufrn.br/cgi-bin/mailman/listinfo/logica-l


[Logica-l] Teoria de probabilidades como lógica

2012-01-18 Por tôpico Ricardo Pereira
Caros colegas da lista,

Estou há pouco tempo estudando lógica e me interessei bastante na
visão da Teoria de Probabilidades como extensão da lógica clássica,
passível de ser usada em caso de incertezas. Tenho lido os livros do
Carnap (de que gerei o djvu) e, principalmente de E. T. Jaynes
(Probability Theory: the logic of sciente).

Embora ache que o Jaynes seja muito duro com os filósofos¹, estou
gostando bastante do livro e a abordagem dele me parece boa. Gostaria
de saber a opinião dos listeiros sobre o livro ou sobre esse tipo de
abordagem, bem como de receber indicações bibliográficas pertinentes
(inclusive críticas, se for o caso).

¹ Conforme ilustrado por:
Carnap was seeking the general inductive rule (i.e., the rule by
which, given the record of past results, one can make the best
possible prediction of future ones). But he suffered from one of the
standard occupational diseases of philosophers; his exposition wanders
off into abstract symbolic logic without ever considering a specific
real example. So he never rises to the level of seeing that different
inductive rules correspond to different prior information. It seems to
us obvious, from arguments like the above, that this is the primary
fact controlling induction, without which the problem cannot even be
stated, much less solved; there is no ‘general inductive rule’. Yet
neither the term ‘prior information’ nor the concept ever appears in
Carnap’s exposition. 

-- 

[]'s ...and justice for all.

Ricardo Gentil de Araújo Pereira
___
Logica-l mailing list
Logica-l@dimap.ufrn.br
http://www.dimap.ufrn.br/cgi-bin/mailman/listinfo/logica-l


[Logica-l] Lista Acadêmica de Lógica

2011-12-06 Por tôpico Ricardo Pereira Tassinari
Prezados.

Participo desta lista, se não me engano, desde 2006, ou seja, faz, pelo
menos, 5 anos, e certamente muitas informações úteis a mim foram
apresentadas e discutidas aqui, ou seja, ela se constitui em um instrumento
indispensável a quem quer se manter atualizado na área.

Apesar disso, estou prestes a sair dela e gostaria de me manifestar
explicando o porquê.

Penso que algumas pessoas não se dão conta de que esta é uma lista pública
(atualmente com quase 400 membros, além de ser aberta à consulta pela
internet) e se comportam como se estivessem em privativo.

Nada contra, se justamente a lista não fosse pública e se não recebêssemos
todas essas mensagens em nossas caixas de e-mail diariamente (mas, parece
haver pessoas que, mesmo sabendo disso, pouco, ou quase nada, se importam
com os outros).

É horrível, mas cheguei a conclusão de que a existência um moderador é
necessária a listas de discussões como esta, pois, neste caso, as pessoas
viriam a se inteirar das netquetas (por exemplo, aquelas citadas na página
inicial da lista

http://www.dimap.ufrn.br/cgi-bin/mailman/listinfo/logica-l

com o endereço

http://www.icmc.usp.br/manuals/BigDummy/netiqueta.html

mas que também poderia ser

http://pt.wikipedia.org/wiki/Netiqueta)

Em termos da nossa comunidade, não teria nada contra a existência de uma
lista como esta (onde as pessoas podem expressar o que bem entendem e da
forma com que bem entendem), se também houvesse outra lista que contivesse
apenas informações (de eventos, artigos, livros, etc.) ou, simplesmente, na
qual a netqueta ou, o que dá no mesmo, os membros, fossem respeitados.

Abraços,
Ricardo.
--
Dr. Ricardo Pereira Tassinari
Departamento de Filosofia
UNESP - FFC/Marília
www.marilia.unesp.br/ricardotassinari
___
Logica-l mailing list
Logica-l@dimap.ufrn.br
http://www.dimap.ufrn.br/cgi-bin/mailman/listinfo/logica-l


[Logica-l] Stanford: Cursos abertos on-line

2011-11-23 Por tôpico Ricardo Pereira
PessoALL,

Caso alguém se interesse, Stanford está abrindo uma série de cursos
on-line, com material disponível (inclusive exercícios e testes) e
video-aulas, para janeiro do próximo ano.

Esse parece bem interessante: http://www.game-theory-class.org/

Ao final da página há uma lista com outros cursos disponíveis.
-- 

[]'s ...and justice for all.

Ricardo Gentil de Araújo Pereira
___
Logica-l mailing list
Logica-l@dimap.ufrn.br
http://www.dimap.ufrn.br/cgi-bin/mailman/listinfo/logica-l


[Logica-l] Questão de raciocínio lógico/probabilidades - interessante

2011-11-07 Por tôpico Ricardo Pereira
http://flowingdata.com/2011/10/28/best-statistics-question-ever

Sinceramente, ainda não sei a resposta.

-- 

[]'s ...and justice for all.

Ricardo Gentil de Araújo Pereira
___
Logica-l mailing list
Logica-l@dimap.ufrn.br
http://www.dimap.ufrn.br/cgi-bin/mailman/listinfo/logica-l


[Logica-l] Símbolos lógicos unicode

2011-11-01 Por tôpico Ricardo Pereira
Não sei se a maioria aqui já usa, mas a seguinte tabelinha tem me
quebrado um galho para escrever os símbolos básicos em programas
ordinários, como editores de texto simples ou e-mails.

http://en.wikipedia.org/wiki/List_of_logic_symbols

Obs.: não se se as teclas de atalho são as mesmas no Windows ou MacOS
X, mas no linux aqui (Ubuntu) é só digitar control+shift e o número
após o U+ na tabela.

Ex.:
¬ = 00ac
∧ = 2227
∨ = 2228
→ = 2192
↔ = 2194
⊥ = 22a5
∀ = 2200
∃ = 2203

Pode parecer difícil, mas com algum uso acho que dá pra decorar, já
que são poucos símbolos.
Fica a dica.
:-)

-- 

[]'s ...and justice for all.

Ricardo Gentil de Araújo Pereira
___
Logica-l mailing list
Logica-l@dimap.ufrn.br
http://www.dimap.ufrn.br/cgi-bin/mailman/listinfo/logica-l


[Logica-l] particularmente sobre consequencia sintatica

2011-10-20 Por tôpico Ricardo Pereira Tassinari
Olá João, Daniel, Andrea e demais colegas.

Já andei me questionando, devido as aulas de Lógica, sobre o uso do
termo consequencia sintática”.

Cheguei a pensar que poderia se tratar de uma contradição em termos.

Porém o e-mail do Daniel me convenceu.

“Penso na lógica como uma disciplina proposicional, que lida
com proposições, e que as proposições, independentemente do tipo de
entidade que sejam, são passíveis de expressão linguística (ou simbólica)
através de sentenças governadas por regras sintático-gramaticais (ou
de manipulação simbólica).”

O adjetivo “sintática” seria então relativo à sintaxe, na idéia, por
exemplo, do verbete abaixo:

http://pt.wikipedia.org/wiki/Sintaxe

Ou seja, a idéia é que a “manipulação simbólica” é parte da sintaxe,
enquanto não leva em conta a semântica.

Nesse sentido, o mal-estar da (aparente?) contradição em termos é salutar
para dar destaque a intenção de se representar com um sistema formal (parte
sintática de um sistema axiomático) a inferência semântica em estudo.

Abraços,
Ricardo.
--
Dr. Ricardo Pereira Tassinari
Departamento de Filosofia
UNESP - FFC/Marília
www.marilia.unesp.br/ricardotassinari
___
Logica-l mailing list
Logica-l@dimap.ufrn.br
http://www.dimap.ufrn.br/cgi-bin/mailman/listinfo/logica-l


[Logica-l] OFF-TOPIC Teste de site para tradução colaborativa.

2011-09-29 Por tôpico Ricardo Pereira
PessoALL,

Pretendo, traduzir o capitulo 16 do livro Language, Proof and Logic e
resolví testar um site para traduções colaborativas.

O texto em questão, obviamente, é do meu interesse, mas a tradução
também será somada às partes já feitas pelo professor Daniel Durante,
aqui da UFRN, como forma de colaboração. Caso alguém não se sinta à
vontade por estar fazendo um trabalho de interesse alheio, acho que
poderíamos escolher um livro importante de lógica, que fosse do
interesse vários membros da lista,  tentar usar o site com ele.

http://traduwiki.org/Status/MatematicalInduction

Obs.: o site ainda é bem precário, voltado basicamente à tradução do
texto e sequer aceitando formatações básicas. Trata-se de um teste
beeem teste, mesmo.

-- 

[]'s ...and justice for all.

Ricardo Gentil de Araújo Pereira
___
Logica-l mailing list
Logica-l@dimap.ufrn.br
http://www.dimap.ufrn.br/cgi-bin/mailman/listinfo/logica-l


[Logica-l] OFF-TOPIC - Atualização sobre a tradução

2011-09-29 Por tôpico Ricardo Pereira
PessoALL,

Parece que o troço não dá pra usar ainda, a não ser em poucos casos.
Aparentemente ele não reconhece formatação básica necessária, como
quebras de linha, etc. Tentei até usando os códigos
unicode e nada.
Que pena...
:-\

Depois tentarei ver se dá pra resolver. Não ví help nem achei nada a respeito.

-- 

[]'s ...and justice for all.

Ricardo Gentil de Araújo Pereira
___
Logica-l mailing list
Logica-l@dimap.ufrn.br
http://www.dimap.ufrn.br/cgi-bin/mailman/listinfo/logica-l


Re: [Logica-l] Logical Foundations of Probability (Carnap)

2011-09-27 Por tôpico Ricardo Pereira
Atualização:

Só pra informar que terminei a etapa de escaneamento do livro
mencionado. Falta preparar as imagens e criar o pdf. Espero fazê-lo
num futuro próximo.
Caso alguém esteja interessado no livro, posso disponibilizar para
download, assim que terminar o trabalho.

2011/7/28 Ricardo Pereira rha...@gmail.com:
 Olá, pessoal!
 Sou novo na lista e gostaria de saber se alguém tem acesso ao livro do
 Carnap mencionado no subject.

 Tive bastante dificuldade para achá-lo, conseguindo-o de um vendedor
 de material antigo no e-bay.
 Como não achei nenhuma versão on-line, gostaria de saber se alguém se
 interessa em digitalizá-lo.
 Se alguém souber onde conseguir a versão já digitalizada (com ou sem
 OCR), ficaria feliz em saber, claro.

 --

 []'s ...and justice for all.

 Ricardo Gentil de Araújo Pereira




-- 

[]'s ...and justice for all.

Ricardo Gentil de Araújo Pereira
___
Logica-l mailing list
Logica-l@dimap.ufrn.br
http://www.dimap.ufrn.br/cgi-bin/mailman/listinfo/logica-l


[Logica-l] Logical Foundations of Probability (Carnap)

2011-07-28 Por tôpico Ricardo Pereira
Olá, pessoal!
Sou novo na lista e gostaria de saber se alguém tem acesso ao livro do
Carnap mencionado no subject.

Tive bastante dificuldade para achá-lo, conseguindo-o de um vendedor
de material antigo no e-bay.
Como não achei nenhuma versão on-line, gostaria de saber se alguém se
interessa em digitalizá-lo.
Se alguém souber onde conseguir a versão já digitalizada (com ou sem
OCR), ficaria feliz em saber, claro.

-- 

[]'s ...and justice for all.

Ricardo Gentil de Araújo Pereira
___
Logica-l mailing list
Logica-l@dimap.ufrn.br
http://www.dimap.ufrn.br/cgi-bin/mailman/listinfo/logica-l


[Logica-l] Outros termos além de fraco e forte para as correções e completudes

2011-06-27 Por tôpico Ricardo Pereira Tassinari
Prezados.

É corrente usarmos os termos “fraca” e “forte” para nomear as seguintes
propriedades de sistemas formais:
Correção fraca,
Correção forte,
Completude fraca e
Completude forte.

Gostaria de saber se existem outros termos, além de “fraca” e “forte”, para
denotar essas mesmas propriedades.

Abraços,
Ricardo.
--
Dr. Ricardo Pereira Tassinari
Departamento de Filosofia
UNESP - FFC/Marília
www.marilia.unesp.br/ricardotassinari
___
Logica-l mailing list
Logica-l@dimap.ufrn.br
http://www.dimap.ufrn.br/cgi-bin/mailman/listinfo/logica-l


Re: [Logica-l] Nomes de certas regras de inferência

2011-06-20 Por tôpico Ricardo Pereira Tassinari
Olá a todos.

Agradeço ao Walter, ao João, ao Marcelo e a Leonor pelas respostas.

Abraços,
Ricardo.
--
Dr. Ricardo Pereira Tassinari
Departamento de Filosofia
UNESP - FFC/Marília
www.marilia.unesp.br/ricardotassinari



Em 14 de junho de 2011 23:38, Ricardo Pereira Tassinari 
rica...@cle.unicamp.br escreveu:

 Caríssimos.

 Alguém saberia me dizer o nome das seguintes regras de inferência:

 B ⊧ A→B

 ~A ⊧ A→B

 Grato,
 Ricardo.
 --
 Dr. Ricardo Pereira Tassinari
 Departamento de Filosofia
 UNESP - FFC/Marília
 www.marilia.unesp.br/ricardotassinari


___
Logica-l mailing list
Logica-l@dimap.ufrn.br
http://www.dimap.ufrn.br/cgi-bin/mailman/listinfo/logica-l


[Logica-l] Nomes de certas regras de inferência

2011-06-14 Por tôpico Ricardo Pereira Tassinari
Caríssimos.

Alguém saberia me dizer o nome das seguintes regras de inferência:

B ⊧ A→B

~A ⊧ A→B

Grato,
Ricardo.
--
Dr. Ricardo Pereira Tassinari
Departamento de Filosofia
UNESP - FFC/Marília
www.marilia.unesp.br/ricardotassinari
___
Logica-l mailing list
Logica-l@dimap.ufrn.br
http://www.dimap.ufrn.br/cgi-bin/mailman/listinfo/logica-l


[Logica-l] II Colóquio Internacional de Epistemologia e Psicologia Genéticas

2011-05-23 Por tôpico Ricardo Pereira Tassinari
II Colóquio Internacional de Epistemologia e Psicologia Genéticas

Mais informações: http://www.fundepe.com/coloquiopiaget2011/

Abraço,
Ricardo.
--
Dr. Ricardo Pereira Tassinari
Departamento de Filosofia
UNESP - FFC/Marília
www.marilia.unesp.br/ricardotassinari
___
Logica-l mailing list
Logica-l@dimap.ufrn.br
http://www.dimap.ufrn.br/cgi-bin/mailman/listinfo/logica-l


Re: [Logica-l] Proporções matemáticas

2011-01-09 Por tôpico Ricardo Pereira Tassinari
 Hoje:
 
 1. Proporções matemáticas (Maria Francisca)
 2. ACM Ubiquity on What is Computation? (Joao Marcos)
 3. Re: Proporções matemáticas (psdias2)
 
 
  --
 
  Message: 1
  Date: Thu, 6 Jan 2011 14:32:43 -0200
  From: Maria Francisca mfran...@netpar.com.br
  Subject: [Logica-l] Proporções matemáticas
  To: Lista acadêmica brasileira dos profissionais e estudantes da área
  de LOGICA logica-l@dimap.ufrn.br
  Message-ID: 004b01cbadbf$59e3ef20$02010...@mfrancis
  Content-Type: text/plain;charset=iso-8859-1
 
  Olá,
 
  Gostaria de saber se há proporções matemáticas no Universo, se há uma
 razão numérica, ou se tudo é casual e caótico. Estou estudando proporções na
 área de Estética.
 
  Desde já agradeço por algum comentário elucidativo.
 
  Abraços,
 
  Maria Francisca
  Curitiba - PR
 
 
  --
 
  Message: 2
  Date: Thu, 6 Jan 2011 15:44:27 -0200
  From: Joao Marcos botoc...@gmail.com
  Subject: [Logica-l] ACM Ubiquity on What is Computation?
  To: Lista acadêmica brasileira dos profissionais e estudantes da área
  de LOGICA logica-l@dimap.ufrn.br
  Message-ID:
  
  aanlkti==cm7m14ycgn+pvplvhfcgs28u_+9vlz3_o...@mail.gmail.comcm7m14ycgn%2bpvplvhfcgs28u_%2b9vlz3_o...@mail.gmail.com
 
  Content-Type: text/plain; charset=UTF-8
 
  Os artigos estão sendo publicados toda semana.
 
  Ubiquity symposium 'What is computation?'
  http://ubiquity.acm.org/article.cfm?id=1870596
 
  8 das 15 respostas já estão on-line.
 
  JM
 
  --
  http://sequiturquodlibet.googlepages.com/
 
 
  --
 
  Message: 3
  Date: Thu, 06 Jan 2011 22:00:04 -0300
  From: psdias2 psdi...@yahoo.com.br
  Subject: Re: [Logica-l] Proporções matemáticas
  To: logica-l@dimap.ufrn.br
  Message-ID: 4d266594.7060...@yahoo.com.br
  Content-Type: text/plain; charset=ISO-8859-1; format=flowed
 
  Prezada Maria Francisca:
 
  Talvez os livros abaixo sejam úteis:
 
  -  Razão Áurea - A História de fi, um número surpreendente
  Autor: Mário Lívio
  Editora Record
 
  - Matemática - A Ciência Dos Padrões
  Autor: DEVLIN, KEITH
  Editora: PORTO EDITORA
 
  Esses dois autores possuem muitos livros sobre diversos temas de
  Matemática, suas relações
  com a Natureza, Artes e Ciências, etc.
 
  Dê uma olhada, por exemplo, no site da Livraria Cultura
  (www.livcultura.com.br).
 
  Há diversos autores que escrevem livros do tipo divulgação científica
  nessa área.
  Se eu me lembrar de mais algum, envio posteriormente.
 
  Paulo
 
 
 
   Olá,
  
   Gostaria de saber se há proporções matemáticas no Universo, se há uma
 razão numérica, ou se tudo é casual e caótico. Estou estudando proporções na
 área de Estética.
  
   Desde já agradeço por algum comentário elucidativo.
  
 
 
 
  --
 
  ___
  Logica-l mailing list
  Logica-l@dimap.ufrn.br
  http://www.dimap.ufrn.br/cgi-bin/mailman/listinfo/logica-l
 
 
  Fim da Digest Logica-l, volume 59, assunto 2
  
 
 
 
 
  ___
  Logica-l mailing list
  Logica-l@dimap.ufrn.br
  http://www.dimap.ufrn.br/cgi-bin/mailman/listinfo/logica-l

 ___
 Logica-l mailing list
 Logica-l@dimap.ufrn.br
 http://www.dimap.ufrn.br/cgi-bin/mailman/listinfo/logica-l




-- 
Dr. Ricardo Pereira Tassinari - Departamento de Filosofia
UNESP - Faculdade de Filosofia e Ciências - Marília
Homepage: http://www.marilia.unesp.br/ricardotassinari
___
Logica-l mailing list
Logica-l@dimap.ufrn.br
http://www.dimap.ufrn.br/cgi-bin/mailman/listinfo/logica-l


Re: [Logica-l] Fwd: math daily Subj-class mailing 8000 1

2010-05-10 Por tôpico Ricardo Pereira Tassinari
Olá João.

Muitíssimo obrigado pelos links, achei muito interessante!

Vou usar também, em minhas aulas de Lógica, como exemplo de utilização da
lógica de primeira ordem nos fundamentos da ciência.

Abraço,
Ricardo.

Em 7 de maio de 2010 11:29, Joao Marcos botoc...@gmail.com escreveu:

 Dois novos artigos do arXiv.org que podem ser de interesse para quem
 trabalha com a axiomatização de teorias físicas contemporâneas.

 JM


 -- Forwarded message --

 --
  Submissions to:
 Logic
  received from  Wed  5 May 10 20:00:00 GMT  to  Thu  6 May 10 20:00:00 GMT

 --

 %-%-%-%-%-%-%-%-%-%-%-%-%-%-%-%-%-%-%-%-%-%-%-%-%-%-%-%-%-%-%-%-%-%-%-%-%-%-%-

 --
 \\
 arXiv:1005.0960 (*cross-listing*)
 Date: Thu, 6 May 2010 10:07:45 GMT   (28kb)

 Title: A logic road from special relativity to general relativity
 Authors: Hajnal Andr\'eka, Judit X. Madar\'asz, Istv\'an N\'emeti and
 Gergely
  Sz\'ekely
 Categories: gr-qc math-ph math.LO math.MP
 \\
  We present a streamlined axiom system of special relativity in first-order
 logic. From this axiom system we derive an axiom system of general
 relativity
 in two natural steps. We will also see how the axioms of special relativity
 transform into those of general relativity. This way we hope to make
 general
 relativity more accessible for the non-specialist.
 \\ ( http://arxiv.org/abs/1005.0960 ,  28kb)

 --
 \\
 arXiv:1005.0973 (*cross-listing*)
 Date: Thu, 6 May 2010 10:55:35 GMT   (347kb)

 Title: First-Order Logic Investigation of Relativity Theory with an
 Emphasis on
  Accelerated Observers
 Authors: Gergely Sz\'ekely
 Categories: gr-qc math-ph math.LO math.MP
 Comments: PhD thesis E\otv\os Lor\'and University
 \\
  This thesis is mainly about extensions of the first-order logic
 axiomatization of special relativity introduced by Andr\'eka, Madar\'asz
 and
 N\'emeti. These extensions include extension to accelerated observers,
 relativistic dynamics and general relativity; however, its main subject is
 the
 extension to accelerated observers (AccRel). One surprising result is that
 natural extension to accelerated observers is not enough if we want our
 theory
 to imply certain experimental facts, such as the twin paradox. Even if we
 add
 the whole first-order theory of real numbers to this natural extension, it
 is
 still not enough to imply the twin paradox. Nevertheless, that does not
 mean
 that this task cannot be carried out within first-order logic since by
 approximating a second-order logic axiom of real numbers, we introduce a
 first-order axiom schema that solves the problem. Our theory AccRel nicely
 fills the gap between special and general relativity theories, and only one
 natural generalization step is needed to achieve a first-order logic
 axiomatization of general relativity from it. We also show that AccRel is
 strong enough to make predictions about the gravitational effect slowing
 down
 time. Our general aims are to axiomatize relativity theories within pure
 first-order logic using simple, comprehensible and transparent basic
 assumptions (axioms); to prove the surprising predictions (theorems) of
 relativity theories from a few convincing axioms; to eliminate tacit
 assumptions from relativity by replacing them with explicit axioms
 formulated
 in first-order logic (in the spirit of the first-order logic foundation of
 mathematics and Tarski's axiomatization of geometry); and to investigate
 the
 relationship between the axioms and the theorems.
 \\ ( http://arxiv.org/abs/1005.0973 ,  347kb)

 %%--%%--%%--%%--%%--%%--%%--%%--%%--%%--%%--%%--%%--%%--%%--%%--%%--%%--%%--%%

 %%%---%%%---%%%---%%%---%%%---%%%---%%%---%%%---%%%---%%%---%%%---%%%---%%%---
 For general information on the new math archive (partitioned by
 keyword subject classification), see http://arXiv.org/new/math.html
 For subscribe options to combined math archives,
 e-mail To: m...@arxiv.org, Subject: subscribe
 ___
 Logica-l mailing list
 Logica-l@dimap.ufrn.br
 http://www.dimap.ufrn.br/cgi-bin/mailman/listinfo/logica-l




-- 
Dr. Ricardo Pereira Tassinari - Departamento de Filosofia
UNESP - Faculdade de Filosofia e Ciências - Marília
Homepage: http://www.marilia.unesp.br/ricardotassinari
___
Logica-l mailing list
Logica-l@dimap.ufrn.br
http://www.dimap.ufrn.br/cgi-bin/mailman/listinfo/logica-l


Re: [Logica-l] O Estranho Caso do Desaparecimento do Sol

2010-03-16 Por tôpico Ricardo Pereira Tassinari
Oi Daniel.

Como o Doria disse, a interação gravitacional não é instantânea e também tem
a velocidade da luz *c*; assim, se o Sol deixasse de existir (Deus me
livre!), ainda continuaríamos ter a nossa trajetória (da Terra) influenciada
por ele até o instante em que não recebessemos a própria luz que vem dele.

O grande desafio de Einstein, quando criou a Relatividade Restrita, foi
justamente incorporar o campo gravitacional nela, pois a interação entre
dois corpos não poderia ser instantânea; no caso de interação instantânea,
haveria até não conservação da energia (urg!).

Foi por isso que ele criou a Relatividade Geral, para incorporar a
gravitação na nova estrutura de Espaço-Tempo.

Quanto à equivalência entre a Terra girar ou o Universo girar em torno dela,
não é bem assim; basta ver que quando giramos algo surgem forças
centrífugas; assim, temos como diferenciar referenciais em rotação (com
essas forças) e sem rotação (sem essas forças), eles não são equivalentes.

Abraços,
Ricardo.

Colegas,

 Estava eu tranquilamente preparando uma aula de teoria do conhecimento,
 lendo o capítulo 3 do The Problems of Philosophy do Russell, quando a
 seguinte dúvida avassaladora apossou-se de mim. Resolvi compartilha-la
 com vocês na esperança de que algum colega com mais conhecimento de
 Física (mecânica clássica) possa me ajudar a resolvê-la.

 QUESTÃO:
 Se o sol deixasse de existir instantaneamente, de um momento para o
 outro, eu suponho que, devido ao campo gravitacional, a Terra seria
 imediatamente afetada . Ou seja, a força gravitacional é um tipo de ação
 à distância INSTANTÂNEA. A terra, então, imediatamente deixaria sua
 órbita e as forças gravitacionais de interação com os outros planetas e
 demais corpos celestes passariam a ser as únicas a guiar seu movimento
 no espaço. O sol, no entanto, fica a aproximadamente 8 minutos luz da
 terra. Então, por 8 minutos não haveria mais o sol e, no entanto,
 continuaríamos vendo o sol e sendo aquecidos pelo sol. Haveria, então,
 eu suponho, 8 minutos muito estranhos. A imagem do sol continuaria no
 céu, o calor do sol continuaria nos aquecendo, mas a terra estaria solta
 no espaço, pois o sol que ainda vemos e que ainda nos aquece não está
 mais lá. Bem, minha pergunta é a seguinte: há algum fenômeno perceptível
 associado à força gravitacional entre terra e sol? Haveria como
 sentirmos, ou ao menos medirmos (sentir através de instrumentos) que o
 sol não está mais lá, apesar de sua imagem e calor persistirem? Ou não
 há como sentirmos/medirmos a atração gravitacional sol-terra? Não sei se
 fui claro. Vou dar um exemplo. Nós aprendemos e aceitamos, desde
 Copérnico, que a terra gira em torno de seu eixo uma vez por dia e este
 é o movimento que explica os fenômenos perceptíveis e mensuráveis do
 percurso dos astros (sol, lua, planetas, estrelas) no mapa celeste.
 Mas a rotação da terra é apenas a explicação mais simples. Não há ponto
 fixo exterior com relação ao qual podemos determinar quem gira em torno
 de quem. Não há como MEDIR que é a terra que gira em seu eixo e não todo
 o resto que gira em torno da terra. Qualquer medição neste sentido é
 uma interpretação teórica de dados que não são logicamente incompatíveis
 com uma possível interpretação geocêntrica. Dizer quem gira em torno de
 quem é apenas uma questão de escolha teórica, não há fenômenos empíricos
 que nos forcem a aceitar uma teoria e rejeitar a outra. Bem, minha
 questão sobre a força gravitacional terra-sol vai neste sentido. Haveria
 algum efeito empiricamente verificável aqui na terra do sumiço do sol
 nos 8 minutos em que ele não estaria mais lá, apesar de sua imagem e
 calor continuarem nos atingindo? Ou a força gravitacional sol-terra é,
 tal como a rotação da terra, mais teoria do que fato?

 Bem, antes que alguém me ofenda :) , esclareço que meus conhecimentos de
 física não vão além do colegial e umas poucas disciplinas no curso de
 Engenharia de Computação. Então, por favor, corrijam meus eventuais
 erros infantis com paciência :) Também sei que deve ser fisicamente
 impossível simplesmente desintegrar o sol para ver o que acontece aqui
 na Terra, mas isso é uma liberdade filosófica a qual podemos nos
 permitir tendo em vista que seu interesse motivador é a teoria do
 conhecimento e não a física.

 Saudações,
 Daniel
 ___
 Logica-l mailing list
 Logica-l@dimap.ufrn.br
 http://www.dimap.ufrn.br/cgi-bin/mailman/listinfo/logica-l




-- 
Dr. Ricardo Pereira Tassinari - Departamento de Filosofia
UNESP - Faculdade de Filosofia e Ciências - Marília
Homepage: http://www.marilia.unesp.br/ricardotassinari
___
Logica-l mailing list
Logica-l@dimap.ufrn.br
http://www.dimap.ufrn.br/cgi-bin/mailman/listinfo/logica-l


Re: [Logica-l] O Estranho Caso do Desaparecimento do Sol

2010-03-16 Por tôpico Ricardo Pereira Tassinari
Ops! Mil desculpas!

Ricardo.

Em 16 de março de 2010 14:51, Famadoria famado...@gmail.com escreveu:

  Soh um detalhe: sou o outro Doria. Quem respondeu foi Manuel meu filho.

 Sent from my iPhone

 On 16/03/2010, at 14:15, Ricardo Pereira Tassinari rica...@cle.unicamp.br
 wrote:

  Oi Daniel.

 Como o Doria disse, a interação gravitacional não é instantânea e também
 tem a velocidade da luz *c*; assim, se o Sol deixasse de existir (Deus me
 livre!), ainda continuaríamos ter a nossa trajetória (da Terra) influenciada
 por ele até o instante em que não recebessemos a própria luz que vem dele.

 O grande desafio de Einstein, quando criou a Relatividade Restrita, foi
 justamente incorporar o campo gravitacional nela, pois a interação entre
 dois corpos não poderia ser instantânea; no caso de interação instantânea,
 haveria até não conservação da energia (urg!).

 Foi por isso que ele criou a Relatividade Geral, para incorporar a
 gravitação na nova estrutura de Espaço-Tempo.

 Quanto à equivalência entre a Terra girar ou o Universo girar em torno
 dela, não é bem assim; basta ver que quando giramos algo surgem forças
 centrífugas; assim, temos como diferenciar referenciais em rotação (com
 essas forças) e sem rotação (sem essas forças), eles não são equivalentes.

 Abraços,
 Ricardo.

 Colegas,

 Estava eu tranquilamente preparando uma aula de teoria do conhecimento,
 lendo o capítulo 3 do The Problems of Philosophy do Russell, quando a
 seguinte dúvida avassaladora apossou-se de mim. Resolvi compartilha-la
 com vocês na esperança de que algum colega com mais conhecimento de
 Física (mecânica clássica) possa me ajudar a resolvê-la.

 QUESTÃO:
 Se o sol deixasse de existir instantaneamente, de um momento para o
 outro, eu suponho que, devido ao campo gravitacional, a Terra seria
 imediatamente afetada . Ou seja, a força gravitacional é um tipo de ação
 à distância INSTANTÂNEA. A terra, então, imediatamente deixaria sua
 órbita e as forças gravitacionais de interação com os outros planetas e
 demais corpos celestes passariam a ser as únicas a guiar seu movimento
 no espaço. O sol, no entanto, fica a aproximadamente 8 minutos luz da
 terra. Então, por 8 minutos não haveria mais o sol e, no entanto,
 continuaríamos vendo o sol e sendo aquecidos pelo sol. Haveria, então,
 eu suponho, 8 minutos muito estranhos. A imagem do sol continuaria no
 céu, o calor do sol continuaria nos aquecendo, mas a terra estaria solta
 no espaço, pois o sol que ainda vemos e que ainda nos aquece não está
 mais lá. Bem, minha pergunta é a seguinte: há algum fenômeno perceptível
 associado à força gravitacional entre terra e sol? Haveria como
 sentirmos, ou ao menos medirmos (sentir através de instrumentos) que o
 sol não está mais lá, apesar de sua imagem e calor persistirem? Ou não
 há como sentirmos/medirmos a atração gravitacional sol-terra? Não sei se
 fui claro. Vou dar um exemplo. Nós aprendemos e aceitamos, desde
 Copérnico, que a terra gira em torno de seu eixo uma vez por dia e este
 é o movimento que explica os fenômenos perceptíveis e mensuráveis do
 percurso dos astros (sol, lua, planetas, estrelas) no mapa celeste.
 Mas a rotação da terra é apenas a explicação mais simples. Não há ponto
 fixo exterior com relação ao qual podemos determinar quem gira em torno
 de quem. Não há como MEDIR que é a terra que gira em seu eixo e não todo
 o resto que gira em torno da terra. Qualquer medição neste sentido é
 uma interpretação teórica de dados que não são logicamente incompatíveis
 com uma possível interpretação geocêntrica. Dizer quem gira em torno de
 quem é apenas uma questão de escolha teórica, não há fenômenos empíricos
 que nos forcem a aceitar uma teoria e rejeitar a outra. Bem, minha
 questão sobre a força gravitacional terra-sol vai neste sentido. Haveria
 algum efeito empiricamente verificável aqui na terra do sumiço do sol
 nos 8 minutos em que ele não estaria mais lá, apesar de sua imagem e
 calor continuarem nos atingindo? Ou a força gravitacional sol-terra é,
 tal como a rotação da terra, mais teoria do que fato?

 Bem, antes que alguém me ofenda :) , esclareço que meus conhecimentos de
 física não vão além do colegial e umas poucas disciplinas no curso de
 Engenharia de Computação. Então, por favor, corrijam meus eventuais
 erros infantis com paciência :) Também sei que deve ser fisicamente
 impossível simplesmente desintegrar o sol para ver o que acontece aqui
 na Terra, mas isso é uma liberdade filosófica a qual podemos nos
 permitir tendo em vista que seu interesse motivador é a teoria do
 conhecimento e não a física.

 Saudações,
 Daniel
 ___
 Logica-l mailing list
 Logica-l@dimap.ufrn.br
 http://www.dimap.ufrn.br/cgi-bin/mailman/listinfo/logica-l




 --
 Dr. Ricardo Pereira Tassinari - Departamento de Filosofia
 UNESP - Faculdade de Filosofia e Ciências - Marília
 Homepage: http://www.marilia.unesp.br/ricardotassinari

Re: [Logica-l] O Estranho Caso do Desaparecimento do Sol

2010-03-16 Por tôpico Ricardo Pereira Tassinari
 sol para ver o que acontece aqui
  na Terra, mas isso é uma liberdade filosófica a qual podemos nos
  permitir tendo em vista que seu interesse motivador é a teoria do
  conhecimento e não a física.
 
  Saudações,
  Daniel
  ___
  Logica-l mailing list
  Logica-l@dimap.ufrn.br
  http://www.dimap.ufrn.br/cgi-bin/mailman/listinfo/logica-l
 
 
 
 
  --
  Dr. Ricardo Pereira Tassinari - Departamento de Filosofia
  UNESP - Faculdade de Filosofia e Ciências - Marília
  Homepage: http://www.marilia.unesp.br/ricardotassinari
 





-- 
Dr. Ricardo Pereira Tassinari - Departamento de Filosofia
UNESP - Faculdade de Filosofia e Ciências - Marília
Homepage: http://www.marilia.unesp.br/ricardotassinari
___
Logica-l mailing list
Logica-l@dimap.ufrn.br
http://www.dimap.ufrn.br/cgi-bin/mailman/listinfo/logica-l


Re: [Logica-l] Fw: [Bulk] Re: [Bulk] Re: Explicação do significado da implicação cl ássica

2010-01-29 Por tôpico Ricardo Pereira Tassinari
Olá Matheus e a todos.

Penso que sua dissertação será interessante; assim que a defender, mande um
link para nós.

Bem, o que quero dizer com a condicional ser um tipo de implicação é (pelo
menos) o seguinte:

Ela serve como implicação em várias teorias formais.

Para citar dois exemplos, temos os estudos realizados em teorias formais dos
números ou em teorias formais de conjunto (cf., por exemplo, Mendelson, E.,
Introduction to Mathematical Logic, Cap.3, Formal Number Theory, e Cap 4.
Axiomatic Set Theory:
http://books.google.com.br/books?id=ZO1p4QGspoYCdq=Mendelson,+Introduction+to+Mathematical+Logicprintsec=frontcoversource=bnhl=pt-BRei=08piS_XFNZSXtgeW_rWkBgsa=Xoi=book_resultct=resultresnum=4ved=0CB0Q6AEwAw#v=onepageq=f=false
)

Sabemos dos defeitos que ela tem, mas, nestes casos, ela tem um desempenho
satisfatório.

Em especial, o Teorema da Dedução nos permite estabelecer uma relação
precisa entre dedução e a condicional.

Só para citar um caso temos que se A deduz B se, e somente se, A=B é
tautologia.

Assim, nestes casos, USAMOS a condicional como implicação. Não vejo nenhum
problema com isso. É uma forma de implicação usada quando fazemos Matemática
dessa forma (usando estas teorias).

Penso também que podemos considerá-la em vários caso da linguagem natural
(como nos exemplos que usamos em sala de aula) e em muitos casos dos modelos
na Ciência.

Bem, é isso.

Abraços,
Ricardo.


2010/1/28 Matheus mateusm...@yahoo.com.br


 - Original Message -
 *From:* Matheus mateusm...@yahoo.com.br
 *To:* Ricardo Pereira Tassinari rica...@cle.unicamp.br
 *Sent:* Thursday, January 28, 2010 11:04 PM
 *Subject:* Re: [Bulk] Re: [Bulk] Re: [Logica-l]Explicação do significado
 da implicação clássica


 Olá Ricardo

 Acho que não fui claro ao mencionar os dois exemplos: são casos em
 que parecem desmentir (e não confirmar) a suposta inadequação da condicional
 material. Quanto disse desmentir posso ter passado sem querer a
 implicatura errada de que se tratava de mais um contra-exemplo à adequação
 da condicional material.

 Sobre a hipótese tentadora de que há vários tipos de frases condicionais:
 num certo sentido é uma hipótese verdadeira, mas trivial. No sentido
 relevante é uma hipotese bastante controversa. É verdadeiro (e também
 trivial) que há frases condicionais indicativas e subjuntivas, por exemplo.
 É importante, mas controverso, que isso demonstre que há um mecanismo
 semântico para cada tipo de classificação gramatical das frases
 condicionais. Stalnaker, por exemplo, defende uma teoria que fornece uma
 explicação do mecanismo semântico de todas as condicionais,
 incluindo indicativas e subjuntivas, sobre uma mesma base de similaridade
 entre mundos possíveis. Outros téoricos, como Jackson, defendem que a
 condicional material fornece as condições de verdade das condicionais
 indicativas ao passo que um mecanismo diferente explica o funcionamento das
 condicionais subjuntivas.  And so on.

 Eu não sei se concordaria em dizer que a condicional material é um tipo
 de condicional por mérito próprio: a condicional material é um conectivo
 verofuncional com um comportamento preciso, que é estabelecido por uma
 tabela de verdade e que foi criada artificialmente para satisfazer
 determinados fins teóricos. Dizer que a condicional material é também um
 tipo de frase condicional me parece tão estranho quanto dizer que um mapa
 que foi criado para representar uma cidade é também um tipo de cidade.

 Abs
 Matheus



-- 
Dr. Ricardo Pereira Tassinari - Departamento de Filosofia
UNESP - Faculdade de Filosofia e Ciências - Marília
Homepage: http://www.marilia.unesp.br/ricardotassinari
___
Logica-l mailing list
Logica-l@dimap.ufrn.br
http://www.dimap.ufrn.br/cgi-bin/mailman/listinfo/logica-l


Re: [Logica-l] Piaget

2010-01-28 Por tôpico Ricardo Pereira Tassinari
2010/1/27 Julio Fontana juliocesarfont...@yahoo.com.br

 Olá,


Olá.



 Não disse que não li e não gostei. Essa é uma afirmação, ao que parece, do
 Doria.


Também eu não disse que você NÃO LEU e não gostou. Perguntei O QUE NÃO
GOSTOU daquilo que LEU, que foi A Representação do Mundo na Criança.

Minha intenção não é de defender uma religião (como você escreve abaixo),
mas defender a área em que trabalho de uma crítica fácil e sem fundamento.

Quanto à crítica do Dória, ele mesmo reconheceu que não sabe nada de Piaget
(veja o e-mail
http://www.dimap.ufrn.br/pipermail/logica-l/2010-January/004412.html).

Que interessa uma crítica assim?



 Já leu a teoria das inteligências múltiplas de H. Gardner?


Infelizmente não li. Gostaria de saber se as análises dele toca nos
problemas da  Teoria do Conhecimento (da relação entre o sujeito que conhece
e o o objeto conhecido, como em Hume, Kant, etc) e da Epistemologia (da
análise crítica do conhecimento científico, como em Granger, por exemplo)?


 Parece que o senhor está defendendo uma religião.


Novamente: apenas tento defender um mínimo de rigor em uma área de trabalho
que é a Teoria do Conhecimento e a Epistemologia, principalmente em relação
às contribuições de Piaget que, penso, tem defeitos, mas que tem muito a
contribuir.
Para citar um exemplo, os especialistas vivem discutindo sobre o Princípio
da Contradição, mas muito poucos pesquisam o USO do Princípio (como, por
exemplo, em Piaget, Recherches sur la contradiction, Paris, Presses univ. de
France, 1974).


 Não é por que o livro tem o título A Representação do Mundo na Criança
 que nele conterá uma descrição exata disso. O livro pode ser ruim, né?


Esta foi a questão: o livro é ruim? Por quê?

Lembrando que, no livro, Piaget pretende explicar a representação do mundo
pela criança para principalmente trazer elementos para discussão em Teoria
do Conhecimento e em Epistemologia. Acho que nós só vamos caminhar nestas
áreas se entrarmos na discussões específicas dos problemas e soluções que
cada obra trás.




 Julio Fontana
 Bacharel em Filosofia pela PUC-Rio
 Especialização em cultura clássica greco-latina.


Abraço,
Ricardo.


 --- Em *qua, 27/1/10, Ricardo Pereira Tassinari 
 rica...@cle.unicamp.br*escreveu:


 De: Ricardo Pereira Tassinari rica...@cle.unicamp.br
 Assunto: Re: [Logica-l] Piaget
 Para: Julio Fontana juliocesarfont...@yahoo.com.br
 Data: Quarta-feira, 27 de Janeiro de 2010, 18:49


 Olá Julio.

 Não entendi. Você chamou o livro A Representação do Mundo na Criança de
 experimento? Bem, nele há VÁRIOS experimentos...

 Qual o(s) outro(s) livro(s) que você leu e não gostou? Por quê?

 Também não entendi: você acha que o livro A Representação do Mundo na
 Criança não explica a representação do mundo pela criança? (pois é isso que
 o livro pretende explicar).

 Já discutimos um pouco sobre Piaget nesta lista, dê uma olhada, por
 exemplo, em:

 http://www.dimap.ufrn.br/pipermail/logica-l/2009-August/003946.html

 Sugiro também que dê uma olhada na bibliografia:


 http://polo1.marilia.unesp.br/Home/Instituicao/Docentes/RicardoTassinari/BP.htm

 Abraço,
 Ricardo.


 2010/1/26 Julio Fontana 
 juliocesarfont...@yahoo.com.brhttp://br.mc1104.mail.yahoo.com/mc/compose?to=juliocesarfont...@yahoo.com.br
 

Mas quem precisa entender Piaget atualmente?

 O pessoal da Pedagogia diz que ele está superado e que existem técnicas
 melhores de ensino/aprendizagem. A única coisa que tem algum interesse, ao
 meu ver, é o seu experimento com crianças, A Representação do mundo na
 criança. Interesse restrito pois não explica muita coisa também.

 Acho a matemática mais fácil de ser ensinada e aprendida do que qualquer
 outra disciplina de humanas. Um filósofo, por exemplo, devido a complexidade
 do seu campo somente começa a produzir algo original após os 50 anos de
 idade. Idade essa que o matemático, segundo G. H. Hardy, já produziu
 tudo aquilo que a sua criatividade o permitiu produzir.

  Portanto, a matemática é mais fácil de ser aprendida e ensinada do que as
 disciplinas de humanas. Se os senhores acham que Brasil vai mal em
 matemática, nem queiram ver como ele vai em história, geografia, português,
 ...

 Julio Fontana

 --
 Veja quais são os assuntos do momento no Yahoo! + Buscados: Top 
 10http://br.rd.yahoo.com/mail/taglines/mail/*http://br.maisbuscados.yahoo.com/-
 Celebridadeshttp://br.rd.yahoo.com/mail/taglines/mail/*http://br.maisbuscados.yahoo.com/celebridades/-
 Músicahttp://br.rd.yahoo.com/mail/taglines/mail/*http://br.maisbuscados.yahoo.com/m%C3%BAsica/-
 Esporteshttp://br.rd.yahoo.com/mail/taglines/mail/*http://br.maisbuscados.yahoo.com/esportes/
 ___
 Logica-l mailing list
 Logica-l@dimap.ufrn.brhttp://br.mc1104.mail.yahoo.com/mc/compose?to=logic...@dimap.ufrn.br
 http://www.dimap.ufrn.br/cgi-bin/mailman/listinfo/logica-l




 --
 Dr. Ricardo Pereira Tassinari - Departamento de Filosofia
 UNESP - Faculdade de

Re: [Logica-l] Explicação do significado da implic ação clássica

2010-01-28 Por tôpico Ricardo Pereira Tassinari
Olá Adolfo e a todos.

Abaixo uma pequena reflexão que gostaria de compartilhar sobre a implicação
na linguagem natural entre duas sentenças (que designarei por se A então
B) e o conectivo condicional.

Notemos então que o condicional pode ser parafraseado, em linguagem natural
por: não é o caso que A e não B, já que ele é equivalente a ~(A~B).

Quando passamos DA IMPLICAÇÃO DA LINGUAGEM NATURAL PARA O CONECTIVO
CONDICIONAL, NÃO TEMOS PROBLEMA (como no exemplo que citou ou em Se chove,
então a rua está molhada), pois, se A então B, então temos que: não é o
caso que A e não B. Exemplo, se assumimos que se chove, então a rua está
molhada, podemos sempre concluir que não é o caso que chove e a rua não
está molhada.

A VOLTA QUE É PROBLEMÁTICA, OU SEJA, PASSAR DO CONECTIVO CONDICIONAL PARA A
IMPLICAÇÃO NA LINGUAGEM NATURAL, ou ainda, passar de não é o caso que A e
não B para se A então B. Exemplo: todos nós concordamos que não é o caso
que a Lua é de queijo e o Sol é de gelo, mas nos é estranho dizer que se a
Lua é de queijo então o Sol é de gelo.

Ou seja, assumir que existe uma implicação na linguagem natural é
condição suficiente (mas não necessária) para assumirmos uma implicação
condicional.

Assim, se um argumento é válido na linguagem natural (isto é se assumir as
premissas implica em assumir a conclusão), então sua formalização com o
condicional (conjunção das premissas condicional conclusão) será sempre
verdadeira; por isso, se o argumento é válido, então sua formalização tem
que ser, isto é, nunca encontraremos premissas verdadeiras e conclusão
falsa, mas nem sempre uma argumento formalmente válido fica bem em linguagem
natural.

Abraço a todos,
Ricardo.

2010/1/28 Adolfo Neto ado...@utfpr.edu.br

 Gostei do exemplo abaixo...

 Fonte: Introdução à Álgebra Abstrata, de Jaime Evaristo
 http://professor.ic.ufal.br/jaime/livros/Capitulo%25201.pdf
 p.11-12

 O exemplo a seguir mostra que o significado matemático do se então,
 embora inusitado, tem
 sentido também no nosso dia a dia. Imagine que um pai anuncie para sua
 filha que vai fazer o
 vestibular para um curso de Medicina: se você passar, então eu lhe dou um
 carro.

 Se a filha foi aprovada (p verdade) e recebeu o carro (q verdade), o
 pai cumpriu a promessa
 (p = q verdade); se a filha foi aprovada (p verdade) e não recebeu o
 carro (q falso), o pai
 descumpriu a promessa (p = q falso); se a filha não foi aprovada (p
 falso) e não recebeu o carro (q
 falso), o pai não descumpriu a promessa (p = q verdade); finalmente,
 se a filha não foi aprovada
 (p falso) e recebeu o carro (q verdade), o pai também não descumpriu a
 promessa e, portanto p = q
 é verdadeiro (nesse caso, o pai pode ter entendido que a filha, mesmo
 não tendo sido aprovada,
 merecia, pelo resultado obtido, o prêmio - foi a primeira dos não
 aprovados, por exemplo).

 Como p = q só é falso se p é verdadeiro e q é falso, a demonstração de
 uma assertiva do tipo
 se p então q pode ser feita supondo-se que p é verdade e provando
 que, a partir daí, q também o é.
 Normalmente, o predicado p é chamado hipótese (que é o que se supõe
 ser verdadeiro) e o
 predicado q é chamado tese (que é o que se quer provar que é verdadeiro).


 ==
 Adolfo Neto
 Departamento Acadêmico de Informática
 Universidade Tecnológica Federal do Paraná
 Fone: (41) 3310-4644 / Fax: (41) 3310-4646
 Web: http://www.dainf.ct.utfpr.edu.br/~adolfo
 Blog: http://professoradolfo.blogspot.com
 Twitter: http://twitter.com/adolfont
 ==
 ___
 Logica-l mailing list
 Logica-l@dimap.ufrn.br
 http://www.dimap.ufrn.br/cgi-bin/mailman/listinfo/logica-l




-- 
Dr. Ricardo Pereira Tassinari - Departamento de Filosofia
UNESP - Faculdade de Filosofia e Ciências - Marília
Homepage: http://www.marilia.unesp.br/ricardotassinari
___
Logica-l mailing list
Logica-l@dimap.ufrn.br
http://www.dimap.ufrn.br/cgi-bin/mailman/listinfo/logica-l


Re: [Logica-l] [Bulk] Re: Explicação do sign ificado da implicação clássica

2010-01-28 Por tôpico Ricardo Pereira Tassinari
 qualquer exigência de
 relevância do antecedente para o consequente é descartada. O problema é que
 há inúmeros exemplos de condicionais que são verdadeiros ou falsos em função
 da relevância do antecedente para o consequente. Por exemplo, a frase Se
 Fernando Pessoa é australiano, então ele é português é falsa, pois exprime
 uma conexão geográfica incorreta. O problema é que a partir da lógica
 clássica teremos que dizer que é uma frase condicional verdadeira, pois o
 consequente ele é português é verdadeiro.


Ok, e esta é uma das motivações da Lógica da Relevância...



 É importante entender o significado dessa discussão sobre a adequação da
 condicional natural. Isso não é uma disputa entre partidários da lógica
 clássica e defensores de lógicas não-clássicas mais desviantes, nem é uma
 disputa entre partidários de uma concepção de linguagem comum e partidários
 de uma concepção de linguagem ideal. Um defeito na lógica clássica não
 estabelece a vantagem de lógicas rivais nem implica que devemos confiar
 apenas numa noção intuitiva de validade. A questão é saber se a condicional
 material é uma tradução correta da condicional natural. A discussão de
 revisão ou não da lógica clássica e propostas de novos sistemas formais que
 captem melhor o funcionamento semântico da condicional natural é uma
 discussão que é relacionada, mas é diferente. É claro que esta também é uma
 discussão importante, sobretudo em semântica formal. Basta ver a quantidade
 enorme de propostas e sistemas na literatura de lógica dos condicionais. Só
 pra citar alguns nomes pensem nos sistemas de  Belnap e Anderson, N.
 Rescher, Ernest Adams e William Cooper.


Ok. E repedindo o que escrevi no outro e-mail:

Penso que, na linguagem natural, não temos apenas UM tipo de implicação, mas
VÁRIOS.

E penso que a forma do pensar ultrapassa os sistemas formais, que descrevem
apenas uma PARTE dessa FORMA.



 Abs

 Matheus


Abraços,
Ricardo.


  Original Message -

 *From:* Ricardo Pereira Tassinari rica...@cle.unicamp.br
 *To:* Adolfo Neto ado...@utfpr.edu.br
 *Cc:* Lista acadêmica brasileira dos profissionais e estudantes da área de
 LOGICA logica-l@dimap.ufrn.br
 *Sent:* Thursday, January 28, 2010 3:49 PM
 *Subject:* [Bulk] Re: [Logica-l]Explicação do significado da implicação
 clássica

 Olá Adolfo e a todos.

 Abaixo uma pequena reflexão que gostaria de compartilhar sobre a implicação
 na linguagem natural entre duas sentenças (que designarei por se A então
 B) e o conectivo condicional.

 Notemos então que o condicional pode ser parafraseado, em linguagem natural
 por: não é o caso que A e não B, já que ele é equivalente a ~(A~B).

 Quando passamos DA IMPLICAÇÃO DA LINGUAGEM NATURAL PARA O CONECTIVO
 CONDICIONAL, NÃO TEMOS PROBLEMA (como no exemplo que citou ou em Se chove,
 então a rua está molhada), pois, se A então B, então temos que: não é o
 caso que A e não B. Exemplo, se assumimos que se chove, então a rua está
 molhada, podemos sempre concluir que não é o caso que chove e a rua não
 está molhada.

 A VOLTA QUE É PROBLEMÁTICA, OU SEJA, PASSAR DO CONECTIVO CONDICIONAL PARA A
 IMPLICAÇÃO NA LINGUAGEM NATURAL, ou ainda, passar de não é o caso que A e
 não B para se A então B. Exemplo: todos nós concordamos que não é o caso
 que a Lua é de queijo e o Sol é de gelo, mas nos é estranho dizer que se a
 Lua é de queijo então o Sol é de gelo.

 Ou seja, assumir que existe uma implicação na linguagem natural é
 condição suficiente (mas não necessária) para assumirmos uma implicação
 condicional.

 Assim, se um argumento é válido na linguagem natural (isto é se assumir as
 premissas implica em assumir a conclusão), então sua formalização com o
 condicional (conjunção das premissas condicional conclusão) será sempre
 verdadeira; por isso, se o argumento é válido, então sua formalização tem
 que ser, isto é, nunca encontraremos premissas verdadeiras e conclusão
 falsa, mas nem sempre uma argumento formalmente válido fica bem em linguagem
 natural.

 Abraço a todos,
 Ricardo.

 2010/1/28 Adolfo Neto ado...@utfpr.edu.br

 Gostei do exemplo abaixo...

 Fonte: Introdução à Álgebra Abstrata, de Jaime Evaristo
 http://professor.ic.ufal.br/jaime/livros/Capitulo%25201.pdf
 p.11-12

 O exemplo a seguir mostra que o significado matemático do se então,
 embora inusitado, tem
 sentido também no nosso dia a dia. Imagine que um pai anuncie para sua
 filha que vai fazer o
 vestibular para um curso de Medicina: se você passar, então eu lhe dou um
 carro.

 Se a filha foi aprovada (p verdade) e recebeu o carro (q verdade), o
 pai cumpriu a promessa
 (p = q verdade); se a filha foi aprovada (p verdade) e não recebeu o
 carro (q falso), o pai
 descumpriu a promessa (p = q falso); se a filha não foi aprovada (p
 falso) e não recebeu o carro (q
 falso), o pai não descumpriu a promessa (p = q verdade); finalmente,
 se a filha não foi aprovada
 (p falso) e recebeu o carro (q verdade), o pai também não descumpriu a
 promessa e, portanto p = q
 é verdadeiro

Re: [Logica-l] [Bulk] Re: Explicação do sign ificado da implicação clássica

2010-01-28 Por tôpico Ricardo Pereira Tassinari
 argumento em que essa afirmação for a conclusão. E, vice versa,
 dada qualquer afirmação contendo um se, então, se ela ocorre como a
 conclusão de um argumento e a substituição de se, então pela condicional
 material é inofensiva, então a substituição não será inofensiva em qualquer
 argumento em que essa afirmação apareceu como uma premissa. Ao dizer que a
 substituição de se, então pela condicional material é inofensiva, eu quero
 dizer que se o argumento em que a condicional material ocorre é válido de
 acordo com o cálculo proposicional, então o argumento em que se, então
 aparece é válido.


 Penso que o exemplo que o Adolfo apresentou é fraco se comparado com
 outros exemplos da linguagem natural que mostram inadequação da condicional
 material. Para citar só dois exemplos.

 1) Todo mundo conhece a regra de fortalecimento do antecedente que nos
 permite a inferência: P então Q, logo (P e R) então Q
 **
 O problema é que se aceitarmos isso teremos que dizer que os argumentos
 abaixo são válidos:


 ‘Se risco o fósforo, ele ascenderá’. Logo, ‘se mergulho o fósforo por uma
 noite inteira na água e o risco, então ele ascenderá’.



 “Se eu colocar açúcar no meu café ele ficará saboroso. Se eu colocar
 açúcar e óleo diesel no meu café, ele fiará saboroso”.

 **

 2) Sabemos que a condicional material é um cáculo verofuncional a partir
 do valor de verdade de suas partes componentes. Não há meios de avaliar o
 conteúdo das condicionais na tabela de verdade e qualquer exigência de
 relevância do antecedente para o consequente é descartada. O problema é que
 há inúmeros exemplos de condicionais que são verdadeiros ou falsos em função
 da relevância do antecedente para o consequente. Por exemplo, a frase Se
 Fernando Pessoa é australiano, então ele é português é falsa, pois exprime
 uma conexão geográfica incorreta. O problema é que a partir da lógica
 clássica teremos que dizer que é uma frase condicional verdadeira, pois o
 consequente ele é português é verdadeiro.



 É importante entender o significado dessa discussão sobre a adequação da
 condicional natural. Isso não é uma disputa entre partidários da lógica
 clássica e defensores de lógicas não-clássicas mais desviantes, nem é uma
 disputa entre partidários de uma concepção de linguagem comum e partidários
 de uma concepção de linguagem ideal. Um defeito na lógica clássica não
 estabelece a vantagem de lógicas rivais nem implica que devemos confiar
 apenas numa noção intuitiva de validade. A questão é saber se a condicional
 material é uma tradução correta da condicional natural. A discussão de
 revisão ou não da lógica clássica e propostas de novos sistemas formais que
 captem melhor o funcionamento semântico da condicional natural é uma
 discussão que é relacionada, mas é diferente. É claro que esta também é uma
 discussão importante, sobretudo em semântica formal. Basta ver a quantidade
 enorme de propostas e sistemas na literatura de lógica dos condicionais. Só
 pra citar alguns nomes pensem nos sistemas de  Belnap e Anderson, N.
 Rescher, Ernest Adams e William Cooper.



 Abs

 Matheus

  Original Message -

  *From:* Ricardo Pereira Tassinari rica...@cle.unicamp.br
 *To:* Adolfo Neto ado...@utfpr.edu.br
 *Cc:* Lista acadêmica brasileira dos profissionais e estudantes da área
 de LOGICA logica-l@dimap.ufrn.br
 *Sent:* Thursday, January 28, 2010 3:49 PM
 *Subject:* [Bulk] Re: [Logica-l]Explicação do significado da implicação
 clássica

 Olá Adolfo e a todos.

 Abaixo uma pequena reflexão que gostaria de compartilhar sobre a
 implicação na linguagem natural entre duas sentenças (que designarei por se
 A então B) e o conectivo condicional.

 Notemos então que o condicional pode ser parafraseado, em linguagem
 natural por: não é o caso que A e não B, já que ele é equivalente a
 ~(A~B).

 Quando passamos DA IMPLICAÇÃO DA LINGUAGEM NATURAL PARA O CONECTIVO
 CONDICIONAL, NÃO TEMOS PROBLEMA (como no exemplo que citou ou em Se chove,
 então a rua está molhada), pois, se A então B, então temos que: não é o
 caso que A e não B. Exemplo, se assumimos que se chove, então a rua está
 molhada, podemos sempre concluir que não é o caso que chove e a rua não
 está molhada.

 A VOLTA QUE É PROBLEMÁTICA, OU SEJA, PASSAR DO CONECTIVO CONDICIONAL PARA
 A IMPLICAÇÃO NA LINGUAGEM NATURAL, ou ainda, passar de não é o caso que A e
 não B para se A então B. Exemplo: todos nós concordamos que não é o caso
 que a Lua é de queijo e o Sol é de gelo, mas nos é estranho dizer que se a
 Lua é de queijo então o Sol é de gelo.

 Ou seja, assumir que existe uma implicação na linguagem natural é
 condição suficiente (mas não necessária) para assumirmos uma implicação
 condicional.

 Assim, se um argumento é válido na linguagem natural (isto é se assumir as
 premissas implica em assumir a conclusão), então sua formalização com o
 condicional (conjunção das premissas condicional conclusão) será sempre
 verdadeira; por isso, se o argumento é válido, então sua formalização tem
 que ser

Re: [Logica-l] Piaget

2010-01-28 Por tôpico Ricardo Pereira Tassinari
Oi Décio.

Na realidade são dois volumes:

Piaget, Jean. - Recherches sur la contradiction - Paris : Presses univ. de
France, 1974. - 2 vol. ; 22 cm. - (Etudes d'épistémologie génétique ; 31 ;
32)
1: Les différentes formes de la contradiction / avec la collab. de Cl.-L.
Bonnet... [et al.].- 1974. - 147 p.
2: Les relations entre affirmations et négations / avec la collab. de A.
Blanchet... [et al.]. - 1974. - 180 p.

Tem as traduções (Inglês e Espanhol):

Piaget, Jean. Experiments in contradiction. Chicago ; London : The
University of Chicago Press, 1980.

Piaget, Jean. Investigaciones sobre la contradiccióncon. Madrid : Siglo
veintiuno ed., 1978.

Não conheço tradução para o Português.

Um abraço,
Ricardo.


2010/1/28 Decio Krause deciokra...@gmail.com

 Ricardo
 Você sabe se o Piaget, Recherches sur la contradiction, Paris, Presses
 univ. de France, 1974 tem tradução para o inglês ou mesmo para o português?
 Ficaria (no caso do inglês) mais fácil de achar no gigapedia...ou nas
 livrarias (no caso do português). Eu não conheço e gostaria de saber o que
 ele fala do assunto.
 Abraço,
 Décio

 
 Decio Krause
 Departamento de Filosofia
 Universidade Federal de Santa Catarina
 88040-990 Florianópolis, SC -- Brasil
 deciokra...@gmail.com
 www.cfh.ufsc.br/~dkrause
 
 *Doctor Bell say we’re connected,*
 *He called me on the phone,*
 *But if we’re really together baby,*
 *How can I feel so all alone?*
 (Bell's Theorem Blues)

 Em 28/01/2010, às 15:06, Ricardo Pereira Tassinari escreveu:

 Piaget, Recherches sur la contradiction, Paris, Presses univ. de France,
 1974).





-- 
Dr. Ricardo Pereira Tassinari - Departamento de Filosofia
UNESP - Faculdade de Filosofia e Ciências - Marília
Homepage: http://www.marilia.unesp.br/ricardotassinari
___
Logica-l mailing list
Logica-l@dimap.ufrn.br
http://www.dimap.ufrn.br/cgi-bin/mailman/listinfo/logica-l


Re: [Logica-l] ZF foundations of consciousness

2010-01-26 Por tôpico Ricardo Pereira Tassinari
Olá a todos.

Estou lendo o artigo abaixo. Se alguém quiser, podemos trocar informações e
discussões sobre ele.

Agradeço ao Jean-Yves pela referência.

Abraço a todos,
Ricardo.

2009/12/23 BEZIAU Jean-Yves jean-yves.bez...@unine.ch

 By Willard L. Miranker, Gregg J. Zuckerman
 in Journal of Applied Logic, Volume 7, Issue 4 (2009)
 We employ the Zermelo–Fränkel Axioms that characterize sets as mathematical
 primitives. The Anti-foundation Axiom plays a significant role in our
 development, since among other of its features, its replacement for the
 Axiom of Foundation in the Zermelo–Fränkel Axioms motivates Platonic
 interpretations. These interpretations also depend on such allied notions
 for sets as pictures, graphs, decorations, labelings and various mappings
 that we use. A syntax and semantics of operators acting on sets is
 developed. Such features enable construction of a theory of non-well-founded
 sets that we use to frame mathematical foundations of consciousness. To do
 this we introduce a supplementary axiomatic system that characterizes
 experience and consciousness as primitives. The new axioms proceed through
 characterization of so-called consciousness operators. The Russell operator
 plays a central role and is shown to be one example of a consciousness
 operator. Neural networks supply striking examples of non-well-founded
 graphs the decorations of which generate associated sets, each with a
 Platonic aspect. Employing our foundations, we show how the supervening of
 consciousness on its neural correlates in the brain enables the framing of a
 theory of consciousness by applying appropriate consciousness operators to
 the generated sets in question.
 ___
 Logica-l mailing list
 Logica-l@dimap.ufrn.br
 http://www.dimap.ufrn.br/cgi-bin/mailman/listinfo/logica-l




-- 
Dr. Ricardo Pereira Tassinari - Departamento de Filosofia
UNESP - Faculdade de Filosofia e Ciências - Marília
Homepage: http://www.marilia.unesp.br/ricardotassinari
___
Logica-l mailing list
Logica-l@dimap.ufrn.br
http://www.dimap.ufrn.br/cgi-bin/mailman/listinfo/logica-l


Re: [Logica-l] Fwd: Por que a maioria das pessoas acha a matemática tão difícil? vídeo da Globonews

2010-01-25 Por tôpico Ricardo Pereira Tassinari
. Ler e reler. Fazer
  exercícios. Nem jogador de futebol consegue ser bom se não treinar.
  Estou tendo problemas sérios com minhas turmas de lógica para computação
  (vejam os Dados de aprovação/reprovação em Lógica para Computação
  http://bit.ly/8cMKbd ). Claro que sempre parte da culpa (talvez boa
 parte) é
  do professor. Mas parece que cada vez mais os alunos não querem se
 esforçar,
  isto é, sentar e estudar.
  []s
  Adolfo
 
  ==
  Adolfo Neto
  Departamento Acadêmico de Informática
  Universidade Tecnológica Federal do Paraná
  Fone: (41) 3310-4644 / Fax: (41) 3310-4646
  Web: 
  http://www.dainf.ct.utfpr.edu.br/~adolfohttp://www.dainf.ct.utfpr.edu.br/%7Eadolfo
  Blog: http://professoradolfo.blogspot.com
  Twitter: http://twitter.com/adolfont
  ==
 
 
 
  2010/1/22 Francisco Antonio Doria famado...@gmail.com
 
  Já comentei: tenho que ler três, quatro vezes o texto. Dou um exemplo:
  tive que ler ***cinco*** vezes o artigo do Newton sobre estruturas (com
 o
  Chuaqui) para começar a entender...
 
  2010/1/22 Valeria de Paiva valeria.depa...@gmail.com
 
  esqueci de mandar pra lista...
 
  -- Forwarded message --
  From: Valeria de Paiva valeria.depa...@gmail.com
  Date: 2010/1/22
  Subject: Re: [Logica-l] Por que a maioria das pessoas acha a
 matemática
  tão difícil? vídeo da Globonews
  To: Francisco Antonio Doria famado...@gmail.com
 
 
  Bom, eu tambem tenho *muita* dificuldade, mas a culpa nao e' da
  matematica...e' minha mesmo.
  Valeria
 
  2010/1/22 Francisco Antonio Doria famado...@gmail.com
 
  Bom, cês são felizes. Tenho dificuldade, sim.
 
  2010/1/22 Valeria de Paiva valeria.depa...@gmail.com
 
  Ja' eu concordo com o Adolfo que matematica do jeito certo nao 'e
  dificil, e' arte pura.
 
  Uma pessoa que descreve bem e' o Lockhart, que pode ser lido a
 partir
  de:
  http://www.maa.org/devlin/devlin_03_08.html.
 
  (Eu nao concordo com tudo o que ele diz sobre educacao matematica,
 mas
  a parte em que ele descreve experiencia matematica, eu acho otima!)
   deem uma olhada...
 
  Valeria
 
  2010/1/21 Francisco Antonio Doria famado...@gmail.com
 
  Não. Matemática ***é*** difícil.
 
  2010/1/21 Adolfo Neto ado...@utfpr.edu.br
 
  Por que a maioria das pessoas acha a matemática tão difícil?
 
  A matemática está na nossas vida mais do que imaginamos.
 
  Mario Livio, diretor do telescópio Hubble, acredita que o sistema
  educacional deixa as pessoas com medo, e isso cria bloqueios na
 hora de
  aprender matemática.
 
  http://bit.ly/4UAfGh
 
  No início e principalmente no final aborda-se a lógica na
 linguagem
  usando exemplo do filme Shrek Terceiro
  ==
  Adolfo Neto
  Departamento Acadêmico de Informática
  Universidade Tecnológica Federal do Paraná
  Fone: (41) 3310-4644 / Fax: (41) 3310-4646
  Web: 
  http://www.dainf.ct.utfpr.edu.br/~adolfohttp://www.dainf.ct.utfpr.edu.br/%7Eadolfo
  Blog: http://professoradolfo.blogspot.com
  Twitter: http://twitter.com/adolfont
  ==
 
 
  ___
  Logica-l mailing list
  Logica-l@dimap.ufrn.br
  http://www.dimap.ufrn.br/cgi-bin/mailman/listinfo/logica-l
 
 
 
  ___
  Logica-l mailing list
  Logica-l@dimap.ufrn.br
  http://www.dimap.ufrn.br/cgi-bin/mailman/listinfo/logica-l
 
 
 
 
  --
  Valeria de Paiva
  http://www.cs.bham.ac.uk/~vdp/ http://www.cs.bham.ac.uk/%7Evdp/
  http://valeriadepaiva.org/www/
 
  ___
  Logica-l mailing list
  Logica-l@dimap.ufrn.br
  http://www.dimap.ufrn.br/cgi-bin/mailman/listinfo/logica-l
 
 
 
 
 
  --
  Valeria de Paiva
  http://www.cs.bham.ac.uk/~vdp/ http://www.cs.bham.ac.uk/%7Evdp/
  http://valeriadepaiva.org/www/
 
 
 
  --
  Valeria de Paiva
  http://www.cs.bham.ac.uk/~vdp/ http://www.cs.bham.ac.uk/%7Evdp/
  http://valeriadepaiva.org/www/
 
  ___
  Logica-l mailing list
  Logica-l@dimap.ufrn.br
  http://www.dimap.ufrn.br/cgi-bin/mailman/listinfo/logica-l
 
 
 
  ___
  Logica-l mailing list
  Logica-l@dimap.ufrn.br
  http://www.dimap.ufrn.br/cgi-bin/mailman/listinfo/logica-l
 
 
 
  ___
  Logica-l mailing list
  Logica-l@dimap.ufrn.br
  http://www.dimap.ufrn.br/cgi-bin/mailman/listinfo/logica-l
 
 
 ___
 Logica-l mailing list
 Logica-l@dimap.ufrn.br
 http://www.dimap.ufrn.br/cgi-bin/mailman/listinfo/logica-l



 ___
 Logica-l mailing list
 Logica-l@dimap.ufrn.br
 http://www.dimap.ufrn.br/cgi-bin/mailman/listinfo/logica-l




-- 
Dr. Ricardo Pereira Tassinari - Departamento de Filosofia
UNESP - Faculdade de Filosofia e Ciências - Marília
Homepage: http://www.marilia.unesp.br/ricardotassinari

Re: [Logica-l] Fwd: Por que a maioria das pessoas acha a matemática tão difícil? vídeo da Globonews

2010-01-25 Por tôpico Ricardo Pereira Tassinari
Eu é que estou impressionado com as operações lógicas fundamentais de
Piaget e a rigidez do sistema dele. Sinceramente, não sei o que andou
lendo... Ao meu ver  parece apenas um comentário preconceituoso.

Abraço,
Ricardo.

2010/1/25 Francisco Antonio Doria famado...@gmail.com

 Foi inspirada nele, essa de ``operações lógicas fundamentais,'' coisa
 assim. Li Piaget há muito tempo, e me lembro que fiquei impressionado com a
 rigidez do sistema dele.

 2010/1/25 Ricardo Pereira Tassinari rica...@cle.unicamp.br

 Olá Dória.

 Não conheço o livro (ou artigo) em que Piaget inventa a Matemática
 Moderna.

 Você saberia me dar a referência?

 Abraço,
 Ricardo.

 2010/1/25 Francisco Antonio Doria famado...@gmail.com

 1) Matemática ***´e*** difícil.

 2) Foi Piaget quem inventou essa da `matemática moderna.' Vivam Trivium e
 Quadrivium!

 2010/1/25 Carlos Gonzalez gonza...@gmail.com

 Caríssimos,

 Eu acho que o nome do vídeo (e desta discussão) deveria ser: por que
 a maioria das pessoas não gosta da matemática, porque senão tem razão
 o Doria: a pessoas acham a matemática difícil porque as equações
 diferenciais são difíceis e tem equações diferenciais que a maioria
 das pessoas não vão conseguir solucionar. Ponto final.

 O vídeo não tem esse tema, e o Lívio esclarece que esse não é o
 problema. Ele fala com muita clareza dos bloqueios produzidos pelo
 ensino da matemática. Eu, que já lecionei matemática, encontrei
 numerosas vezes esses bloqueios psicológicos: a maioria fica
 paralisado (como se tiver medo), outros tem claros sinais de rejeição
 (alguns deles chegam a ter expressões faciais de desgosto o como se
 tiveram alguma coisa amarga na boca). Eu teve alunos que me disseram
 que perderam o medo da matemática, outros que ensinei a gostar da
 matemática, coisa que achavam impossível.

 No ensino da lógica, e de questões computacionais que tem a ver com a
 lógica e a matemática, também encontramos bloqueios em muitas pessoas.

 Uma coisa que fala o vídeo é que tem professor de matemática que não
 gosta da matemática. Eu vi isso como professor numa licenciatura em
 matemática, na qual muitos alunos não gostavam da matemática. Tem
 muita vaga como professor de matemática, em 3 anos se forma, presta
 concurso e tem emprego estável. Possivelmente todos nós temos sofrido
 em algum momento um professor de matemática que não gosta de
 matemática. Imagina um cozinheiro que não experimenta a comida que
 está fazendo porque detesta essa comida. Por isso eu não acredito que
 a aplicabilidade da matemática como motivação principal. Seria como
 dizer: faça uma comida gostosa que vai saciar sua fome. Quando a
 pessoa estuda matemática, por exemplo, porque gosta de física, tem uma
 certa tendência a este ponto de vista. Por favor, desculpem a minhas
 palavras, mas eu fico com um sentimento como se falassem: vamos
 solucionar essa equação diferencial nojenta, assim podemos, por fim,
 conhecer esse orbital molecular. Ou calcula todos esses espantosos
 vetores de forças, assim vamos saber se a ponte não cai. Tipo: a
 matemática é um remédio amargo que tem que ser engolido para poder se
 divertir.

 Eu acho que os dois principais atrativos da matemática são a beleza e
 o desafio em solucionar problemas.

 Suponha que usamos algum livro de Smullyan para passar para os alunos
 charadas lógicas. Vários alunos vão se interessar, enquanto outros vão
 querer saber a solução rápido e acaba logo com isso. Eu colocava o
 paralelo com o futebol. É fácil fazer um gol? Ganha todas as partidas?
 Não termina muitos jogos cansado e com dores musculares? Mas a alegria
 de fazer um gol, a alegria de ganhar um jogo, compensa tudo isso, se
 não as pessoas não jogariam futebol. Temos que encarar a questão de
 solucionar problemas matemáticos e lógicos com o mesmo espírito
 esportivo.

 Eu gosto de matemática desde o primeiro grau. Mas foi vítima da
 matemática moderna. Quando tinha 13 anos, um professor de matemática
 explicou (muito mal explicado) grupos, anéis, etc. Eu odiei. Confesso,
 eu também perguntei para que serve?. Esse povo da matemática moderna
 esqueceu de Pestalozzi, de Piaget e de um monte de outras pessoas. Sem
 um treino e um domínio básico de técnicas dedutivas, não faz sentido
 ensinar estruturas algébricas. Quando estudei, em sério, teoria de
 grupos (depois de estudar lógica e técnicas básicas de demonstração) e
 vi a demonstração de unicidade do elemento neutro, eu gostei. Teve que
 dar marcha ré no meu ódio por estruturas algébricas e ver a beleza que
 elas tem. Como uma estrutura tão simples como um grupo pode estar
 presente na aritmética, nas permutações, nas simetrias e em muitos
 outros lugares? Por que no crescimento de uma planta podemos encontrar
 a sequência de Fibonacci? Etc. etc. etc.

 A pergunta para que serve? e didática baseada na utilidade e
 aplicabilidade de matemática, são um testimonium paupertatis de um
 ensino da matemática sem beleza, sem desafio e sem raciocínio.

 Este é o meu ponto de vista, mas

Re: [Logica-l] FW: Lógica, Matemática e Psicol ogia.

2009-12-21 Por tôpico Ricardo Pereira Tassinari
 EXPLICAÇÃO DO COMPORTAMENTO HUMANO, teremos de
nos contentar com o Método de Aproximação acima e mesmo com a aproximação ao
conjunto de leis C = f (E) = f (P, A) que determinariam o comportamento
humano.

Um forte abraço,
Ricardo.



2009/12/19 Rodrigo Oliveira rodrigo@hotmail.com


 Olá Ricardo,

 Estou lendo este livro e já havia pensado em lhe perguntar a respeito, pois
 vi o vocabulário na sua página. Se pudesse, gostaria de vê-lo falar um pouco
 mais sobre a psicologia topológica e vetorial. Que perspectivas você vê?
 Fala-se que a teoria dos jogos talvez pudesse servir como um modelo para a
 interação humana. Que acha disso? Ainda não terminei o livro, mas a proposta
 dele sem dúvida é atraente.

 Abraço

 Rodrigo


 --
 Date: Sat, 19 Dec 2009 14:03:11 -0200
 From: rica...@cle.unicamp.br
 To: logica-l@dimap.ufrn.br
 Subject: [Logica-l] Lógica, Matemática e Psicologia.

 Olá a todos.

 Para aqueles que gostam do tema da utilização da Matemática e da Lógica em
 Psicologia (ou mesmo de fundamentos da Psicologia) recomendo fortemente o
 livro de Kurt Lewin, *Princípios de Psicologia Topolótica* (
 http://books.google.com.br/books?id=4fbaSj481NwCprintsec=frontcoverdq=topological+psychologycd=1#v=onepageq=f=false)
 que tem tradução para o Português (
 http://www.traca.com.br/livro/57074/principios-de-psicologia-topologica) e
 que, apesar de esgotada, pode ser achada facilmente nos sebos.

 Pode-se encontrar o vocabulário desse livro em meu site (
 http://www.marilia.unesp.br/Home/Instituicao/Docentes/RicardoTassinari/LewinV.htm
 ).

 Existem ainda outros livros dele nos quais ele aplica os conceitos desse
 livro.

 Acho a Psicologia Topológica e Vetorial criada por ele genial e acho que a
 Psicologia perde muito em objetividade (enquanto ciência) por não dar
 continuidade aos seus trabalhos (ou mesmo por não criticá-los de forma
 consistente); e não dá para entender, já que sua matemática é simples, pois
 sendo topologias em conjuntos finitos, trata-se apenas de teoria de
 conjuntos finitos.

 Quem quiser pesquisar e conversar sobre o tema, é uma psicologia que gosto
 muito.

 Abraços,
 Ricardo.
 --
 Dr. Ricardo Pereira Tassinari - Departamento de Filosofia
 UNESP - Faculdade de Filosofia e Ciências - Marília
 Homepage: http://www.marilia.unesp.br/ricardotassinari


 --
 Chegou Windows 7. Agora com exibição de redes sem fio. 
 Conheça.http://www.microsoft.com/brasil/windows7/default.html?WT.mc_id=1539
 --
 Chegou Windows 7. Agora com exibição de redes sem fio. 
 Conheça.http://www.microsoft.com/brasil/windows7/default.html?WT.mc_id=1539

 ___
 Logica-l mailing list
 Logica-l@dimap.ufrn.br
 http://www.dimap.ufrn.br/cgi-bin/mailman/listinfo/logica-l




-- 
Dr. Ricardo Pereira Tassinari - Departamento de Filosofia
UNESP - Faculdade de Filosofia e Ciências - Marília
Homepage: http://www.marilia.unesp.br/ricardotassinari
attachment: Fig1.pngattachment: Fig2.pngattachment: Fig3.pngattachment: Fig4.pngattachment: Fig5.pngattachment: Fig6.png___
Logica-l mailing list
Logica-l@dimap.ufrn.br
http://www.dimap.ufrn.br/cgi-bin/mailman/listinfo/logica-l


[Logica-l] Lógica, Matemática e Psicologia .

2009-12-19 Por tôpico Ricardo Pereira Tassinari
Olá a todos.

Para aqueles que gostam do tema da utilização da Matemática e da Lógica em
Psicologia (ou mesmo de fundamentos da Psicologia) recomendo fortemente o
livro de Kurt Lewin, *Princípios de Psicologia Topolótica* (
http://books.google.com.br/books?id=4fbaSj481NwCprintsec=frontcoverdq=topological+psychologycd=1#v=onepageq=f=false)
que tem tradução para o Português (
http://www.traca.com.br/livro/57074/principios-de-psicologia-topologica) e
que, apesar de esgotada, pode ser achada facilmente nos sebos.

Pode-se encontrar o vocabulário desse livro em meu site (
http://www.marilia.unesp.br/Home/Instituicao/Docentes/RicardoTassinari/LewinV.htm
).

Existem ainda outros livros dele nos quais ele aplica os conceitos desse
livro.

Acho a Psicologia Topológica e Vetorial criada por ele genial e acho que a
Psicologia perde muito em objetividade (enquanto ciência) por não dar
continuidade aos seus trabalhos (ou mesmo por não criticá-los de forma
consistente); e não dá para entender, já que sua matemática é simples, pois
sendo topologias em conjuntos finitos, trata-se apenas de teoria de
conjuntos finitos.

Quem quiser pesquisar e conversar sobre o tema, é uma psicologia que gosto
muito.

Abraços,
Ricardo.
-- 
Dr. Ricardo Pereira Tassinari - Departamento de Filosofia
UNESP - Faculdade de Filosofia e Ciências - Marília
Homepage: http://www.marilia.unesp.br/ricardotassinari
___
Logica-l mailing list
Logica-l@dimap.ufrn.br
http://www.dimap.ufrn.br/cgi-bin/mailman/listinfo/logica-l


Re: [Logica-l] Comunicado- novo Diretor do Centro de L ógica, Epistemologia e História da Ciência (CLE) da Unicamp

2009-10-09 Por tôpico Ricardo Pereira Tassinari
Gostaria de parabenizar o Prof. Walter Carnielli pelo resultado da eleição
de Diretor do CLE, em especial, pela expressiva votação a seu favor (90%).

Abraço a todos e ao Walter em especial.
Ricardo Tassinari.


Parabéns pela

2009/10/8 Marcelo Esteban Coniglio conig...@cle.unicamp.br

 Comunicado- novo Diretor do Centro de Lógica, Epistemologia e História
 da Ciência (CLE) da Unicamp


 Gostaríamos de  comunicar  que o prof. Walter Carnielli, Professor
 Titular do Departamento de Filosofia do Instituto de Filosofia e
 Ciências Humanas (IFCH) da Unicamp, foi designado, após consulta aos
 membros, para exercer a função de Diretor do Centro de Lógica,
 Epistemologia e História da Ciência (CLE) da Unicamp, no período de 30
 de setembro de 2009 a 30 de setembro de 2012.
 Participaram do Processo de Consulta, 43 membros do CLE. O Prof.
 Carnielli obteve 90% dos votos (39 votos).

 Atenciosamente,


 Marcos Antonio Munhoz
 Assistente Técnico
 Centro de Lógica, Epistemologia e História da Ciência – CLE
 Fone (19) 3521-6518

 ___
 Logica-l mailing list
 Logica-l@dimap.ufrn.br
 http://www.dimap.ufrn.br/cgi-bin/mailman/listinfo/logica-l




-- 
Dr. Ricardo Pereira Tassinari - Departamento de Filosofia
UNESP - Faculdade de Filosofia e Ciências - Marília
Homepage: http://www.marilia.unesp.br/ricardotassinari
___
Logica-l mailing list
Logica-l@dimap.ufrn.br
http://www.dimap.ufrn.br/cgi-bin/mailman/listinfo/logica-l


Re: [Logica-l] V axioma de Frege e o paradoxo de Russel

2009-09-28 Por tôpico Ricardo Pereira Tassinari
Olá Rodrigo.

Dê uma olhada nas traduções das cartas entre Russell e Frege no site da Olga
Pombo: http://www.educ.fc.ul.pt/docentes/opombo/traducoes/index.htm

Abraços,
Ricardo.

2009/9/27 Rodrigo Oliveira rodrigo@hotmail.com


 Alguém pode me dar uma idéia, ou indicar uma referência, mais detalhada de
 como o paradoxo de Russel invalida o V axioma de Frege?

 Sei o básico, que o V axioma pressupoe que conjuntos possam ser elementos
 de outros conjuntos e que (por 1 axioma da teoria de conjuntos) dada uma
 propriedade bem definida podemos criar um cojunto cujos elementos serão
 aqueles que possuem tal propriedade e assim criamos o conjunto cujos
 elementos são os conjuntos que não são elementos de si mesmo..., mas
 gostaria de ouvir mais sobre o assunto se houer alguém interessado. Sei de
 algumas tentativas de reconstruir a demonstração de Frege e evitar a
 inconsistência, o que dizem sobre isso?

 Rodrigo

 --
 Quer uma internet mais segura? Baixe agora o novo Internet Explorer 8. É
 grátis!http://brasil.microsoft.com.br/IE8/mergulhe/?utm_source=MSN%3BHotmailutm_medium=Taglineutm_campaign=IE8

 ___
 Logica-l mailing list
 Logica-l@dimap.ufrn.br
 http://www.dimap.ufrn.br/cgi-bin/mailman/listinfo/logica-l




-- 
Dr. Ricardo Pereira Tassinari - Departamento de Filosofia
UNESP - Faculdade de Filosofia e Ciências - Marília
Homepage: http://www.marilia.unesp.br/ricardotassinari
___
Logica-l mailing list
Logica-l@dimap.ufrn.br
http://www.dimap.ufrn.br/cgi-bin/mailman/listinfo/logica-l


[Logica-l] Fwd: 2 problemas de tradução, e a fixação de terminologia

2009-08-13 Por tôpico Ricardo Pereira Tassinari
-- Forwarded message --
From: Ricardo Pereira Tassinari rica...@cle.unicamp.br
Date: 2009/8/13
Subject: Re: [Logica-l] 2 problemas de tradução, e a fixação de terminologia
To: Decio Krause deciokra...@gmail.com


Oi Décio.

2009/8/12 Decio Krause deciokra...@gmail.com

 Oi, RicardoObrigado pela aula sobre Piaget. O congresso vai ser bacana sem
 dúvida.
 Eu conheço a Epistemologia Genética e sempre invoquei (mas de leve, como
 suspeita apenas) com os que liam apenas a primeira parte, aquela dos
 estágios do desenvolvimento, pois acho que é difícil (ou impossível?) ler
 aquelas outras sobre as epistemologias da lógica, da matemática e da física
 sem ter um grande background, coisa que pedagogos e similares em geral não
 têm nessas disciplinas. Acho que ele se baseava muito nas opiniões de,
 p.ex., pessoas como Bar Hillel, que tabalhou com ele, mas não pescava bem
 essas coisas mais técnicas, apesar de ser genial como foi. Eu acho confuso
 lê-lo, mesmo hoje quando pego a EG e tento entender, por exemplo, a seção
 sobre a epistemologia da lógica, pois pressupõe várias coisas que, para
 quem conhece o assunto, são quase que triviais e, para quem não conhece,
 incompreensíveis.


Mesmo para quem conhece a EG, as seções que você citou não são nada
triviais, ao contrário.

De fato, Piaget deixa a desejar no que tange a parte sintática, ou melhor,
axiomática, das estruturas que visa apresentar, o que para um lógico ou
epistemólogo acostumado com a axiomatização causa certa confusão e, muitas
vezes, irritação. Lembro de, ao ler certas partes da obra dele, dizer isso
aqui não pode ser assim, é contraditório!. Lembro-me também de alguma
irritação do Prof. Leônidas Hegenberg quanto ao texto de Lógica do Piaget,
no tempo que ele fazia parte de nosso grupo de EG na USP. Há problemas na
exposição formal de certas estruturas (não de todas, pois algumas como o
Grupo INRC, ou mesmo do Grupo Prático de Deslocamente, estão bem feitas, ou
melhor, no mesmo nível que os físicos em geral fazem a sistematização de
suas teorias). Porém, nesses caso, como disse o Prof. Paul Ledergerber que
participou de minha banca de mestrado: há problemas com a sintaxe, mas não
com a semântica; trata-se então de arrumar a sintaxe.

Por outro lado, a Teoria de Piaget é bastante complexa e os conceitos
elaborados por ele se encontram espalhados por toda a sua extensa obra, em
torno de 95 livros e 600 artigos. Os livros ou são um tijolo, onde ele
lentamente vai desenvolvendo seu racicíono, expondo os resultados
experimentais junto com suas reflexões e conceitos que introduz, ou são
fininhos, mas extremamente densos, cheio de referências bibliográficas as
suas obras-tijolo. Esse é o caso do *A Epistemologia Genética* que você
citou.

Mas depois de muito estudá-lo, desde a orientação com a Prof. Zélia
Ramozzi-Chiorotino, que foi orientanda do Prof. Granger na França,
justamente sobre a obra de Piaget (dessa orientação resultou um livro
fininho dela que recomendo fortemente, *Piaget: Modelo e Estrutura*), posso
dizer que sua epistemologia é uma grande obra, nada ingênua. E que, apesar
do problema com certas formalizações, ele tem muita clareza das questões que
trata, mesmo, por exemplo, no caso das interpretações que faz dos teoremas
de Gödel.*

*Em geral, o pessoal tende a vê-lo como um psicólogo de criança (sem falar
na história dele ser pedagogo..., novamente, não sei de onde esse pessoal
tira essas idéias lendo a obra dele). Ele mesmo diz, tachativamente,
em *Sabedoria
e Ilusões da Filosofia*: Quanto a mim, decidi consagrar-me à filosofia
assim que a conheci! Mas reconheço que é não é fácil entender a obra de um
filósofo que, ao mesmo tempo: por um lado, elabora suas idéias filosófica
sobre o conhecimento; por outro, realiza experimentos e constroí uma teoria
psicológica. Ora, essas idéias filosóficas acabam só sendo entendidas se se
entende os experimentos e os modelos psicológicos que elabora em sua
psicologia; bem como, para entender sua psicologia, é necessário se despir
dos preconceitos filosóficos irrefletidos que a maioria dos cientistas têm,
a partir do preconceito geral na área, e entender o que ele está criticando
(um bom exemplo desse entrelaçamento se encontra no *Nascimento da
Inteligência na Criança*, principalmente na Introdução e na Conclusão, onde
ele discute ao mesmo tempo as principais correntes psicológicas de sua época
juntamente com os pressupostos epistemológicos que elas carregam, muitas
vezes sem o saber).
*
*

  Em todo caso, gosto dele, e de várias coisas se sua obra. Mas as relações
 entre os pensamentos dele e de Schrödinger, pelo que sei, jamais foram
 aventados. Conhece algo a respeito? Daria samba.


Não conheço a obra filosófica de Schrödinger, assim não posso julgar a
qualidade do samba. :)

Mas certamente, um confronto de idéias seria esclarecedor, não apenas do
pensamento de ambos, mas de como fazemos e compreendemos a MQ.


 Grato pela resposta.
 Abração,
 Décio

  Eu que agradeço,
um forte abraço

Re: [Logica-l] 2 problemas de tradução, e a fixa ção de terminologia

2009-08-12 Por tôpico Ricardo Pereira Tassinari
Olá Bruno.

2009/8/12 Bruno Woltzenlogel Paleo bruno.wp.mailingl...@googlemail.com

 Olá Ricardo,

 ---
 Bem, por outro lado, que são essas coisas sintáticas chamadas fórmulas?

 Há infinitas delas (!)
 e elas nem são exatamente objetos físicos, pois são tipos (type) e não
 ocorrências (token)!

 Se você acredita em na existência das fórmulas, então não é difícil
 acreditar na existência do Modelo Padrão.
 --

 Há infinitas formulas, mas elas sao recursivamente enumeraveis... Já as
 infinitas verdades do modelo padrao nao sao recursivamente enumeraveis.

 Eu nao quis dizer que nao acredito na existencia do modelo padrao. Meu
 ponto
 foi simplesmente de que a versao sintatica da incompletabilidade é mais
 clara pra mim do que a semantica. Fiz esse comentario porque voce havia
 comentado que pra voce ocorre justamente o contrario.  No fim, acho que
 acaba sendo uma questao de gosto ou costume... :-)


Estamos de acordo que o entendimento da incompletude sintática é mais
direto. Lembro-me de uma sentença do Shoenfield que diz que os conceitos da
matemática são usualmente muito abstratos e portanto difíceis de
compreender. Uma sentença, por outro lado, é um objeto concreto; então pelo
estudo dos axiomas como sentenças, nos abordamos o abstrato com o concreto.
Tinha percebido que você estava expressando essa característica. Mas, como
não me ficou claro a questão sobre a existência dos objetos matemáticos,
revolvi expor um argumento que acho interessante e que mostra, por um lado,
o quanto vai se caminhando do concreto para o abstrato de uma forma quase
contínua, tendo cada pesquisador de escolher onde parar, se não quer ser um
platonista total e, por outro lado, que a crença em um concreto
independente do abstrato é algo, no mínino, pouco refletido.

Não penso que a verdade aritmética (por exemplo, expressa em uma linguagem
de primeira ordem) seja mais imediata que a demonstração em um sistema
formal, ao contrário; mas também não acho que ela seja algo que não sabemos
o que é, como algumas vezes alguns colocam. Por exemplo, Smullyan afirma que
algumas pessoas dizem “Pelo segundo teorema de Gödel, nunca poderemos saber
se a aritmética é, ou não, consistente”, em função do qual ele se expressa:
Bobagem!. São confusões desse tipo que tentei esclarecer também. Não
pensei que fosse essa a sua posição, mas achei que valia pena apresentar o
argumento aqui na lista.


 
 Há essa também é boa para definir um tipo de platonismo: defina a
 Aritmética
 como sendo o conjunto das fórmulas que são verdadeiras no modelo
 apresentado
 acima. Nesse caso, a Aritmética não é (recursivamente) axiomatizável. E ela
 são as verdades eternas...
 --

 É, isso é o que eu conheco como true arithmetic. Já que estavamos falando
 sobre traducoes para o portugues, alguem sabe qual é a melhor ou mais usada
 traducao pra true arithmetic??

 
 Tem mais sobre essa discussão em minha tese:

 http://www.marilia.unesp.br/Home/Instituicao/Docentes/RicardoTassinari/Bibli
 otecaOnline/Tese%20Tassinari.pdfhttp://www.marilia.unesp.br/Home/Instituicao/Docentes/RicardoTassinari/Bibli%0AotecaOnline/Tese%20Tassinari.pdf
 .
 -

 Obrigado pela tese! Já fiz o download, mas vou ler só depois de alguns
 meses, quando eu terminar de escrever a minha... :-)

 Até!

 Bruno


Um grande abraço e obrigado pela discussão.
Ricardo.


 
 Bruno Woltzenlogel Paleo
 Website: http://www.logic.at/people/bruno/




-- 
Dr. Ricardo Pereira Tassinari - Departamento de Filosofia
UNESP - Faculdade de Filosofia e Ciências - Marília
Homepage: http://www.marilia.unesp.br/ricardotassinari
___
Logica-l mailing list
Logica-l@dimap.ufrn.br
http://www.dimap.ufrn.br/cgi-bin/mailman/listinfo/logica-l


Re: [Logica-l] Falecimento de M. S. Lourenço

2009-08-11 Por tôpico Ricardo Pereira Tassinari
 Lourenço. A incapacidade
 de reconhecer quem tanto nos deu é uma falta moral grave, e poderá parecer
 que esse é o meu caso, pois não cheguei a ter oportunidade de lhe dedicar um
 livro, como fiz a Adriana Silva Graça, que me ensinou lógica. Isto aconteceu
 porque nunca compreendi praticamente coisa alguma das aulas teóricas de
 lógica do Manel Lourenço — quem realmente me ensinou lógica foi a Adriana,
 de uma maneira maravilhosa, e por isso lhe dediquei o meu livro O Lugar da
 Lógica na Filosofia. As aulas teóricas do Manel Lourenço eram cativantes,
 mas ininteligíveis para um aluno, mesmo um aluno como eu que acabaria por
 ter nota máxima na disciplina; na verdade, os outros alunos deixavam até de
 ir às suas aulas e assistiam apenas às aulas da Adriana. Contudo, com o
 Manel Lourenço aprendi a precisão do raciocínio e a importância do estilo,
 não nas aulas, mas pelos seus escritos. Espero vir a ter oportunidade de
 dedicar um livro ao Manel Lourenço.

 Na minha casa de Lisboa tenho alguns livros do Manel Lourenço, como a
 Espontaneidade da Razão e Os Degraus do Parnaso. Assim que tiver
 oportunidade, tentarei publicar alguns excertos na Crítica, para que mais
 pessoas possam ganhar com o legado impressionante do meu querido professor
 M. S. Lourenço, extemporaneamente falecido aos 73 anos.

 --
 Quer uma internet mais segura? Baixe agora o novo Internet Explorer 8. É
 grátis!http://brasil.microsoft.com.br/IE8/mergulhe/?utm_source=MSN%3BHotmailutm_medium=Taglineutm_campaign=IE8

 ___
 Logica-l mailing list
 Logica-l@dimap.ufrn.br
 http://www.dimap.ufrn.br/cgi-bin/mailman/listinfo/logica-l




-- 
Dr. Ricardo Pereira Tassinari - Departamento de Filosofia
UNESP - Faculdade de Filosofia e Ciências - Marília
Homepage: http://www.marilia.unesp.br/ricardotassinari
___
Logica-l mailing list
Logica-l@dimap.ufrn.br
http://www.dimap.ufrn.br/cgi-bin/mailman/listinfo/logica-l


Re: [Logica-l] 2 problemas de tradução, e a fixa ção de terminologia

2009-08-11 Por tôpico Ricardo Pereira Tassinari
Olá a todos.

Começando pela sugestão do Jõao Marcos de usar demonstr ao invés de
prov, eu aprovo! Tenho tentado manter sempre (já tinha ouvido o Jairo
falar sobre isso e acabei adotando), apesar de sempre acabar usando Teoria
da Prova e não Teoria da Demonstração.

Quanto à incompletude, foi bem lembrado os dois tipos:

Sintática: se existe uma fórmula A tal que nem A nem ~A são demonstráveis no
sistema;
Semântica: se existe uma fórmula válida que não é teorema do sistema (tem
também a versão forte: existe uma fórmula A e um conjunto de fórmulas C tal
que A é conseqüência semântica de C, mas A não é deduzida no sistema a
partir de C).

Quanto ao termo incompletabilidade sugerido pelo João, ou incompletável,
tem-se que ver se é no sentido sintático ou semântico. No caso do Teorema de
Gödel, é semântico (em relação ao Modelo Padrão ou outro isomorfo).
Não me é claro que é impossível encontrar uma extensão de certas teorias
aritméticas axiomáticas que seja sintaticamente completa (mas é claro que
essa extensão não será mais correta em relação ao Modelo Padrão).

De qualquer modo, acho boa a noção de incompletabilidade.

Bem, eu também gosto do termo Metademonstração quando se trata de uma
demonstração que não é feito no(s) sistema(s) formal(is) mas sobre esse(s)
sistema(s).

Ficamos assim com Primeiro Metateorema do Incompletamento de Gödel?! ;)

Isso não é um tanto quanto bárbaro? :))

Abraços.
Ricardo.

2009/8/10 Bruno Woltzenlogel Paleo bruno.wp.mailingl...@googlemail.com

 Olá,

 
 (1) os teoremas de Gödel
 São mesmo teoremas de incompletude?  Parece que neste caso o próprio
 Gödel é responsável pela má escolha do termo incompleteness, em
 inglês, dando suporte à tradução do seu artigo feita por van
 Heijenoort.
 

 Vale lembrar também que o titulo original do paper foi Über formal
 unentscheidbare Sätze der Principia Mathematica und verwandter Systeme I.

 Unentscheidbar traduz-se como indecidivel...  Ou seja, nao é possivel
 'decidir' G, no sentido de que nem G nem not G sao demonstraveis...

 A partir daí vem uma nocao de completude para teorias. Uma teoria é
 definida
 como completa se, para toda sentenca F, ou F ou not F pertencem a
 teoria (i.e. sao teoremas).
 Aí fica facil ver como o teorema de Gödel sobre sentencas indecidiveis
 acaba
 virando um teorema sobre incompletude de teorias. (nao me lembro se isso já
 é feito no proprio paper do Gödel, ou se só foi feito depois.)

 Depois surge uma outra nocao de incompletude um pouco diferente: um calculo
 de demontracoes (proof calculus) C é completo se e somente se, se uma
 sentenca F é valida, entao existe uma prova de F em C.
 Entao também decorre do teorema de Gödel que nao há um calculo completo com
 relacao à interpretacao padrao da linguagem da aritmetica. Ou seja, existem
 sentencas que sao verdadeiras no modelo padrao da Aritmetica, mas que nao
 sao demonstraveis...  Essa consequencia do teorema de Gödel acabou se
 popularizando bem mais que o teorema em si... (na epoca do paper do Gödel,
 essas distincoes entre verdade, demonstrabilidade, decibilidade ainda nem
 estavam tao claras...)


 Aproveitando, gostaria de perguntar algo àqueles que entendem de logicas
 paraconsistentes:

 O teorema de Gödel pode ser enunciado aproximadamente assim: nao existe
 nenhuma teoria T extendendo a teoria minima da aritmetica Q, que seja
 simultaneamente (recursivamente) axiomatizavel, (omega)-consistente e
 completa

 Como fica isso do ponto de vista paraconsistente? Será que é possivel
 provar
 que existe uma teoria paraconsistente T' que seja completa, axiomatizavel,
 inconsistente, mas 'paraconsistente'? Ou existe uma versao paraconsistente
 do teorema de Gödel?  Isso já foi discutido em algum paper? Estou viajando
 demais :-) ?


 Até...

 Bruno

 
 Bruno Woltzenlogel Paleo
 Website: http://www.logic.at/people/bruno/

 ___
 Logica-l mailing list
 Logica-l@dimap.ufrn.br
 http://www.dimap.ufrn.br/cgi-bin/mailman/listinfo/logica-l




-- 
Dr. Ricardo Pereira Tassinari - Departamento de Filosofia
UNESP - Faculdade de Filosofia e Ciências - Marília
Homepage: http://www.marilia.unesp.br/ricardotassinari
___
Logica-l mailing list
Logica-l@dimap.ufrn.br
http://www.dimap.ufrn.br/cgi-bin/mailman/listinfo/logica-l


Re: [Logica-l] Paul Watzlawick, o teorema de Gödel e o Tractatus de Wittgenstein

2009-08-07 Por tôpico Ricardo Pereira Tassinari
 podem dizer em que
 consiste esse sentido?)
 Existe, sem dúvida, o inexpressável. Este *mostra-se* a si mesmo, é o
 místico...
 Do que não podemos falar, devemos guardar silêncio.


Guardemos silêncio então!

Ops, mas eu não sou wittgensteiniano. Ok então, não preciso guardar
silêncio.


 --
 Conheça os novos produtos Windows Live. Clique 
 aqui!http://www.windowslive.com.br

 ___
 Logica-l mailing list
 Logica-l@dimap.ufrn.br
 http://www.dimap.ufrn.br/cgi-bin/mailman/listinfo/logica-l


Abraços e peço desculpa por algumas brincadeiras.
Ricardo.
-- 
Dr. Ricardo Pereira Tassinari - Departamento de Filosofia
UNESP - Faculdade de Filosofia e Ciências - Marília
Homepage: http://www.marilia.unesp.br/ricardotassinari
___
Logica-l mailing list
Logica-l@dimap.ufrn.br
http://www.dimap.ufrn.br/cgi-bin/mailman/listinfo/logica-l


Re: [Logica-l] Hélio Schwartsman x Deus: 1 x 0

2009-07-26 Por tôpico Ricardo Pereira Tassinari
Olá Márcio.

Gostei muito da citação do Carl Sagan que está na assinatura eletrônica de
seu e-mail, você poderia por favor me passar a referência bibliográfica?

Abraço,
Ricardo.

2009/7/26 Márcio Palmares marciopalma...@gmail.com


 2009/7/26 Frank Thomas Sautter ftsaut...@gmail.com

 João Marcos,
 ótima a indicação das colunas do Hélio, felizmente ele está de volta após
 um período de ausência.


 Os argumentos são idênticos aos do Richard Dawkins (não são originais), mas
 vale a pena pela coragem intelectual do Hélio:

 Avacalhação Oportuna
 http://www1.folha.uol.com.br/folha/pensata/ult510u309530.shtml

 []'s

 M.

 --
 We are a way for the Cosmos to know itself. (Carl Sagan)

 ___
 Logica-l mailing list
 Logica-l@dimap.ufrn.br
 http://www.dimap.ufrn.br/cgi-bin/mailman/listinfo/logica-l




-- 
Dr. Ricardo Pereira Tassinari - Departamento de Filosofia
UNESP - Faculdade de Filosofia e Ciências - Marília
Homepage: http://www.marilia.unesp.br/ricardotassinari
___
Logica-l mailing list
Logica-l@dimap.ufrn.br
http://www.dimap.ufrn.br/cgi-bin/mailman/listinfo/logica-l


Re: [Logica-l] [off] Does science make belief in God obsolete?

2009-07-23 Por tôpico Ricardo Pereira Tassinari
Olá Adolfo.

Eu gostei muito! Só vi os títulos, as descrições (e as fotos!), mas vou ler
com certeza.

Grato,
Ricardo.

2009/7/23 Adolfo Neto ado...@utfpr.edu.br

 Não li, mas acho que pode interessar a alguns desta lista...

 Does science make belief in God obsolete?
 Thirteen views on the question
 http://www.templeton.org/belief/essays/essays.pdf
 Online at www.templeton.org/belief

 []s
 Adolfo

 ==
 Adolfo Neto
 Departamento Acadêmico de Informática
 Universidade Tecnológica Federal do Paraná
 Fone: (41) 3310-4644 / Fax: (41) 3310-4646
 Web: 
 http://www.dainf.ct.utfpr.edu.br/~adolfohttp://www.dainf.ct.utfpr.edu.br/%7Eadolfo
 Blog: http://professoradolfo.blogspot.com
 ==


 ___
 Logica-l mailing list
 Logica-l@dimap.ufrn.br
 http://www.dimap.ufrn.br/cgi-bin/mailman/listinfo/logica-l




-- 
Dr. Ricardo Pereira Tassinari - Departamento de Filosofia
UNESP - Faculdade de Filosofia e Ciências - Marília
Homepage: http://www.marilia.unesp.br/ricardotassinari
___
Logica-l mailing list
Logica-l@dimap.ufrn.br
http://www.dimap.ufrn.br/cgi-bin/mailman/listinfo/logica-l


Re: [Logica-l] Deus de Aristoteles

2009-07-21 Por tôpico Ricardo Pereira Tassinari
Olá Amauri.

Não sei se devemos considerar o Deus de Aristóteles (o Primeiro Motor
Imóvel) mecânico.

Primeiramente, porque a noção de movimento em Aristóteles é entendida como
passagem da potência ao ato. Assim, movimento/mudança/transformação é a
manifestação de algo que já estava em potência.

Há em Aristóteles a causa final, além das causas material, formal e efetiva:
ou seja, algo tende a um fim, justamente porque aquele é o fim do algo,
diferente de como entendemos as coisas na mecânica atualmente.

Em Aristóteles a vida tem fins em si mesmos.

E, por fim, talvez o mais importante para a sua questão, temos que
considerar o desejável, a intelecção e o inteligível e suas relações com o
1º Motor.

Bem, para entender melhor essas relações e, mesmo o que é o 1º Motor, é
indispensável a leitura do Livro X da Metafísica (acho que você vai gostar
da leitura, já que está interessado nessa questão; leve em consideração os
comentários acima).

Mas, para esboçar uma idéia geral do que penso, Aristóteles diz lá no Livro
X que: a inteligência se entende a si mesma e o pensamento é o pensamento do
pensamento; que a intelecção de si mesma realiza a inteligência em um todo
que dura eternamente; que as coisas estão ordenadas reciprocamente, que
todos os seres devem necessariamente discriminar-se entre si mutuamente e
todos em suas funções distintas colaboram para uma conservação do Universo.

Note então que essa ordenação se expressa como ordenação lógica, salientando
que, em Aristóteles não temos a noção de lógica como hoje; a lógica é um
instrumento para a Ciência, que diz daquilo que é, sendo que a lógica está
expressa nos livros do Organon (cf. minha nota de aula
http://polo1.marilia.unesp.br/Home/Instituicao/Docentes/RicardoTassinari/LII8.pdf
).

Mais ainda, há definições essenciais que expressam qualidades essenciais aos
seres, como, por exemplo, o homem é um animal (gênero) racional (espécie,
diferença específica). Assim, a essência é uma propriedade que faz com que a
coisa seja aquilo que ela é e a ordenação dessas propriedades é uma
ordenação lógica, chegando, por exemplo, nos silogismos,a obedecer uma
lógica das classes.

Assim, há uma ordenação lógica do mundo, já que a Ciência é conhecimento
daquilo que é e a lógica serve a Ciência e a ordenação do mundo, ao o
inteligível, segundo gêneros (e classes), que se usam da lógica.

Por fim, como fim supremo do desejável, temos o belo, de forma que no 1º
Motor há a contemplação eterna do Belo e do inteligível eterno.

Nesse sentido acho que não é muito correto considerar o Deus de Aristóteles
mecânico.

Abraço,
Ricardo.

2009/7/20 Amauri N. Sanches Jr amau...@gmail.com

 Olá Ricardo

 A pergunta é se o deus mecanico de Aristoteles poderia ser considerado
 lógico

 abraços
 Amauri

 2009/7/20 Ricardo Pereira Tassinari rica...@cle.unicamp.br

 Olá Amauri.

 Não entendi bem sua pergunta.

 Abraço,
 Ricardo.

 2009/7/18 Amauri Jr amau...@gmail.com

 Olá a todos


 Existe alguma realação logica dentro DA idéia de Aristóteles?

 Abraços
 Amauri
 ___
 Logica-l mailing list
 Logica-l@dimap.ufrn.br
 http://www.dimap.ufrn.br/cgi-bin/mailman/listinfo/logica-l




 --
 Dr. Ricardo Pereira Tassinari - Departamento de Filosofia
 UNESP - Faculdade de Filosofia e Ciências - Marília
 Homepage: http://www.marilia.unesp.br/ricardotassinari




 --
 Sds

 [o]Amauri[o]

 Meu blog:
 http://masterofdarkness.multiply.com/journal

 Se vc não tem atitude, mantenha pelo menos a palavra!

 (Quem quer que haja construído um novo céu, só no seu próprio inferno
 encontrou energia para fazê-lo! Nietszche)

 Não importa o que fizeram a você.
 O que importa é o que você faz com
 aquilo que fizeram para você.

 Jean-Paul Sartre (1905-1980)
   Filósofo francês

 ___
 Logica-l mailing list
 Logica-l@dimap.ufrn.br
 http://www.dimap.ufrn.br/cgi-bin/mailman/listinfo/logica-l




-- 
Dr. Ricardo Pereira Tassinari - Departamento de Filosofia
UNESP - Faculdade de Filosofia e Ciências - Marília
Homepage: http://www.marilia.unesp.br/ricardotassinari
___
Logica-l mailing list
Logica-l@dimap.ufrn.br
http://www.dimap.ufrn.br/cgi-bin/mailman/listinfo/logica-l


Re: [Logica-l] O Problema da Parada

2009-07-13 Por tôpico Ricardo Pereira Tassinari
Olá Doria e Marcelo.

2009/7/13 Francisco Antonio Doria famado...@gmail.com

 A brincadeira do Marcelo mostra que não expliquei tudo sobre o algoritmo de
 Kunen. Mea culpa. Vai aqui, então.

 - Pode-se provar a consistência de PA? Sim. Você parte dos axiomas de PA e,
 numa etapa crucial, introduz uma indução transfinita a \epsilon_0, como um
 axioma extra. Daí sai Consis PA. (Há outros modos de fazê-lo.)


Parece haver uma explicação simples para isso. Desde que haja uma
demonstração em uma teoria (recursivamente) axiomatizada, então existe uma
máquina que pode realizá-la, o que está relacionado à visão de que as
teorias formais são como máquinas de demonstrar teoremas.



 - Kunen mostra como provar Paris-Harrington. Parte dos axiomas de PA, usa
 num determinado momento a indução transfinita, e prova PH. Como PA prova PH
 --- Consis PA, tira-se por modus ponens Consis PA.

 - A indução transfinita é a coisa. Fazê-la envolve o uso de um ordinal
 construtivo simples. A operação em causa é recursiva. Pode-se então
 mecanizar tudo.

 Qual o galho? PA não prova que este algoritmo converge adequadamente. ``Do
 lado de fora,'' meta, a gente vê que há a convergência, mas PA não consegue
 fazê-lo.


Fazendo uma digressão: se somos máquinas e vemos que a Aritmética (por
exemplo, PA) é consistente, será que é porque temos a indução transfinita em
nossa programação. :))

Abraços,
Ricardo.



 2009/7/13 Francisco Antonio Doria famado...@gmail.com

 No comment :))

 2009/7/13 Marcelo Finger mfin...@ime.usp.br

 Dória.

 2009/7/13 Francisco Antonio Doria famado...@gmail.com

 A gente tem que tomar muito cuidado com esses resultados de
 indecidibilidade e incompletude, porque frequentemente seu alcance é
 exagerado ou confundido. Por exemplo: o segundo teorema de incompletude de
 Gödel diz que (slight handwaving...) se PA é consistente, então a sentença
 de Gödel que diz da sua consistência não pode nem ser provada nem
 desprovada.

 Ora, em 1995 Ken Kunen publicou um algoritmo que, trivialmente, prova
 Consis PA. Vou repetir: ***Um algoritmo que prova Consis PA***. Como pode?


 Em Java ficaria assim:

 public static main( String []args) {
 System.out.println(A Aritmética de Peano é decidível?);
 System.out.println(Sim);
 }

 :-


 Outra coisa: é trivial (menos, menos, mas quase...) mostrar que, dado um
 conjunto finito de instâncias sendo investigadas para o Problema da Parada,
 ***existe*** um algoritmo que as decide. (Ênfase no ***existe***.)


 Uéu.  Aqui v precisa ordernar os N algoritmos na mão, dar para alguém
 mostrar se eles páram ou não e criar um vetor de N posições chamado de
 VEREDITO.  Então, v faz um algoritmo assim:

 Entrada: TEXT: string contendo um dos N algoritmos contemplados
 Saída: veredito

 if( TEXT.equals(algo0) )
 System.out.println( VEREDITO[0]);
 else if( TEXT.equals(algo1) )
 System.out.println( VEREDITO[1]);

 etc;

 Fácil, né!



 Mais complicado (mas factível) é explicitar tal (tais) algoritmo(s).


 Feito!

 ;-)




 --
 Marcelo Finger
 Departamento de Ciencia da Computacao
 Instituto de Matematica e Estatistica
 Universidade de Sao Paulo
 Rua do Matao, 1010
 05508-090Sao Paulo, SP Brazil
 Tel: +55 11 3091-9688, 3091-6135, 3091-6134 (fax)
 http://www.ime.usp.br/~mfinger http://www.ime.usp.br/%7Emfinger




 ___
 Logica-l mailing list
 Logica-l@dimap.ufrn.br
 http://www.dimap.ufrn.br/cgi-bin/mailman/listinfo/logica-l




-- 
Dr. Ricardo Pereira Tassinari - Departamento de Filosofia
UNESP - Faculdade de Filosofia e Ciências - Marília
Homepage: http://www.marilia.unesp.br/ricardotassinari
___
Logica-l mailing list
Logica-l@dimap.ufrn.br
http://www.dimap.ufrn.br/cgi-bin/mailman/listinfo/logica-l


Re: [Logica-l] O problema da parada

2009-06-05 Por tôpico Ricardo Pereira Tassinari
Olá Doria e Newton.

Vejo uma ligação direta entre o que vocês escreveram.

Criar a teoria PA+{x1,...xn}, no qual x1, ..., xn são as sentenças que PA
não prova, é equivalente a inserir com a mão as decisões de caso de parada
que PA não decide, como os algorítimos que o Newton citou, mas, no caso
dele, ele insere *todas* as respostas com a mão.

Ricardo.


2009/6/5 Francisco Antonio Doria famado...@gmail.com

 Alguma coisa tá complicada na comunicação.

 O que, em princípio, bloqueia algoritmos para resolver o problema da parada
 é que existem algoritmos parciais; algoritmos que entram em loop infinito.

 Agora: cada caso onde o algoritmo diverge no modelo standard é dada por uma
 sentença ∏1. Logo, se PA for sound, ou PA prova essa sentença x, ou PA + x a
 prova, no caso trivialmente. A teoria PA + x é uma teoria que inclui os
 teoremas de PA, tem sua mesma linguagem, e a mesma ``força de prova.''

 Não dá pra enumerar instâncias finitas, pois há algoritmos onde os pontos
 de divergência são em número infinito.

 2009/6/5 Newton José Vieira nvie...@dcc.ufmg.br

 Dória e amigos,

 Entendi perfeitamente que estava falando do problema da parada.

 Seja P um conjunto finito de programas e E um conjunto finito de entradas
 possíveis para programas. Então o problema de determinar se o programa p
 pára se sua entrada for x, para qualquer par (p,x) em P x E é *obviamente*
 decidível: se P x E tem n elementos, existem 2^n tabelas de n respostas
 sim/não, apenas uma delas com todas as respostas corretas e cada uma das
 restantes 2^n-1 tabelas com alguma resposta incorreta. Assim, dos 2^n
 algoritmos baseados no uso das tabelas para resolver o problema, apenas um é
 correto; os outros 2^n-1 são incorretos. O algoritmo correto existe, mesmo
 que não se saiba qual é. Costumo brincar com os alunos que o seguinte
 problema é decidível (supondo que a resposta só possa ser sim ou não): *Deus
 existe?* . Como ele tem apenas uma instância, existem 21=2 algoritmos:
  A1: retorne sim.
  A2: retorne não.
 Embora não saibamos qual, um deles é o correto!

 Em resumo: qualquer problema de decisão constituído de um conjunto finito
 de instâncias (incluindo qualquer subconjunto finito de instâncias do
 problema da parada) é decidível. Ou seja, o fato de um subconjunto *finito*
 de instâncias do problema da parada ser decidível não depende do fato de que
 ele seja constituído de instâncias do *problema da parada*!

 Peço desculpas se não fui claro da outra vez ou se continuo sem entender o
 que disse...

 Newton.

 Francisco Antonio Doria escreveu:

 Não, é o halting problem, em ciência da computação, para meaquinas de
 Turing. Decidir onde há divergências é a questão. O teorema de Turing é:

 Para uma máquina de Turing qualquer, e um input arbitrário, não há um
 algoritmo geral que decida se a máquina para ou não.

 2009/6/4 Newton José Vieira nvie...@dcc.ufmg.br mailto:
 nvie...@dcc.ufmg.br

Prezado Dória,

Tenho  ficado encucado com o que disse, lendo e relendo para ver
onde é que entendi errado... Para mim é óbvio que todo problema de
decisão com conjunto finito de instâncias é decidível (se n é o
número de instâncias, um algoritmo para o problema é aquele que
consulta a tabela de respostas corretas dentre as 2^n tabelas
possíveis). Você acha que isso é pouco percebido em geral ou quiz
dizer alguma outra coisa?

Newton.

Francisco Antonio Doria escreveu:

Não existe nenhum algoritmo capaz de resolver todas as
instâncias do Problema da Parada, mas - fato pouco percebido -
é que, dado um conjunto finito de tais instâncias, podemos
nesse caso particular escrever sempre um algoritmo que resolva
o problema específico. Não dá é para fazer um algoritmo
global; entre outras coisas porque a complexidade
computacional cresce sem limite.

  



___
Logica-l mailing list
Logica-l@dimap.ufrn.br mailto:Logica-l@dimap.ufrn.br
http://www.dimap.ufrn.br/cgi-bin/mailman/listinfo/logica-l


--You don't really understand something if you only understand it
 in
one way.
Marvin Minsky




 --
 You don't really understand something if you only understand it in one
 way.
 Marvin Minsky



 ___
 Logica-l mailing list
 Logica-l@dimap.ufrn.br
 http://www.dimap.ufrn.br/cgi-bin/mailman/listinfo/logica-l




-- 
Dr. Ricardo Pereira Tassinari - Departamento de Filosofia
UNESP - Faculdade de Filosofia e Ciências - Marília
Homepage: http://www.marilia.unesp.br/ricardotassinari
___
Logica-l mailing list
Logica-l@dimap.ufrn.br
http://www.dimap.ufrn.br/cgi-bin/mailman/listinfo/logica-l


[Logica-l] Frege Revolucionário e Teste de Pater nidade

2009-01-26 Por tôpico Ricardo Pereira Tassinari
Olá a todos.

Se não interessante, pelo menos volumosa essa discussão sobre a revolução
fregeana e paternidade da Lógica Moderna.

Fica em minha mente as seguintes questões.

Qual a importância (histórica e conceitual) dos *Principia Mathematica* para
a Lógica Moderna?

Qual a importância (histórica e conceitual) da obra de Frege para a
elaboração dos *Principia Mathematica*?

Vale a transitividade da importância (da histórica e da conceitual)? Isto é
se A é importante (historicamente ou conceitualmente) para B e B é
importante (historicamente ou conceitualmente) para C, então A é importante
(historicamente ou conceitualmente) para B?

Abraços a todos.
Ricardo.

-- 
Dr. Ricardo Pereira Tassinari - Departamento de Filosofia
UNESP - Faculdade de Filosofia e Ciências - Marília
Homepage: http://www.marilia.unesp.br/ricardotassinari
___
Logica-l mailing list
Logica-l@dimap.ufrn.br
http://www.dimap.ufrn.br/cgi-bin/mailman/listinfo/logica-l


[Logica-l] Unicidade da Lógica e Unicidade da Fi losofia

2008-10-05 Por tôpico Ricardo Pereira Tassinari
Olá a todos.

Não leio e-mail faz 2,5 dias, só agora estou retomando as discussões.

Primeiro. O problema para o intuicionista da demonstração que apresentei,
pelo que posso entender são basicamente dois:

(1) Usa como hipótese Av~A, ou melhor, Rac(s^s)V~Rac(s^s); o que me parece
não é aceito por um intucionista originário;

Arthur: é verdade que um clássico poderia falar ao intuicionista que quis
dizer ~(A~A) (com a tradução de Gödel), ou seja,  ~(Rac(s^s)V~Rac(s^s)); de
qualquer modo os dois continuam não se entendendo quanto ao uso do conectivo
'ou', não pensam da mesma forma sobre esse conectivo; mas admito que há essa
equivalência entre as duas formas de pensar (i.e., considerando a tradução,
como eu poderia negar!:-) mas então porque dizer haver uma única lógica, a
qual é a lógica intuicionista, principalmente considerando o problema (2)
abaixo?

(2) Demonstra a existência de dois números irracionais sem verdadeiramente
exibi-los, o que, novamente, parece-me não ser aceito por um intuicionista
originário.

Poderia ser dado exemplos de outras lógicas; porém, bem entendido, o
problema que estou colocando é o da UNICIDADE DA LÓGICA.

Tendo a concordar com William Steinle de que uma pluralidade de lógicas
não-clássicas mostra que essas são, ao que tudo indica, possíveis, e se são
possíveis, não há motivos, a não ser pessoais, para sustentarmos que existe
apenas uma lógica. Porém, para mim, isso não é uma conclusão (assim, não
usa uma lógica) é um princípio.

A questão que podemos tirar daí, penso é: admitindo que há várias lógicas e
que uma filosofia pressupõe uma lógica, como não admitir que há várias
filosofias?

Abraço a todos
Ricardo.

PS.
Daniel: sem recusar as disjunções, responda francamente: você continua ou
não batendo na Paula?


-- 
Dr. Ricardo Pereira Tassinari - Departamento de Filosofia
UNESP - Faculdade de Filosofia e Ciências - Marília
Homepage: http://www.marilia.unesp.br/ricardotassinari
___
Logica-l mailing list
Logica-l@dimap.ufrn.br
http://www.dimap.ufrn.br/cgi-bin/mailman/listinfo/logica-l


Re: [Logica-l] A cura pela paraconsistência

2008-10-01 Por tôpico Ricardo Pereira Tassinari
Opa Décio!

Nesse caso então o Edson tem razão. Pois como ele afirmou na entrevista que
A e depois disse ~A, segue-se que A!

Abraços.
R.

P.S.: Apesar de seguir ~A também; mas isso é mero detalhe já que o Edson
queria provar que A.


2008/10/1 Décio Krause [EMAIL PROTECTED]

 Você não conhece a nossa gente, pelo jeito. Abraços.
 D.

 2008/10/1 Décio Krause [EMAIL PROTECTED]

  To lista:
  Não precisa, Edson. Newton é paraconsistente.
 D.

  2008/10/1 Edson Dognaldo Gil [EMAIL PROTECTED]

 Ele nega ter dado a entrevista ou nega ter dito o que consta na

 entrevista ou afirma ter mudado de opinião? Em qualquer desses casos,
 seria interessante que ele entrasse em contato com a revista para
 acertar as coisas, não acha? Devia colocar também uma nota de pé de
 página na próxima edição do livro. Mas note que não se diz que a
 paraconsistência poderia curar doenças psicológicas, mas que a busca
 da cura foi o principal motivo inicial de suas investigações lógicas
 [em segundo lugar, o marxismo e em terceiro, a lógica propriamente
 dita]. Agora, definitivamente, esgotei o que tinha a dizer. Abraço,
 edg

 2008/10/1 Francisco Antonio Doria [EMAIL PROTECTED]:
  Falei com o Newton: non è vero.
 ___
 Logica-l mailing list
 Logica-l@dimap.ufrn.br
 http://www.dimap.ufrn.br/cgi-bin/mailman/listinfo/logica-l




 --
 _
 Décio Krause
 Departamento de Filosofia
 Universidade Federal de Santa Catarina
 C.P. 476
 88040-900 Florianópolis, SC - Brasil
 Tel.: +(48)3331-9248
 www.cfh.ufsc.br/~dkrause http://www.cfh.ufsc.br/%7Edkrause
 Grupo de Lógica e Fundamentos da Ciência
 www.logica.cfh.ufsc.br
 _




 --
 _
 Décio Krause
 Departamento de Filosofia
 Universidade Federal de Santa Catarina
 C.P. 476
 88040-900 Florianópolis, SC - Brasil
 Tel.: +(48)3331-9248
 www.cfh.ufsc.br/~dkrause http://www.cfh.ufsc.br/%7Edkrause
 Grupo de Lógica e Fundamentos da Ciência
 www.logica.cfh.ufsc.br
 _

 ___
 Logica-l mailing list
 Logica-l@dimap.ufrn.br
 http://www.dimap.ufrn.br/cgi-bin/mailman/listinfo/logica-l




-- 
Dr. Ricardo Pereira Tassinari - Departamento de Filosofia
UNESP - Faculdade de Filosofia e Ciências - Marília
Homepage: http://www.marilia.unesp.br/ricardotassinari
___
Logica-l mailing list
Logica-l@dimap.ufrn.br
http://www.dimap.ufrn.br/cgi-bin/mailman/listinfo/logica-l


Re: [Logica-l] American Philosophical Association (APA)

2008-10-01 Por tôpico Ricardo Pereira Tassinari
Parabéns mesmo! Você merece! É um trabalho de qualidade!
R.

2008/10/1 Décio Krause [EMAIL PROTECTED]

 Pessoal
 Já que estamos falando em esforço em pesquisa, permitam-me partilhar com
 todos a boa notícia que acabo de receber do Otávio Bueno (para quem não
 sabe, ex aluno da USP e hoje full professor na Un. Miami): a APA vai fazer
 uma sessão sobre o meu livro com Steven French. Ver abaixo.

 Dear Steven and Décio
 I'm writing to confirm the Author Meets Critics session at the Pacific APA
 meeting on *Identity in Physics*. (The session is sponsored by the Society
 for Realist/Anti-Realist Discussion.) I'm delighted to tell you that the
 critics will be Bas van Fraassen, Don Howard, and me. I'm very much looking
 forward to the session!
 I take it that both of you will be able to attend the meeting, right?
 Needless to say, it will be brilliant to have both of you there! The meeting
 will be on April 8-12, 2009 in Vancouver.
 By the way, when would you need to receive the comments from the critics?
 Thanks again for writing such a great book -- and for taking part in the
 session!
 All the best,
 Otávio
 Para quem não conhece o livro, ver

 http://www.amazon.com/Identity-Physics-Historical-Philosophical-Analysis/dp/0199278245/ref=sr_1_5?ie=UTF8s=booksqid=1222897192sr=8-5

 Abraços,
 D. (UFA!!)


 --
 _
 Décio Krause
 Departamento de Filosofia
 Universidade Federal de Santa Catarina
 C.P. 476
 88040-900 Florianópolis, SC - Brasil
 Tel.: +(48)3331-9248
 www.cfh.ufsc.br/~dkrause http://www.cfh.ufsc.br/%7Edkrause
 Grupo de Lógica e Fundamentos da Ciência
 www.logica.cfh.ufsc.br
 _

 ___
 Logica-l mailing list
 Logica-l@dimap.ufrn.br
 http://www.dimap.ufrn.br/cgi-bin/mailman/listinfo/logica-l




-- 
Dr. Ricardo Pereira Tassinari - Departamento de Filosofia
UNESP - Faculdade de Filosofia e Ciências - Marília
Homepage: http://www.marilia.unesp.br/ricardotassinari
___
Logica-l mailing list
Logica-l@dimap.ufrn.br
http://www.dimap.ufrn.br/cgi-bin/mailman/listinfo/logica-l


Re: [Logica-l] (Com) Lógica

2008-10-01 Por tôpico Ricardo Pereira Tassinari
Olá a todos.

É impressão minha ou Sócrates estava impedido quando fez o Gol?

Abraços.
Ricardo.

2008/10/1 Guilherme Bittencourt [EMAIL PROTECTED]

 Caros,

 já que está faltando falar em futebol (dentro do escopo da filosofia),
 proponho assistir o vídeo abaixo como subsídio para iniciar a discussão.

 Saudações, GB

 http://br.youtube.com/watch?v=moWZm66J_yM



 On Wed, 1 Oct 2008, Ricardo Pereira Tassinari wrote:

  Olá a todos.

 Aproveitando a onda dos temas polêmicos (e ainda não falando de futebol),
 gostaria de perguntar: alguém da lista acha que existe uma única lógica,
 ou
 melhor, uma única forma correta de pensar?

 Abraços.
 Ricardo.

 --
 Dr. Ricardo Pereira Tassinari - Departamento de Filosofia
 UNESP - Faculdade de Filosofia e Ciências - Marília
 Homepage: http://www.marilia.unesp.br/ricardotassinari


 ~~
  
 Guilherme Bittencourt  ignotum per ignotius
  
 Departamento de Automação e SistemasFone: +55 (48) 3721 7665
 Universidade Federal de Santa Catarina  Fax: +55 (48) 3721 9934
 UFSC/CTC/DASE-mail: [EMAIL PROTECTED]
 88040-900 Florianópolis SC Brasil   Internet:
 http://www.das.ufsc.br/~gb http://www.das.ufsc.br/%7Egb




-- 
Dr. Ricardo Pereira Tassinari - Departamento de Filosofia
UNESP - Faculdade de Filosofia e Ciências - Marília
Homepage: http://www.marilia.unesp.br/ricardotassinari
___
Logica-l mailing list
Logica-l@dimap.ufrn.br
http://www.dimap.ufrn.br/cgi-bin/mailman/listinfo/logica-l


Re: [Logica-l] (Com) Lógica

2008-10-01 Por tôpico Ricardo Pereira Tassinari
Olá a todos, principalmente Arthur, Desidério, Edson e Daniel.

Considere a demonstração de que, no conjunto dos números reais existem dois
números irracionais x e y, tal que x^y (x elevado a y) é racional.

Seja x = V2  (raiz de 2).
Sabemos então que x é irracional.
Consideremos então x^x (x elevando a x), temos duas possibilidades:
(1) x^x é racional, daí resolvemos nossa questão;
(2) x^x é irracional. Nesse caso, (x^x) e V2 resolvem a questão, já que
(V2^V2)^V2 = V2^(V2*V2) = V2^2 = 2. (Puxa! Fica uma droga quando não podemos
escrever em duas dimensões.)
Logo, existem dois irracionais que um elevado ao outro resulta um racional.

Arthur: para um matemático em geral, essa demonstração é aceita e não é
intuicionista (já que usamos que A ou não A).

Desidério: Um intuicionista não aceita a demonstração acima, para ele não é
uma forma correta de pensar; um clássico acha que é uma forma correta de
pensar. Então quais das alternativas abaixo você considera correta:
(a) O intuicionista pensa corretamente;
(b) O clássico pensa corretamente;
(c) Nenhum deles pensa corretamente;
(d) Ambos pensam corretamente (como, já que um pensa que o outro não pensa
corretamente?)
(e) Nenhuma das alternativas anteriores (explicar).

Edson: nesse caso o intuicionista pode pensar intuicionisticamente e o
clássico classicamente: apenas ambos não concordam que a forma que o outro
pensa é correta.

Daniel: existem dois números irracionais tais que um elevado a outro é
racional?

Abraços.
R.

2008/10/1 Ricardo Pereira Tassinari [EMAIL PROTECTED]

 Olá a todos.

 Aproveitando a onda dos temas polêmicos (e ainda não falando de futebol),
 gostaria de perguntar: alguém da lista acha que existe uma única lógica, ou
 melhor, uma única forma correta de pensar?

 Abraços.
 Ricardo.

 --
 Dr. Ricardo Pereira Tassinari - Departamento de Filosofia
 UNESP - Faculdade de Filosofia e Ciências - Marília
 Homepage: http://www.marilia.unesp.br/ricardotassinari




-- 
Dr. Ricardo Pereira Tassinari - Departamento de Filosofia
UNESP - Faculdade de Filosofia e Ciências - Marília
Homepage: http://www.marilia.unesp.br/ricardotassinari
___
Logica-l mailing list
Logica-l@dimap.ufrn.br
http://www.dimap.ufrn.br/cgi-bin/mailman/listinfo/logica-l